geometrical optics book with special lecture

264
How to Solve IIT-JEE problems in few seconds? A point object is moving with a speed V before an arrangement of two mirrors as shown in figure. Find the velocity of image in mirror M1with respect to image in mirror M2. a) 2V Sin θ b) 2V Cosθ c) 2V tanθ d) V Sinθ GEOMETRICAL OPTICS Physics for Engineering and Medical Entrance Exam IIT-JEE/AIEEE/AIPMT V12=Velocity of I1with respect to I2 = V1-V2 = 2V Sin θ Lecture notes on How to solve IIT-JEE problems in few seconds? | ANUJ DUBEY

Upload: chdeepak96

Post on 29-Nov-2015

706 views

Category:

Documents


9 download

DESCRIPTION

i

TRANSCRIPT

Page 1: Geometrical Optics Book With Special Lecture

How to Solve IIT-JEE problems in few seconds?

A point object is moving with a speed V before an arrangement of two mirrors as shown

in figure. Find the velocity of image in mirror M1with respect to image in mirror M2.

a) 2V Sin θ b) 2V Cosθ c) 2V tanθ d) V Sinθ

GEOMETRICAL OPTICS Physics for Engineering and Medical Entrance Exam

IIT-JEE/AIEEE/AIPMT

V12=Velocity of I1with respect to I2 = V1-V2 = 2V Sin θ

Lecture notes on How to solve IIT-JEE problems in few seconds?

| ANUJ DUBEY

Page 2: Geometrical Optics Book With Special Lecture

ANUJ PHYSICAL SCIENCES PHYSICS FOR IIT-JEE /AIEEE/AIPMT

Office: 128/652 “Y” Block Kidwai Nagar Kanpur -11 UP, Website: www.apskanpur.in Mob: 09795647531, 09889787531, 0848664308 E-mail:[email protected]

2

GEOMETRICAL OPTICS

Physics for Engineering and Medical entrance exam (IIT-JEE/ AIEEE/AIPMT)

Author:

ANUJ DUBEY

M.Sc. (Physics), Ph.D. (General Relativity) Director: Anuj Physical Sciences, Kanpur

(10 years’ teaching experience of IIT-JEE Physics)

Very little of this book is wholly original. When I drew up notes, I decided from outset that I

would collect together the best approaches to the material known to me. So fact is that

many of approaches in the book have been borrowed from one author or another, there is

a little that I have written completely afresh. My intention has been to organize the

material in such a way that it is the more readily accessible to majority of the students.

Copyright Author:

All rights reserved, No part of this publication may be reproduced in any form or by any

means, electronic, mechanical, and photocopying, without prior permission of author

and the publisher. The author and publisher do not take any legal responsibility for any

errors or misrepresentations that might have crept in.

Developed & Published by:

ANUJ PHYSICAL SCIENCES, KANPUR

Page 3: Geometrical Optics Book With Special Lecture

ANUJ PHYSICAL SCIENCES PHYSICS FOR IIT-JEE /AIEEE/AIPMT

Office: 128/652 “Y” Block Kidwai Nagar Kanpur -11 UP, Website: www.apskanpur.in Mob: 09795647531, 09889787531, 0848664308 E-mail:[email protected]

3

Notes to Students:

I have been involved in teaching IIT –JEE Physics for last 10 years .This Lectures on

“Geometrical optics” is an opportunity to present my experiences. During my interaction

with IIT –JEE aspirants I, realized that most feared topics in Physics is Geometrical optics.

Some of the reasons put forward by students behind this thought were, No spontaneous

thoughts appear after reading a problem, Mind goes blank & cannot t proceed in a

problem. How to proceed in a problem? Which law is applicable, horrible thought appear

in mind?

Total confusion about sign conventions.

How to solve problems based on Variable refractive index

Confusion about Apparent depth & Normal shift Problems

No Proper understandings of Prism theory

No Proper understandings of cut lenses & silvered lenses.

Short cut approach in relative motion for velocity of images problems.

No single book is available that gives large no of solved problems with elaboration

of concepts in a solution.

Probably you think questions set in competitive examinations are very hard and tough to

crack. No they are not. Including AIIMS & IIT-JEE, I have observed that the questions are so

created as to test the clarity of your basic or fundamental concepts. Most of the students

feel that derivations of formulae are of no use in such examinations. These students will

simply memorise the formulae, practise a lot of numerical problems on them and go to the

examination hall. That is exactly where they feel they got tripped. Questions can be and

are framed on the intermediate steps used in deriving the formulae. My main thrust is to

tell the students the way that they should prepare for the various competitive

examinations; their “funda” should be crystal clear. These lectures should help the students

to achieve this aim.That is why keeping the above problems of students in mind I am going

to write a series of Lecture notes on How to solve IIT-JEE problems in few seconds. This

booklet will help the students in building analytical and quantitative skills, addressing key

misconceptions & developing confidence in problem solving skill.

Every effort has been made to check the mistakes & misprints, yet it is very difficult to

claim perfection. Any error, omission and suggestions for the improvement of the book will

be gratefully acknowledged & suggestions for the improvement of the book will be

gratefully and incorporated in the next edition. This booklet is similar to my class room

lectures; you are always welcomed for suggestions & improvement of my ideas & thoughts.

If you find any problems or printing mistakes please email us at [email protected]

or ring me at 09889787531.

Page 4: Geometrical Optics Book With Special Lecture

ANUJ PHYSICAL SCIENCES PHYSICS FOR IIT-JEE /AIEEE/AIPMT

Office: 128/652 “Y” Block Kidwai Nagar Kanpur -11 UP, Website: www.apskanpur.in Mob: 09795647531, 09889787531, 0848664308 E-mail:[email protected]

4

T h a n k s t o ,

Thanks to my respected teachers Prof. Asoke Kumar Sen, Prof. R Bhattacharjee, Dr. A Deshmukhya, Dr. B I Sharma, Dr. H S Das, Dr. P K Shukla, Dr. R T Pandey, Mr. Ravindra Yadaw & Mr. Ravindra Bajpai, Dr. R K Dwivedi, Dr. J N Sharma, Dr. S P Singh, Dr Mahendra Kumar, Dr. Rajesh Kumar, Dr. H Bisht, Dr. Sandeep Yadaw, who always inspired me during my academic and teaching career and played an indispensible role in my journey of learning Physics.

I thank to my friends & faculty Dr. Ved Prakash Tiwari, Vikash Sisodia,

Narendra Pratap Singh, Rakesh Verma, Amarpal Singh, Amritaksh Kar,

Tanay Ghosh and Amarkant Sharma for his invaluable ideas and kind

suggestions.

A special thanks to Poonam, Astha, Anoop and Ragini who checked, improved

and presented my classroom lectures in such format and spent a large share

of his invaluable time with me and made me feel the real taste of writing the

lecture notes.

It is with a deep sense of gratitude and pleasure that I acknowledge my

indebtedness to my students, Foyez Ahmad Laskar, Arindom Das, Deopriyo

Dey, Rajneel Bhattacharjee, Sayan Paul, Sanjeeda Laskar, Kausik, Jaynta,

Shamil, Suradeep, Sagarika Barman, Surabhi Tomar & others for all the

discussions and questions they have raised which results in this format of

lecture notes on Geometrical optics.

Above all, I thank my lord SHIVA who has given me, wisdom knowledge and

guidance throughout my life.

Page 5: Geometrical Optics Book With Special Lecture

ANUJ PHYSICAL SCIENCES PHYSICS FOR IIT-JEE /AIEEE/AIPMT

Office: 128/652 “Y” Block Kidwai Nagar Kanpur -11 UP, Website: www.apskanpur.in Mob: 09795647531, 09889787531, 0848664308 E-mail:[email protected]

5

Contents:

Lecture-1: Basics of optics: Various theories to understand the nature of light, Optical path &

Fermat’s Principle, Derivation of Law of Reflection & Refraction by Fermat’s principle, Deviation in

plane mirror, Rotation of mirror, Images formed by two plane mirrors in contact, Formation of

images by plane mirror, Minimum size of plane mirror to see full height of a man & a building

behind the man, Power of an optical instrument.

Lecture-2: Laws of Reflection & Refraction: Vector form of Laws of Reflection, Problem based

on Snell’s Law, Variable Refractive Index Problems.

Lecture-3: Reflection from spherical mirrors: Formulas related to mirrors & Magnification,

Relative position, size and nature of image for different positions of object in spherical mirror.

Lecture-4: Velocity of image: in the Plane mirror & Spherical mirror.

Lecture-5: Refraction at Plane Surfaces (I): Apparent Depth & Normal Shift.

Lecture-6: Refraction at Plane Surfaces (II): Critical Angle, Total Internal Reflection.

Lecture-7: Prism Theory: Refraction through prism, Deviation in prism, Maximum deviation in

Prism, Minimum deviation in prism, Condition of no emergence: (TIR) in prism, Dispersion of light

& Causes of dispersion, Angular dispersion & Dispersive power, Dispersion without deviation,

Deviation without dispersion.

Lecture-8: Defects of images (Aberration): Chromatic & Monochromatic Abberation

( Spherical ,Coma ,Astigmatism Curvature & Distortion).

Lecture-9: Refraction from curved surfaces (I): Refraction from single curved surface, Lens

makers formula, Lense formula, Displacement methods .

Lecture-10: Refraction from curved surfaces (II): Magnification problems Lenses, Positions,

nature and size of the image for different positions of Object, Formula related two combinations of

thin lenses.

Lecture-11: Refraction from curved surfaces (III):

Cut lense ,Silvered lenses, Combination of lenses & mirrors.

Lecture-12: Defects of Vision & Optical Instrument:

Human eye, Defects of vision, Simple microscope or Magnifier, Compound Microscope,

Astronomical Telescope, Terrestrial Telescope, Galilean Telescope, Reflecting Telescope

(Newtonian & Cassegranian reflecting telescope), Camera & Prism Binocular.

A Special Lecture: On Methods of Extreme Cases

How to solve III-JEE Problems in few seconds?

Page 6: Geometrical Optics Book With Special Lecture

ANUJ PHYSICAL SCIENCES PHYSICS FOR IIT-JEE /AIEEE/AIPMT

Office: 128/652 “Y” Block Kidwai Nagar Kanpur -11 UP, Website: www.apskanpur.in Mob: 09795647531, 09889787531, 0848664308 E-mail:[email protected]

6

Lecture-1

Geometrical Optics

Physics for IIT - JEE

Basics of Optics

Various theories to understand the nature of light

Optical path & Fermat’s Principle

Derivation of Law of Reflection & Refraction by

Fermat’s principle

Deviation in plane mirror

Rotation of mirror

Images formed by two plane mirrors in contact

Formation of images by plane mirror

Minimum size of plane mirror to see full height of a

man & a building behind the man

Power of an optical instrument

Page 7: Geometrical Optics Book With Special Lecture

ANUJ PHYSICAL SCIENCES PHYSICS FOR IIT-JEE /AIEEE/AIPMT

Office: 128/652 “Y” Block Kidwai Nagar Kanpur -11 UP, Website: www.apskanpur.in Mob: 09795647531, 09889787531, 0848664308 E-mail:[email protected]

7

Optics: The study of light’s vision is called optics: Light is a form of energy which

produces the sensation of sight in us.

Geometrical Optics:

Based on rectilinear propagation of light

Valid only If λ Light < < Size of obstacle

Deals with image formation reflection and refraction

Notes: Geometrical optics can be treated as the limiting case of wave optics when size

of obstacle is very much large as compared to wavelength of light under such conditions

the wave nature of light can be ignored and light can be assumed to be travelling in

straight line rectilinear propagation. But when size of obstacle or opening is comparable

to wavelength of light rectilinear propagation no longer valid and resulting

phenomenon are explained by using wave nature of light.

Wave optics:

Light is propagated as wave motion.

It deals with interference, diffraction & polarization.

Optics

Physical optics

Wave optics Quantum

optics

Geometrical optics

Page 8: Geometrical Optics Book With Special Lecture

ANUJ PHYSICAL SCIENCES PHYSICS FOR IIT-JEE /AIEEE/AIPMT

Office: 128/652 “Y” Block Kidwai Nagar Kanpur -11 UP, Website: www.apskanpur.in Mob: 09795647531, 09889787531, 0848664308 E-mail:[email protected]

8

Quantum Optics:

Assumes that light is a stream of particles called photon.

The concept of light particles known as photons is of importance in the study of

origin spectra photoelectric effect, Compton Effect, Radiation pressure & laser.

Newton’s Corpuscular theory:

Light is considered to be a multitude of minute particles.

Explains rectilinear propagation of light reflection of light.

Could not explain refraction speed of light is more in a denser medium than air

which later turned to be wrong. In fact speed of light is maximum in air or

vacuum.

Also could not explain refraction, interference, polarization diffraction.

Hygen’s wave theory:

Maxwell showed that light is an electro-magnetic wave which consist of

oscillation of electric & magnetic vector perpendicular to each other &

perpendicular to direction of propagation.

Explains reflection, refraction, interference, diffraction, and polarisation.

Could not explain photoelectric effect, Compton Effect and several other

phenomenons have associated with emissions& absorption of light.

Various theories to understand Wave/

Particle nature of light

Newton's corpuscular theory

Hygen's wave theory Duality quantum theory

Page 9: Geometrical Optics Book With Special Lecture

ANUJ PHYSICAL SCIENCES PHYSICS FOR IIT-JEE /AIEEE/AIPMT

Office: 128/652 “Y” Block Kidwai Nagar Kanpur -11 UP, Website: www.apskanpur.in Mob: 09795647531, 09889787531, 0848664308 E-mail:[email protected]

9

Duality quantum theory: (Einstein & Plank)

Few phenomenons could not explain by above theorise therefore quantum

theory of light is developed.

At present it believed that light has dual nature propagate as wave and interacts

with matter as a particle.

Note:

Sensitivity

Wavelength 5500Å

The wavelength of light typically varies from 400 nm To 700 nm. The limits of the

visible spectrum are not well defined because the eye sensitivity curve approaches the

axis asymptotically at both long & short wavelength.

Optical path: It is equivalent distance travelled in air or Vacuum.

Optical path = μ d , Where μ =c/v for air or vacuum μ =c/v = c/c =1 Optical path = μ1d1+ μ2d2+ μ3d3+ μ4d4

= ∑

μ1 μ2 μ3 μ4

d1 d2 d3 d4

Page 10: Geometrical Optics Book With Special Lecture

ANUJ PHYSICAL SCIENCES PHYSICS FOR IIT-JEE /AIEEE/AIPMT

Office: 128/652 “Y” Block Kidwai Nagar Kanpur -11 UP, Website: www.apskanpur.in Mob: 09795647531, 09889787531, 0848664308 E-mail:[email protected]

10

Fermat’s principle: (Minimization Principle):

Light must travel along a path in which time is least or distance is

Minimum.

Illustration 1: In figure light starts from point A and after reflection

from the inner surface of the sphere reaches the diametrically opposite

point B. Calculate the length of the hypothetical path APB and using

Fermat’s principle find the actual path of light. Is the path minimum?

A B

P

Φ

2r

Solution: A light signal is switched on from point A. How it will reach

just diametrically opposite point B. Let it moves along the path APB.

L = μ AP + μ PB

L= μ 2r cos ɸ + μ 2r sin ɸ = 2μr (cos ɸ + sin ɸ)

L= L (ɸ) → from minimisation principle

dL /dɸ= 0 = 2μr (- sin ɸ+ cos ɸ) = 0

Sin ɸ = cos ɸ ⇒ tan ɸ=⇒ ɸ =450

Lmin = μ2r cos 450 + μ2r sin 450 = 2√2μr

For air μ=1

Lmin= 2√2r > 2r

Answer: Lmin= 2√2 r, ɸ =450

Page 11: Geometrical Optics Book With Special Lecture

ANUJ PHYSICAL SCIENCES PHYSICS FOR IIT-JEE /AIEEE/AIPMT

Office: 128/652 “Y” Block Kidwai Nagar Kanpur -11 UP, Website: www.apskanpur.in Mob: 09795647531, 09889787531, 0848664308 E-mail:[email protected]

11

Illustration 2:

Law of Reflection & Refraction from Fermat’s principle:

A

μ1 B μ2

a

α β b

x O d-x

d

Optical path L= μ1 AD+μ2OB = μ1√a2+x2 +μ2√b2+(d-x)2

L=L(x)

dL/ dx =0 from minimization principle.

dL/ dx = μ1(1/2)[2x/(√a2+x2)]+μ2(1/2)[2(d-x)(-1)]/[√b2+(d-x)2]

dL/ dx = μ1[x/(√a2+x2)]+ μ2 [(d-x)(-1)] /[√b2+(d-x)2]]

μ1 [x/√(a2+x2)] = μ2 (d-x) /[√b2+(d-x)2]

μ1 sin α = μ2 sin β

If medium (1) & (2) is same μ1= μ2

sin α = sin β ⇒ α = β Law of reflection

If μ1 ≠ μ2 medium is different μ1 sin α = μ2 sin β

sin α/sin β= μ2/ μ1=(c/v2)/(c/v1)= v1/v2

sin α/sin β= μ2/ μ1=v1/v2 = sin i/sin r Snell’s Law of refraction

Note: it is my suggestion to use Snell’s Law in this manner

μ1 sin α = μ2 sin β = μ3 sin γ

μ sin α = constant

Page 12: Geometrical Optics Book With Special Lecture

ANUJ PHYSICAL SCIENCES PHYSICS FOR IIT-JEE /AIEEE/AIPMT

Office: 128/652 “Y” Block Kidwai Nagar Kanpur -11 UP, Website: www.apskanpur.in Mob: 09795647531, 09889787531, 0848664308 E-mail:[email protected]

12

Problems on optical path& Fermat’s principle:

Problem 1: A man walks on the hard ground with a speed of 5 ft. /sec but he has a

speed of 3ft. /sec on the sandy ground. Suppose he is standing at the border of sandy

and hard ground. The man can reach the tree by walking100 ft. along the border and

120 ft on the sandy ground normal to the border.

Find out the value of path which requires minimum time to reach tree.

(a) 190 or 10 (b) 180 or 20 (c) 170 or 30 (d) none

Problem 2: In figure two stations A & B in different territories are separated by a

border line CD. A messenger can travel in the upper territory with speed Va and in the

lower territory with speed Vb. Several messenger start from A and follow different paths

like APB having different positions of p specified by a distance x from M. It is found that

the messenger who chooses x as 4.0 km reaches B in minimum time.

Answer the following question?

(1) What is the relation between Va and Vb

(a) Va /Vb = ½√ (73/52) (b) Va /Vb = ¼√(73/52)

(c) Va /Vb = ½√(52/73) (d) None

O

6 km i

C N D

M x r 3 km

12km

(2) If the speed Va and Vb are interchanged.

What will be the new value of x to give the fastest path?

(a) X ≈ 10.2 (b) X ≈ 15.7 (c) X≈5.3 (d) None

Page 13: Geometrical Optics Book With Special Lecture

ANUJ PHYSICAL SCIENCES PHYSICS FOR IIT-JEE /AIEEE/AIPMT

Office: 128/652 “Y” Block Kidwai Nagar Kanpur -11 UP, Website: www.apskanpur.in Mob: 09795647531, 09889787531, 0848664308 E-mail:[email protected]

13

Deviation: Single Reflection:

i r

δ = (π-2i)

Angle between incident ray & emergent ray δ=π-2i

When i=0 (Normal incidence)

δ=π = δmax

When i = π / 2 (Grazing incidence)

δ=π - 2 π / 2 = 0 = δmin

Graphical variation between angle of incidence and deviation is:

δ

π

(π/2) i

Graph between deviation & angle of incidence

Page 14: Geometrical Optics Book With Special Lecture

ANUJ PHYSICAL SCIENCES PHYSICS FOR IIT-JEE /AIEEE/AIPMT

Office: 128/652 “Y” Block Kidwai Nagar Kanpur -11 UP, Website: www.apskanpur.in Mob: 09795647531, 09889787531, 0848664308 E-mail:[email protected]

14

Problem 3: Two plane mirrors are inclined to each other such that a ray of light

incident on first mirror and parallel to the second is reflected from the second mirror

parallel to the first mirror.

1) The angle between the two mirrors is

(a) θ =600

(b) θ= 1200

(c) θ= 900

(d) None

2) Total deviation produced in the incident ray due to the two reflections

(a) 2400 anticlockwise or 1200 clockwise

(b) 1200 anticlockwise or 2400 clockwise

(c) 600 anticlockwise or 1200 clockwise

(d) 1200 clockwise or 600 anticlockwise

Solution:

M1 θ

δ1 θ δ

θ θ θ M2

δ2 δ2

(1) 3θ = 1800 , θ = 600

(2) From figure total deviation δ = θ+π

δ = 60+120 =2400 anticlockwise or 1200 clockwise

or δnet = δ1+ δ2 =1200 Anticlockwise + 1200Anticlockwise = 2400 Anticlockwise

Page 15: Geometrical Optics Book With Special Lecture

ANUJ PHYSICAL SCIENCES PHYSICS FOR IIT-JEE /AIEEE/AIPMT

Office: 128/652 “Y” Block Kidwai Nagar Kanpur -11 UP, Website: www.apskanpur.in Mob: 09795647531, 09889787531, 0848664308 E-mail:[email protected]

15

Rotation of mirror:

Problem 4: If mirror is turned by an angle θ then angle turned by reflected ray.

(a) θ

(b) 2θ

(c) 3θ

(d) 4θ

Solution: If mirror is turned by angle θ

Thus angle turned by reflected ray =δ1-δ2

δ1= π-2ɸ and δ2 = π- 2(θ+ɸ)

δ1- δ1-δ2 = (π-2ɸ) – { π- 2(θ+ɸ)}

δ1-δ2 = π-2ɸ – π+ 2θ+2ɸ = 2θ

I N1 N2

R1 δ1- δ2

Φ θ (θ +ϕ) R2

Θ δ1

θ δ2

Note:

If mirror is kept fixed & incident ray is rotated then reflected ray will rotate in

opposite sense by same angle.

If mirror and incident ray both are rotated then net rotation suffered by reflected

ray will be algebraic sum of rotation suffered by reflected ray due to mirror.

Rotation and incident ray rotation separated keeping sense of rotation in mind.

Page 16: Geometrical Optics Book With Special Lecture

ANUJ PHYSICAL SCIENCES PHYSICS FOR IIT-JEE /AIEEE/AIPMT

Office: 128/652 “Y” Block Kidwai Nagar Kanpur -11 UP, Website: www.apskanpur.in Mob: 09795647531, 09889787531, 0848664308 E-mail:[email protected]

16

Images formed by two mirrors in contact:

Symmetric position of the object

θ

Asymmetric position of the object

Suppose θ is the angle between the two mirrors.

(a) If 3600/ θ is even integer.

No of images n = [(360/θ) -1] for all positions of the object.

(b) If 3600/θ is odd integer.

No of images n=(360/θ) if the object is placed Asymmetric

( when object is placed off the bisector of the mirror)

No of images n=[ (360/θ) -1] if object is placed Symmetric

(when object is placed on the bisector of the mirrors)

(c) If 3600/θ is a fraction

No of images formed will be equal to its integral part.

Check 360/θ

Even Odd Fraction

n = [(360/θ) -1] (both) n = Integral part

Asymmetric Symmetric

n= 360/θ n= [(360/θ) -1]

Image formed by two mirrors in contact working procedure:

Step 1st: find the value of 3600/θ

Step 2nd: check this is even no, odd no or fraction

Step 3rd:

Even no n=[(360/θ) -1]

Odd no n=[(360/θ) -1] → symmetric n=360/θ → Asymmetric

Fraction n= Integral part

Page 17: Geometrical Optics Book With Special Lecture

ANUJ PHYSICAL SCIENCES PHYSICS FOR IIT-JEE /AIEEE/AIPMT

Office: 128/652 “Y” Block Kidwai Nagar Kanpur -11 UP, Website: www.apskanpur.in Mob: 09795647531, 09889787531, 0848664308 E-mail:[email protected]

17

No of images

θ (degree) 3600/θ Asymmetric Symmetric

0 ∞ ∞ ∞

30 12 11 11

45 8 7 7

60 6 5 5

72 5 5 4

75 4.8 4 4

90 4 3 3

Formation of image by plane mirror: The image formed by a plane mirror has following characteristics:

(1) it is visual & erects

(2) Of the same size as the object

(3) Laterally inverted

(4) As far behind the mirror as the object in front.

Minimum size of plane mirror required to see full height of the observer

himself.

A(head) A’

x α

x α M

h

y β N

y β

B(feet) B’

Man Minimum size of mirror

Page 18: Geometrical Optics Book With Special Lecture

ANUJ PHYSICAL SCIENCES PHYSICS FOR IIT-JEE /AIEEE/AIPMT

Office: 128/652 “Y” Block Kidwai Nagar Kanpur -11 UP, Website: www.apskanpur.in Mob: 09795647531, 09889787531, 0848664308 E-mail:[email protected]

18

x+ y = MN = size of the mirror

2x+ 2y=h

x + y= h/2

Thus in order to see full height, a person requires a plane mirror.

This relation is true for any distance of observer from plane mirror.

Also the lower edge of mirror should be kept at half of the eye level at a height y

from the feet level.

Note:

(1) It should be noted that a person can see his full height by turning his head or eyes

even in a smaller mirror.

(2) An observer can see the image of a tall building in a very small mirror by keeping

mirror at a large distance.

Problem 5: A man is standing exactly at the cube of the hall. He wants to see the

image of his back wall in a mirror hanging on front wall. Find the minimum size of the

mirror required.

C C’

A

h E y

B

D x x 2x D’

Solution: Suppose the height of the wall be h and required height of the mirror be y.

In the similar triangle EAB & EC’D’ we have

y / x = h/3x

y= h/3 Answer

Page 19: Geometrical Optics Book With Special Lecture

ANUJ PHYSICAL SCIENCES PHYSICS FOR IIT-JEE /AIEEE/AIPMT

Office: 128/652 “Y” Block Kidwai Nagar Kanpur -11 UP, Website: www.apskanpur.in Mob: 09795647531, 09889787531, 0848664308 E-mail:[email protected]

19

Power of an optical instrument (V Imp concept):

Powers of an optical instrument depend upon its ability to converge or diverge

the light rays.

One who can converge or diverge light rays by larger amount is more powerful

than the one who converges/diverges by a lesser amount.

δ1 δ2

F1 F2

P1 P2

(a) Optical instrument (b) Optical instrument

As from a&b it is clear that δ1 > δ2 so P1 > P2 i.e. the instrument in a is more

powerful than instrument b.

The optical instrument which is more powerful has smaller focal length as in the

figure it can be seen that f1 < f2 thus P α (1/f)

A part from this the optical instrument which converges light rays is said to be of

positive power & the one from which light rays diverge has the negative power.

Instrument →light rays convergent → +ive power

Instrument →light rays divergent → -ive power

P = +ive P = -ive

(a) (b)

convergent lens has +ive power & +ive focal length. Divergent lens has –ive power & -

ive focal length.

Page 20: Geometrical Optics Book With Special Lecture

ANUJ PHYSICAL SCIENCES PHYSICS FOR IIT-JEE /AIEEE/AIPMT

Office: 128/652 “Y” Block Kidwai Nagar Kanpur -11 UP, Website: www.apskanpur.in Mob: 09795647531, 09889787531, 0848664308 E-mail:[email protected]

20

(a)

δ

P = 1/f Convex lens

f Converge → +ive power & +ive focal length

(b)

δ

P = 1/f Concave lens

f Diverge → -ive power → -ive focal length

(c)

P=-1/f Convex mirror

P = -ive, f = +ive } → Diverge rays

f

(d)

P=-1/f Concave mirror

P =+ive, f = -ive} → Converge rays

Note: Power of lens=1/focal length But Power of mirror = - 1/focal length

Page 21: Geometrical Optics Book With Special Lecture

ANUJ PHYSICAL SCIENCES PHYSICS FOR IIT-JEE /AIEEE/AIPMT

Office: 128/652 “Y” Block Kidwai Nagar Kanpur -11 UP, Website: www.apskanpur.in Mob: 09795647531, 09889787531, 0848664308 E-mail:[email protected]

21

Lecture-2

Geometrical Optics

Physics for IIT - JEE

Vector form of Laws of Reflection

Problem based on Snell’s Law

Variable Refractive Index Problems

Page 22: Geometrical Optics Book With Special Lecture

ANUJ PHYSICAL SCIENCES PHYSICS FOR IIT-JEE /AIEEE/AIPMT

Office: 128/652 “Y” Block Kidwai Nagar Kanpur -11 UP, Website: www.apskanpur.in Mob: 09795647531, 09889787531, 0848664308 E-mail:[email protected]

22

Laws of Reflection in vector form:

Problem 1: If ê1=unit vector along incident ray,ê2=unit vector along reflected ray &

n =unit vector along normal, then vector form of laws of Reflection is

a) ê2=ê1+2 n n b) ê2=ê1-2(ê1 n n

c) ê2= + n n d) ê2=ê1-(ê1 n n

N P

n e2

i

((π/2)-i)

O R

e1

Q

Solution: From vector law of addition in triangle OPQ

OP= OQ+QP

OP ê2=OQê1+2QRn

ê2=ê1+2QR/OP n (OP=OQ)

sin [(π/2)-i] = cos i = PR/OP = QR/OP

ê2=ê1+2 cos i

Now ê1.n = ||n1 cos (π-i) = - cos

ê2=e1-2(ê1 n n

This is vector form of law of reflection.

Problem 2: A light ray parallel to the x axis strikes the outer reflecting surface of a

sphere at a point (2, 2, 0). Its centre is at the point (0, 0,-1).

1. Find the unit vector along the direction of reflected ray

(a) (-i+8j+4k)/9 (b) (i+8j+4k)/9

(c) (-i+8j-4k)/9 (d) (-i+-j+4k)/9

2. If the unit vector along the direction of reflected ray is xi+yj+zk. Find the value of

yz/x2

Page 23: Geometrical Optics Book With Special Lecture

ANUJ PHYSICAL SCIENCES PHYSICS FOR IIT-JEE /AIEEE/AIPMT

Office: 128/652 “Y” Block Kidwai Nagar Kanpur -11 UP, Website: www.apskanpur.in Mob: 09795647531, 09889787531, 0848664308 E-mail:[email protected]

23

(a) 32 (b) 28 (c) 32/9 (d) None

Solution:

If ê1=unit vector along incident ray,ê2=unit vector along reflected ray & n =unit vector

along normal, then vector form of laws of Reflection is ê2=e1-2(ê1 n n

Y

(2, 2,0)

X

(0, 0, -1)

n = (2i+2j+k)/3 ê2=? e1=-i

Using vector form of laws of Reflection ê2=e1-2(ê1 n n

ê2= -i-2(-2/3) (2i+2j+k)/3 = -i+4 (2i+2j+k)/9= (-i+8j+4k)/9

According to problem the unit vector along the direction of reflected ray is xi+yj+zk

Now on comparisons of two equations we get the values of x, y, & z as -1/9, 8/9 & 4/9

The value of yz/x2 = (8/9) (4/9) / (-1/9)2 =32

Problem 3: If x-y plane is reflecting plane than the equation of incident ray is

=xî+yî+zk thenfind the equation of reflecting ray .

ri = x i +y j +z k z rr = x i +y j - z k

O reflecting surface x-y plane

Y

x

Solution: Whenever reflection takes place, the component of incident ray parallel to reflecting

Page 24: Geometrical Optics Book With Special Lecture

ANUJ PHYSICAL SCIENCES PHYSICS FOR IIT-JEE /AIEEE/AIPMT

Office: 128/652 “Y” Block Kidwai Nagar Kanpur -11 UP, Website: www.apskanpur.in Mob: 09795647531, 09889787531, 0848664308 E-mail:[email protected]

24

surface remains unchanged while component perpendicular to reflecting surface(i.e

along the normal)reverses in direction.The equation of reflecting ray is rr=xî+y -zk .

Comprehension and analitical ability:

Problem 4:An optical image is formed due to intersection of reflected rays when

reflected rays actually meet,the image so formed is called real.when reflected ray apper

to meet,than image formed is called virtual.There are three plane mirrors arranged

perpendicular to each forming principal co-ordinate planes namely x-y,z-y and z-x.Their

common point of intersection is taken as origin.A point object having co-ordinate

(1,2,3)is placed as shown in fig

Z object (1, 2, 3)

3

O y

1

X 2

(1) Which of the following is not the coordinates of images formed due to

meeting of rays undergoing single reflection only

(a) (1,2,-3) (b)(1,-2,3) (c)(-1,2,3) (d)none

(2) Which of the following is not the coordinate of images formed due to

meeting of rays undergoing two reflection only

(a) (-1,-2,3) (b)(1,-2,-3) (c)(-1,2,3) (d)none

(3) Which of the following is not the coordinate of images formed due to

meeting of rays undergoing one reflection from each mirror

(a) (-1,-2,3) (b)(-3,-2,1) (c)(-2,-3,-1) (d)(-1,3,2)

(4) The number of images formed are

(a) 3 (b) 4 (c) 5 (d) 6 (e) 7

Answer: 1(d) 2(c) 3(a) 4(e)

Problem 5 (AIEEE 2011): Let the x-z plane be the boundary between two

transparent media. Medium 1 in z ≥ 0 has a refractive index of √2 and medium 2 with z

< 0 has a refractive index of √3. A ray of light in medium 1 given by the vector

A = 6√3 +8√3 -10k is incident on the plane of separation. The angle of refraction in

medium 2 is?

(a) 450 (b) 600 (c) 750 (d) 300

Page 25: Geometrical Optics Book With Special Lecture

ANUJ PHYSICAL SCIENCES PHYSICS FOR IIT-JEE /AIEEE/AIPMT

Office: 128/652 “Y” Block Kidwai Nagar Kanpur -11 UP, Website: www.apskanpur.in Mob: 09795647531, 09889787531, 0848664308 E-mail:[email protected]

25

Solution: As refractive index for z > 0 and z ≤ 0 is different x-y plane should be

boundary between two media. Angle of incidence cos i = |[AZ/√Ax2+Ay2+Az2]| = ½

i=600 from Snell’s law sin i / sin r = √3/2 r = 450

Problem 6: Excellent (IIT-JEE 1999): The x-y plane is the boundary between two

transparent media-1 with z≥0 has a refractive index 2 and media-2 with z≤ 0 has a

refractive index 3 .A ray of light in medium-1 given by the vector A=6 3î+8 3 -10k is

incident on the plane of separation.

(1) The angle of incidence

(a) 300 (b)600 (c)900 (d) none

(2) The angle of refraction

(a) 300 (b)450 (c)600 (d)900

(3)The unit vector in the direction of the refractive ray in medium-2

(a)1/(10√2) (6î+8 -10k ) (b) 1/(10√2) (6î-8 +10k )

(c) 1/(10√2) (6î+10 -8k ) (d) None

A E

Z ≥ 0 i

μ1 = √2

B

Z ≤ 0 r μ2 = √3

D C

Solution: The vector of incident ray is given by

AB = 6√3 +8√3 -10k In Δ ABE AE+EB=AB

AE =6√3 +8√3 , EB = -10k

The angle of incidence between AB and EB can be obtained as

cos i = AB.EB/AB.EB = [(6√3 +8√3 -10k )( -10k )]/[√(6√3)2 +(5√3)2 +(10)2](√102)

cos i = 100/[10√((36*3)+(64*3)+(100))]= 100/[100√400] = 100/(10 x 20) = ½

Page 26: Geometrical Optics Book With Special Lecture

ANUJ PHYSICAL SCIENCES PHYSICS FOR IIT-JEE /AIEEE/AIPMT

Office: 128/652 “Y” Block Kidwai Nagar Kanpur -11 UP, Website: www.apskanpur.in Mob: 09795647531, 09889787531, 0848664308 E-mail:[email protected]

26

cos i = ½ ⇒ i=600 Angle of incidence

By Snell’s law: μ1 sin i = μ2 sin r

√2 sin 60 = √3 sin r

√2 (√3/2) = √3 sin r

(1/√2) = sin r ⇒ r = 450 angle of refraction

The vector of refracted ray can be written as.

BC=BD+DC

DC=A√2 = 6√3 +8√3

Unit vector along DC = e = (6√3 +8√3 )/√(6√3)2 +(8√3)2 = (6√3 +8√3 )/10√3

= (6 +8 )/10

BC=BD+DC= BC cos r(-k )+BC sin r ê

BC/BC=cos r (-k )+ sin r ê = -k cos 450 +e sin 450 = (1/√2)(-k ) +(6 +8 )/10√2

= (1/10√2)[-10k + 6 +8 ]

BC/BC= (1/10√2)[ 6 +8 -10k ]

Unit vector in the direction of refracted ray.

Problem 7: A ray of light passes through four transparent media with refractive

Indices μ1, μ2,μ3,μ4 as shown in figure. The surface of all media are parallel if the

emergent ray CD is parallel to incident ray AB, we must have

(a) μ1 = μ3 (b) μ2 =μ3 (c) μ3= μ4 (d) μ4=μ1

A

μ1 μ2 μ3 μ4 D

B e C ic

iB

Solution: Applying Snell’s law at B and C

μ sin i=constant

μ1siniB= μ4 sin iC

iB = iC since AB ‖ CB μ1=μ4

Page 27: Geometrical Optics Book With Special Lecture

ANUJ PHYSICAL SCIENCES PHYSICS FOR IIT-JEE /AIEEE/AIPMT

Office: 128/652 “Y” Block Kidwai Nagar Kanpur -11 UP, Website: www.apskanpur.in Mob: 09795647531, 09889787531, 0848664308 E-mail:[email protected]

27

Problem 8: Two plane mirrors A and B are aligned parallel to each other, as shown in

the figure. A light ray is incident at an angle 300at a point just inside one end of A the

plane of incidence coincides with the plane of the figure. The maximum number of times

the ray undergoes reflections (including the first one) before it emerges out

(a) 28 (b) 30 (c) 32 (d) 34 2√3m

B d

0.2 m 300

A

Solution: L =2√3m

d/ 0.2 = tan 30 ⇒ d=0.2 tan 30

d = 0.2/√3

L/d = 2√3/ 0 2/√3 =30

Therefore maximum number of reflection is 30

Problem 9: A ray light is incident at the glass water interface at an angle i, it emerges

finally parallel to the surface of water, than the value of μg would be (a) (4/3) sin i

(b) 1/ sin i (c) 4/3 (d) 1

Air (2)

μω = 4/3

water r (1)

glass i

Solution: Apply Snell’s law at surface 1 and 2 μg sin i=μa sin 90 μg =1/sin i

Page 28: Geometrical Optics Book With Special Lecture

ANUJ PHYSICAL SCIENCES PHYSICS FOR IIT-JEE /AIEEE/AIPMT

Office: 128/652 “Y” Block Kidwai Nagar Kanpur -11 UP, Website: www.apskanpur.in Mob: 09795647531, 09889787531, 0848664308 E-mail:[email protected]

28

Problem 10: The x-y plane separates two media A and B of refractive indicesμ1=1.5

and μ2=2. A ray of light travels from A to B, its directions in to two media are given by

unit vectors u1= a î+b and u2=c î+d then (a) a/c = 4/3 (b) a/c = 3/4

(c) b/d = 4/3 (d) b/d = ¾

Solution:

(a + b ) y

μ1 = 1.5

i

μ2 = 2 r (c +d )

√a2+b2=√c2+d2 =1 Unit vectors Magnitude unity

From Snell’s law μ1 sin i = μ2 sin r 1.5 sin i=2 sin r 1.5 [a/√(a2+b2)] = 2c/√ c2+d2 (3/2)a = 2c, a= (4/3)c

a/c=4/3

Page 29: Geometrical Optics Book With Special Lecture

ANUJ PHYSICAL SCIENCES PHYSICS FOR IIT-JEE /AIEEE/AIPMT

Office: 128/652 “Y” Block Kidwai Nagar Kanpur -11 UP, Website: www.apskanpur.in Mob: 09795647531, 09889787531, 0848664308 E-mail:[email protected]

29

Variable Refractive Index

Problem 11: IIT JEE (1995) Subjective

A ray of light travelling in air is incident at grazing angle (angle of incidence ≈ 900) on a

long rectangular slab of a transparent media of thickness t=1.0m. The point of incidence

is the origin A(0, 0). The medium has a variable refractive index given by

n(y)=(ky3/2+1)1/2 Where k=1.0(m)-3/2 , the refractive index of air is 1

1. Obtain a relation between the slope of the trajectory of the ray at appoint B(x,y)

in the medium and the incidence angle at that point

(a) cot θ = dy/dx (b) tan θ = dy/dx (c) cot θ = (1/2)(dy/dx) (d) none

2. Obtain an equation for the trajectory y(x) of the ray in the medium

(a) y=K2(x/4)4 (b) y=kx2 (c) y=k(x/4)2 (d) none

3. Determine the coordinates(x1,y1) of the point p where the ray intersects the

upper surface of the slap-air boundary

a) (4m, 1m) (b) (2m, 1m) (c) (1m, 1m) (d) none

4. Indicate the path of the ray subsequently.

(a) The ray will emerge parallel to boundary (b) Perpendicular to boundary

(c) Neither (a) nor (b (d) Data insufficient

Y Trajectory of light

Air P (x1, y1)

t = 1m

B (x, y)

ϕ θ x

A (0, 0) air

Solution:

Page 30: Geometrical Optics Book With Special Lecture

ANUJ PHYSICAL SCIENCES PHYSICS FOR IIT-JEE /AIEEE/AIPMT

Office: 128/652 “Y” Block Kidwai Nagar Kanpur -11 UP, Website: www.apskanpur.in Mob: 09795647531, 09889787531, 0848664308 E-mail:[email protected]

30

(a) Trajectory of the curve is shown by dotted curve the slope of the target at point B tan ɸ= dy/dx The angle of incidence at B is θ=(90-ɸ) tan ɸ=tan (90-θ)=cot θ= dy/dx (1)

(b) from Snell’s law at A and B

1 sin 90=n(y) sinθ (2)

sinθ= 1/n(y) = 1/[ky3/2 +1]1/2 (3)

cosθ= √1-sin2θ =√1- [1/[ky3/2 +1]1/2]

cot θ = cos θ/sin θ =[√1- [1/[ky3/2 +1]1/2]]/[1/[ky3/2 +1]1/2]

cot θ = √[(ky3/2 +1-1)/( ky3/2 +1)]/[ 1/[ky3/2 +1]1/2]

cot θ= √[(ky3/2)/( ky3/2 +1)][ [ky3/2 +1]1/2]

cot θ= k1/2y3/4 (4)

From equation (3)

Sinθ =1/n(y) = cos ɸ= 1/[√1+tan2ϕ]

cot θ=cot(90-ɸ)=tan ɸ

1/n(y) = 1/[√1+cot2θ]

n(y)= √1+cot2θ = √1 +(dy/dx)2

[ky3/2 +1]1/2 = √1 +(dy/dx)2

(dy/dx)2 = ky3/2

dy/dx = k1/2y3/2

∫[dy/y]3/4 = ∫k1/2dx

∫y-3/4 = √k ∫dx

[y-(3/4)]/[(-3/4)+1]=√k x +c

4y1/4=√k x +c

Applying boundary condition

x=0, y=0, c=0

4y1/4=√k x

y1/4= (√k/4) x

y = k2(x/4)4

(c) At point P

x=?, y=1, k=1

1= k2 (x/4)4

1=1(x/4)4

1=(x/4)

x=4m

The coordinates of P (4, 1)

(d) From Snell’s law nA sin iA=nP sin iP , iA=iP=90

The ray will emerge parallel to the boundaries

Page 31: Geometrical Optics Book With Special Lecture

ANUJ PHYSICAL SCIENCES PHYSICS FOR IIT-JEE /AIEEE/AIPMT

Office: 128/652 “Y” Block Kidwai Nagar Kanpur -11 UP, Website: www.apskanpur.in Mob: 09795647531, 09889787531, 0848664308 E-mail:[email protected]

31

Problem 12: Due to a vertical temperature gradient in the atmosphere the index of

refraction varies as n=n0√1+ay, where n0 is the index of refraction at the surface and

a=2.0*10-6m-1 .A person of height h=2.0 m stand is on a level surface. Beyond what

distance he cannot see the runway (a) 2000 m (b) 200 m (c) 100 m (d) 1000 m

Y Trajectory of light ray

P

Y i

θ x

Solution: Let O be the distant object just visible to the man. Let P be a point on the

trajectory of the ray. From figure θ=90-i (1) Slope of tangent at point p=tan θ=dy/dx =tan(90-i)=cot i tan θ = cot i (2) From Snell’s law n sin i=constant at the surface, y=0 n=n0 , i=900 n0sin 90=n sin i n0 = n0√1+ay sin i

sin i= 1/√(1+ay)

1 √(1+ay)

i

ay

cot i= √ay = dy/dx dy/ dx =ay ∫

= ∫

dx

= √a∫

2y1/2 = √a x

x= 2 √y/a , On substituting y=2m ,

X=2√(2/9) = 2√(2/2*10-6) =2√106 =2*103

x = 2000m

Page 32: Geometrical Optics Book With Special Lecture

ANUJ PHYSICAL SCIENCES PHYSICS FOR IIT-JEE /AIEEE/AIPMT

Office: 128/652 “Y” Block Kidwai Nagar Kanpur -11 UP, Website: www.apskanpur.in Mob: 09795647531, 09889787531, 0848664308 E-mail:[email protected]

32

Problem 13: A ray of light travelling in air is incident at angle of incident 300 on one

surface of slab in which refractive index varies with y. The light travels along the curve

y=4 x2 (y & x are in meter) in the slab. Find out the refractive index of the slab at

y=1/2m in the slab

(a) μ =3/2 (b) 4/3 (c) 9/8 (d) none

Solution:

Y air

Slab of variable refractive index

300

P

Let R.I at y=y1 is μ and corresponding angle of refraction is θ μ sin θ=1.sin 30 tan [(π/2)-θ] = dy/dx = cot θ, y=4x2 dy/dx = d/dx(4x2) = 8x = cot θ cot θ = 8x = 8(y/a)1/2 =4y1/2 cot θ = 4y1/2 , at y=1/2 cot θ = 4/√2 =2√2 ,

1 3

θ

2√2

sin θ=1/3

μ sin θ =1 sin 30,

μ (1/3) =1/2

μ=3/2

Page 33: Geometrical Optics Book With Special Lecture

ANUJ PHYSICAL SCIENCES PHYSICS FOR IIT-JEE /AIEEE/AIPMT

Office: 128/652 “Y” Block Kidwai Nagar Kanpur -11 UP, Website: www.apskanpur.in Mob: 09795647531, 09889787531, 0848664308 E-mail:[email protected]

33

Lecture-3

Geometrical Optics

Physics for IIT - JEE

The Cartesian sign convention

Rules for ray diagrams

Formulas related to mirrors & Magnification

Relative position, size and nature of image for different

positions of object in spherical mirror

Page 34: Geometrical Optics Book With Special Lecture

ANUJ PHYSICAL SCIENCES PHYSICS FOR IIT-JEE /AIEEE/AIPMT

Office: 128/652 “Y” Block Kidwai Nagar Kanpur -11 UP, Website: www.apskanpur.in Mob: 09795647531, 09889787531, 0848664308 E-mail:[email protected]

34

The Cartesian sign convention:

A sign convention facilitates the computations of the object and image distances,

assesses the nature of image (real or virtual), its magnification and orientation.

The object is placed to the left of the optical surface (a mirror or refracting surface).

The light is incident from left to right. The centre of the optical surface is called vertex.

The vertex is taken as origin. The horizontal axis is called the optic axis.

(1) Distance measured to the right of the origin along optic axis are positive distances,

since they are along the positive axis of the standard Cartesian coordinate system.

(2) Distances measured to the left of the origin along the optic axis are negative

distances, since they are along the negative axis of the standard Cartesian coordinate

system.

(3) The sign convention for magnification: Magnification is defined as the ratio of the

size of image to the size of object ∣m∣ =Image size/object size.

Positive distance

Negative distance positive distances

Optical axis optical axis

origin (vertex)

negative distances optical surface

This is referred to as lateral magnification. If the magnification m is positive the image

of the object is erect (upright), meaning that the image has same orientation as the

object. If the magnification is negative, the image is inverted (upside down).

Page 35: Geometrical Optics Book With Special Lecture

ANUJ PHYSICAL SCIENCES PHYSICS FOR IIT-JEE /AIEEE/AIPMT

Office: 128/652 “Y” Block Kidwai Nagar Kanpur -11 UP, Website: www.apskanpur.in Mob: 09795647531, 09889787531, 0848664308 E-mail:[email protected]

35

Rules for ray diagrams:

We can locate the image of any extended object graphically by drawing any two of the

following four principle rays.

1. A ray, initially parallel to the principal axis is reflected through the focus of the mirror

2. A ray, initially passing through the focus is reflected parallel to the principal axis.

3. A ray passing through the so line joining point object and its image cuts principal axis

at centre of curvature.

4. A ray incident at the pole is reflected symmetrically. So line joining O and I’ or O’ and I

will cut principal axis at pole.

(i) Spherical mirrors bring paraxial rays to an approximate focus at a point on the

mirror axis.

(ii) The focal length of a spherical mirror is equal to half the radius of curvature of the

mirror.

The mirror equation for spherical mirror is

(1/v) + (1/u) = (1/f) = 2/R

when the object point O is located infinitely far away from the mirror, then u = -∞ and

the position of the image is called the focal length. If we substitute u = - ∞ into the

mirror equation,

(1/-∞) + (1/v) = (2/R)

since v =f, 0 +(1/f) = 2/f, f = R/2

parallel incident rays intersect in case of concave mirror and appear to intersect in case

of convex mirror.

Convex mirror

Light from object point O f F C

at u = -∞m f >0m, f = (R/2)>0m

(a)

Concave mirror

Light from object point C F O f <0m

at u = -∞m f =(R/2)>0m

F

(b)

Page 36: Geometrical Optics Book With Special Lecture

ANUJ PHYSICAL SCIENCES PHYSICS FOR IIT-JEE /AIEEE/AIPMT

Office: 128/652 “Y” Block Kidwai Nagar Kanpur -11 UP, Website: www.apskanpur.in Mob: 09795647531, 09889787531, 0848664308 E-mail:[email protected]

36

The magnification is m = Image size/object size = -v/u

Note that the law of reflection is independent of the medium in which light is travelling.

The mirror and magnification equation can be applied irrespective of the medium

surrounding the mirror.

(1) A ray parallel to axis is reflected back through focal point as shown in fig. (a)

(2) A ray that passes through the focal point on the ray to the mirror is reflected back

parallel to the mirror axis as shown in fig. (b)

(3) A ray from the object is directed towards the centre of curvature of the mirror, after

reflection the ray retraces its path because it strikes the mirror along the normal to the

mirror.

(4) A ray that strikes the vertex of the mirror reflects at an equal angle on the other side

of the mirror axis.

Any two of these rays are sufficient to locate the position of the image.

We can assess two things from ray diagram:

(a) If the image is in front of the mirror, it is real; if the image is at the back of the

mirror, the image is virtual.

(b) We are able to guess if the magnification is positive or negative and greater or less

than unity.

Note: Mirror equation 1/v +1/u =1/f

(i)Mirror equation is valid for both type of object irrespective of its position.

(ii) In case of spherical mirrors if object distance (x1) are measured from focus instead

of pole, then u = f + x1, and v = f +x2 the mirror formula reduces to

[1/(f+x2)] + [1/(f+x1)] = 1/f or x1x2 = f2

which is known as Newton’s formula.

This formula is applicable to real objects and real images only.

(iii) In numerical problems it is convenient to use mirror equation in following form:

v = [uf/(u-f)],

u =[ vf/(v-f)],

f = [ uv/(u +v)]

(iv)While using mirror equation known quantities is substituted with proper sign and

quantities to be calculated (unknown quantities) are not be given any sign.

Page 37: Geometrical Optics Book With Special Lecture

ANUJ PHYSICAL SCIENCES PHYSICS FOR IIT-JEE /AIEEE/AIPMT

Office: 128/652 “Y” Block Kidwai Nagar Kanpur -11 UP, Website: www.apskanpur.in Mob: 09795647531, 09889787531, 0848664308 E-mail:[email protected]

37

Magnification: This is defined as the ratio of size of image to the size of object

magnification (m) = size of image (I)/ size of object (O)

Three types of magnification are produced by a spherical mirror which are described

below:

(a) Linear magnification (Transverse magnification/Lateral magnification):

A

O C M P

A’

In this case, size of object and image is measured perpendicular to principal axis.

m = Height of image(H1)/Height of object(HO)

From similar triangles APO and A’PM

AO/A’M = PO/PM (1)

(HO/-HI) = (-u/-v)

or(HI/HO) =- (v/u)

thus m = -v/u

Note:

(i) The above formula is valid for convex as well as concave mirror and it is independent

of nature of object (real or virtual) irrespective of its position.

(ii) In terms of focal length, magnification can be expressed as

m = f/(f-u) = (f-v)/f

(iii) Magnification can be either positive or negative depending on the nature of the

image. If m is negative, then image is inverted with respect to object and if m is positive

then image is erect with respect to object.

(iv) Real image is not always inverted and virtual image is not always erect.

(b) Superficial magnification: when a small surface is placed perpendicular to the

principal axis, both length and breadth are magnified in the ratio v/u. The superficial

magnification is ms = Area of image/Area of object = [(-v/u)(-v/u)] = v2/u2

(c) Axial magnification (Longitudinal magnification): In this case size of image and

object is measured along principal axis

Page 38: Geometrical Optics Book With Special Lecture

ANUJ PHYSICAL SCIENCES PHYSICS FOR IIT-JEE /AIEEE/AIPMT

Office: 128/652 “Y” Block Kidwai Nagar Kanpur -11 UP, Website: www.apskanpur.in Mob: 09795647531, 09889787531, 0848664308 E-mail:[email protected]

38

uB

uA

B A

A’ B

vB

vA

m = Length of image/Length of object

For linear object placed along the principal axis, the magnification is,

mL = [vA - vB]/[uA - uB] (2)

Note: For small object, mL = dv/du

From mirror object, mL = dv/du

From mirror equation we have (1/v) + (1/u) = (1/f)

Differentiating both sides, we get

–(1/v2)dv – (1/u2)du = 0

or dv/du = -(v2/u2)

Thus, Longitudinal magnification ≈ Transverse magnification)2

Formulas related to mirrors

(1) The focal length of a spherical mirror of radius R is given by f = R/2 with proper

sign convention.

(2) The power of a mirror P = -1/f(in m) = - 100/f(in cm)

(3) Magnification: If a thin object of linear size O is situated vertically on the axis of

a mirror at a distance u from the pole & its image of size I is formed at a distance v

(from the pole) magnification transverse (m) = -v/u = I/O

m → -ive means image is inverted with respect to object

m → +ive means image is erect with respect to object

M traverse = -v/u M linear = I/O =-[(v2 – v1)/(u2 –u1)]= -dv For small object

Page 39: Geometrical Optics Book With Special Lecture

ANUJ PHYSICAL SCIENCES PHYSICS FOR IIT-JEE /AIEEE/AIPMT

Office: 128/652 “Y” Block Kidwai Nagar Kanpur -11 UP, Website: www.apskanpur.in Mob: 09795647531, 09889787531, 0848664308 E-mail:[email protected]

39

-m → real image, +m → virtual image

(4) (a)In case of small linear objects: (1/v) + (1/u) = (1/f)

(-dv/v2) + (-du/u2) = (-df/f2) → zero

(-dv/du) = (v2/u2) ⇒ dv du = -(v/u)2

mL = -(dv/du) =m2

(b) If a 2D object is placed with its plane ⏊ to principle axis. Its magnification is called

superficial magnification

b mb

a ma

ms = area of image/ area of object = (ma)(mb)/ ab = m2

(c) In case of more than one optical component, the image formed by first component

will act as an object for the second component & so on. So overall magnification

m = I/O = (I1/O)(I2/I1). . . = (m1)(m2(m3). . .

I O u P

V O P I

u v

-m→ inverted image, real image +m→ Erect image, virtual image

m = -v/u m = -v/u

m = -(-v)/(-u) m = -(v/-u)

m = -ve

Page 40: Geometrical Optics Book With Special Lecture

ANUJ PHYSICAL SCIENCES PHYSICS FOR IIT-JEE /AIEEE/AIPMT

Office: 128/652 “Y” Block Kidwai Nagar Kanpur -11 UP, Website: www.apskanpur.in Mob: 09795647531, 09889787531, 0848664308 E-mail:[email protected]

40

Relative position, size and nature of image for different positions of

object:

(1) Object is at infinity:

F P

f

fig: object at infinity

Here, u → ∞

from mirror formula, we have (1/v) + (1/u) = (1/f)

or (1/v) + (1/∞) = (1/-f)

or (1/v) = (1/-f)

⇒ v = -f

also m = ∣v/u∣ = (-f/∞) = 0

thus,

position: At F

nature: Real and inverted

size: Diminished (very small)

Note:

θ

d θ

focal plane

If rays from distant object are not parallel to principal axis, then image is formed on the

focal plane.

(d/f) = tan θ

or d ≈ f θ (if θ is very small)

Page 41: Geometrical Optics Book With Special Lecture

ANUJ PHYSICAL SCIENCES PHYSICS FOR IIT-JEE /AIEEE/AIPMT

Office: 128/652 “Y” Block Kidwai Nagar Kanpur -11 UP, Website: www.apskanpur.in Mob: 09795647531, 09889787531, 0848664308 E-mail:[email protected]

41

(ii) The object lies beyond centre of curvature:

Here ∞ > u > 2f

or 0 < (1/u) < (1/2f)

or {0-(1/f)} < {(-1/f) + (1/u)} < {(-1/f) + (/2f)}

or (-1/f) < (1/v) < (-1/2f)

or –f < v < -2f

also, m = ∣v/u∣ < 1

A

B C B’ F f P

A’

v

u

fig : Object beyond 2f

Position: At centre of curvature

Nature: Real and inverted

Size: small

(iii) Object is at centre of curvature:

A

B

B’ C F f P

A’

u = v

fig: object at centre of curvature

Page 42: Geometrical Optics Book With Special Lecture

ANUJ PHYSICAL SCIENCES PHYSICS FOR IIT-JEE /AIEEE/AIPMT

Office: 128/652 “Y” Block Kidwai Nagar Kanpur -11 UP, Website: www.apskanpur.in Mob: 09795647531, 09889787531, 0848664308 E-mail:[email protected]

42

Here u = -2f

from mirror formula, we have v = uf/ (u-f) = [(-2f)(-f)]/[-2f-(-f)] = -2f

also, m = ∣v/u∣ = 1

thus,

Position: At centre of curvature

Nature: Real and inverted

Size: same

(iv) Object is between focus and centre of curvature:

A

B’ C B F f P

A’ u

v

fig: Object between F and C

Here, f < u < 2f

or (1/f) > (1/u) > (1/2f)

From mirror formula, we have (1/v) = (1/-f) + (1/u)

since, u > f

(1/v) > (1/-f) + (1/f)

or (1/v) > 0

⇒ v < ∞

Again, for u < 2f

or (1/v) < [(1/-f) + (1/2f)]

⇒ (1/v) < (1/2f)

∴ v > -2f

Also, m = ∣v/u∣ > 1

Position: Beyond centre of curvature

Nature: Real and inverted

Size: Enlarged

Page 43: Geometrical Optics Book With Special Lecture

ANUJ PHYSICAL SCIENCES PHYSICS FOR IIT-JEE /AIEEE/AIPMT

Office: 128/652 “Y” Block Kidwai Nagar Kanpur -11 UP, Website: www.apskanpur.in Mob: 09795647531, 09889787531, 0848664308 E-mail:[email protected]

43

(v) Object is kept at focus:

A

C B P

To infinity u = f

Here u = -f

From mirror formula, we get v = [uf/(u-f)] = [(-f)(-f)]/[(-f)-(-f)]= ∞

Also, m = ∣v/u∣ = infinite

Thus,

Position: Infinite

Nature: Real and inverted

Size: Highly magnified

(vi) Object is kept between pole and focus:

A’

A

C F B P B’

U v

f

fig: object between F and pole

Here, u < -f

or (1/u) > (-1/f)

From mirror formula, we have (1/v) = (1/-f) + (1/u)

since, (-1/u) > (1/f)

∴ v is positive

thus,

Position: other side of mirror

Nature: virtual and erect

Size: magnified

Page 44: Geometrical Optics Book With Special Lecture

ANUJ PHYSICAL SCIENCES PHYSICS FOR IIT-JEE /AIEEE/AIPMT

Office: 128/652 “Y” Block Kidwai Nagar Kanpur -11 UP, Website: www.apskanpur.in Mob: 09795647531, 09889787531, 0848664308 E-mail:[email protected]

44

Notes:

C I P O

(i) Concave mirror always forms real image of a virtual object irrespective of its

position. Here, u = +ive, and f = -ive

From mirror equation, we have v = [uf/(u-f)]

v = -ive Hence, image formed is always real.

(ii)

B B’

A P A’ F

Whatever be the position of object in front of convex mirror, image is always formed

behind mirror between pole and focus, small in size, virtual and erect.

(iii)

I P O F P F O I

u < f u > f

Page 45: Geometrical Optics Book With Special Lecture

ANUJ PHYSICAL SCIENCES PHYSICS FOR IIT-JEE /AIEEE/AIPMT

Office: 128/652 “Y” Block Kidwai Nagar Kanpur -11 UP, Website: www.apskanpur.in Mob: 09795647531, 09889787531, 0848664308 E-mail:[email protected]

45

Convex mirror can form real or virtual image of a virtual object depending on its

position.

(iv) In general all situations in spherical mirror can be summarised in u-v graph as

shown below:

y

Virtual image of a real object y Virtual image of a

virtual object

f

-f o u

-f Real image of a virtual object o f u

Real image of a virtual

object

Real image of a real object

Concave mirror Convex mirror

While interpreting these graphs for numerical problems, remember following points:

(a) Object/image before mirror is real so, v and u are negative.

(b) Object/image behind mirror is virtual so, v and u are positive.

Page 46: Geometrical Optics Book With Special Lecture

ANUJ PHYSICAL SCIENCES PHYSICS FOR IIT-JEE /AIEEE/AIPMT

Office: 128/652 “Y” Block Kidwai Nagar Kanpur -11 UP, Website: www.apskanpur.in Mob: 09795647531, 09889787531, 0848664308 E-mail:[email protected]

46

Problem 1(IIT 1988): A short linear object of length b lies along the axis of a

concave mirror of focal length f at a distance 4 from the pole of the mirror. The size of

image is approximately equal to ½

(a) b[(u-f)/f]1/2 (b) b[f/(f-u)]1/2 (c) b[(u-f)/f] (d) b[f/(u-f)]2

Solution:

Concave mirror

(1/v) + (1/u) = (1/f) [mirror formula]

for concave mirror (1/v) + (1/u) = (1/f)

[differentiating]

(-dv/v2) + (-du/u2) = (-df/f2) =0

dv/du = -v2/u2 (1)

(1/v) + (1/u) = (1/f)

(u/v) + (u/u) = (u/f)

(u/v) = (u/f)-1

u/v = (u-f)/f (2)

now using (2) in eq.(1)

dv /du = -[v2/u2]=-[f/(u-f)]2

-dv/du =-[f/(u-f)]2

dv = (-du) [f/(u-f)]2

dv ⇒ size of image, du⇒ size of object = b

dv = b[f/(u-f)]2

size of image dv = b[f/(u-f)]2

hence option (d)

Page 47: Geometrical Optics Book With Special Lecture

ANUJ PHYSICAL SCIENCES PHYSICS FOR IIT-JEE /AIEEE/AIPMT

Office: 128/652 “Y” Block Kidwai Nagar Kanpur -11 UP, Website: www.apskanpur.in Mob: 09795647531, 09889787531, 0848664308 E-mail:[email protected]

47

Problem 2: A pencil is placed 20.0 cm in front of a concave spherical mirror of focal

length f = 15.0 cm. Find the location of the image. State whether the image is real or

virtual, erect or inverted and give its lateral magnification?

Solution:

C F

The ray diagram shows that the image is real that, relative to the object, it is farther

away from the mirror, inverted and larger.

The given parameters are

f = -15 cm

u = -20 cm

From mirror equation (1/v) = (1/f) – (1/u)

= (1/-15) – (1/-20) = -(1/60)

v = -60 cm

because the image distance is negative, the image is real and is formed in front of the

mirror, the reflected rays actually pass through the image.

The lateral magnification is m = -v/u = - [(-60)/ (-20)] = -3

the lateral magnification is negative indicating that the image is inverted with respect to

the object.

Problem 3: If the pencil in above Problem is at 10.0 cm in front of the concave mirror

of focal length 15 cm, characterize the new image.

Solution: The ray diagram shows that the image is virtual, upright and larger relative

to the object. The given parameters are u = -10 cm & f = -15 cm

From mirror equation (1/v) = (1/f) – (1/u) = (1/-15) –(1/-10) = 1/30

v = + 30 cm

The positive sign shows that image is behind the mirror, virtual; the rays appear to

emanate from the image.

The lateral magnification is m = -v/u = - [(+30)/(-10)] = +3

The magnification is positive indicating that the image is upright; m > 1 implies that is

magnified.

Page 48: Geometrical Optics Book With Special Lecture

ANUJ PHYSICAL SCIENCES PHYSICS FOR IIT-JEE /AIEEE/AIPMT

Office: 128/652 “Y” Block Kidwai Nagar Kanpur -11 UP, Website: www.apskanpur.in Mob: 09795647531, 09889787531, 0848664308 E-mail:[email protected]

48

Problem 4: Find out position, size and nature of image of an object of height 2mm

kept between two mirrors in situation as shown in figure after two successive reflection

considering first reflection at concave mirror and then at convex mirror.

M1 M2

B

P1 A P2

20cm

f1= 15 cm f2 =20 cm

50 cm

Solution: consider reflection at concave mirror (M1) u = -20 cm, f = -15 cm

using mirror equation we get, v = uf/(u-f) =[(-20)(-15)]/[(-20)-(-15)] = -60 cm

m1= -v/u = -(-60)/-20 = -3 (inverted)

A’B’= m1(AB) = 6mm

The image (A’B’) formed by concave mirror acts as object for convex mirror. Now,

consider reflection at convex mirror (M2).

M1 M2

P1 B A

A A’’ P2 B 6mm

12mm B’’

20cm

u = +10 cm, f =+20cm

v = vf/(u-f) = [(10)(20)]/[10-20] = -20 cm

m2 = v = -v/u = -20/10 = 2 (Erect)

A’’B’’ = m2(AB) = 12mm

Hence, for final image A’’B’’

Position: 20cm in front of convex mirror (M2)

Nature: Real and inverted

Size: 12mm

Page 49: Geometrical Optics Book With Special Lecture

ANUJ PHYSICAL SCIENCES PHYSICS FOR IIT-JEE /AIEEE/AIPMT

Office: 128/652 “Y” Block Kidwai Nagar Kanpur -11 UP, Website: www.apskanpur.in Mob: 09795647531, 09889787531, 0848664308 E-mail:[email protected]

49

Problem 5:

Y f1= 15 cm f 2 = 20cm

P

2mm

O 20cm

M2

M1 50 cm

Find the co-ordinates of image of point object P formed after two successive reflections

in situation as shown in figure. Considering first reflection at concave mirror and then at

convex mirror.

Solution:

Y f1=15cm f 2=20cm

(20cm, 2mm) P C’’ C’

O A 2mm A’ x

20cm 6mm

M1 P’’ M2 P’(60cm, -6mm)

30 cm (30cm, -14mm)

50cm

60cm

For reflection at concave mirror M1

u = -20 cm, f1 = -15 cm

v1 = uf1/(u-f1) = [(-20)(-15)]/[-20+15] = -60 cm

Magnification (m1) = -v1/u = -[(-60)/(-20)] = -3 (Inverted)

A’P’ = m1(AP) = 3*2 = 6mm

For reflection at convex mirror M2

u = +10 cm, f1 = +20 cm

v2 = uf2/(u-f2) = [(10)(20)]/[10-20] = -20 cm

Magnification (m2) = -v2/u = -[(-20)/(10)] = 2

C’’P’’ = m2(C’P’) = 2*8 = 16mm

so, the co-ordinate of image of point object P (30 cm, -14mm)

Page 50: Geometrical Optics Book With Special Lecture

ANUJ PHYSICAL SCIENCES PHYSICS FOR IIT-JEE /AIEEE/AIPMT

Office: 128/652 “Y” Block Kidwai Nagar Kanpur -11 UP, Website: www.apskanpur.in Mob: 09795647531, 09889787531, 0848664308 E-mail:[email protected]

50

Problem 6: A concave mirror forms on a screen a real image of thrice the linear

dimensions of the object. Object and screen are moved until the image is twice the size

of the object. If the shift of the object is 6 cm then find the shift of the screen and the

focal length of the mirror.

a)36 cm, 54cm, b)54 cm, 36 cm,

c)36 cm, 36 cm, d)None

Solution: Initial magnification = 3

i.e. v1/u1 = 3

v1 = 3u1

f = u1v1/(u1+v1)

f= u1(3u1)/(u1+3u1) = (3/4)u1 (1)

In the second case, m = 2

i.e. v2/u2 = 2

v2 = 2u2

f = u2v2/(u2+v2)

f= u2(2u2)/(u2+2u2) = (2/3)u2 (2)

From eq. (1) and (2) focal length is same

(3/4)u1 = (2/3)u2

or u2 = (9/8)u1 (3)

It is given that the shift of the object = 6 cm= u2- u1

∴ [(9/8)u1]–u1 = 6

[(9/8)–1]u1 = 6

(1/8) u1 = 6

u1 = 6*8 = 48 cm

and v1 = 3 u1 =3*48 = 144 cm

substituting the value of u1 and v1 in equation (1)

f= (3/4)u1

f= (3/4)48 = 36 cm

From equation (3)

u2= (9/8)u1

u2= (9/8)48 = 54 cm

v2= 2u2 = 2*54 = 108 cm

Thus, shift of the screen = v1-v2

v1-v2 = 144-108 = 36 cm

Page 51: Geometrical Optics Book With Special Lecture

ANUJ PHYSICAL SCIENCES PHYSICS FOR IIT-JEE /AIEEE/AIPMT

Office: 128/652 “Y” Block Kidwai Nagar Kanpur -11 UP, Website: www.apskanpur.in Mob: 09795647531, 09889787531, 0848664308 E-mail:[email protected]

51

Problem 7: A thin rod of length f/3 is placed along the principle axis of a concave

mirror of focal length f such that its image just touches the rod. Calculate magnification?

Solution: Since image touches the rod, the rod must be placed with one end at centre

of curvature.

Case (I):

C A F P

A’

For A:

u = -[2f-(f/3)] = -5f/3 f = -f

v = (uf)/(u-f)

v= [(-5f/3)(-f)]/[(-5f/3)-(-f)] = 5f/2

m = Length of the image/Length of the object

m= [vA-vc]/[uA-uc]

m= [(-5f/2)-(-2f)]/[-5f/3-(-2f)]

m= -3/2 Ans.

Case (II):

A C

A’ P

For A: u = -[2f+(f/3)] = -7f/3 f = -f

v = (uf)/(u-f) = [(-7f/3)(-f)]/[(-7f/3)-(-f)] = -7f/4

m= [vA-vc]/[uA-uc] = [(-7f/4)-(-2f)]/[-7f/3-(-2f)]

m= -3/4

Page 52: Geometrical Optics Book With Special Lecture

ANUJ PHYSICAL SCIENCES PHYSICS FOR IIT-JEE /AIEEE/AIPMT

Office: 128/652 “Y” Block Kidwai Nagar Kanpur -11 UP, Website: www.apskanpur.in Mob: 09795647531, 09889787531, 0848664308 E-mail:[email protected]

52

Problem 8: A diamond ring is placed in front of a mirror of radius of curvature. The

image is twice the size of the ring. Find the object distance of the ring.

Solution:

The first question arises: is the mirror concave or convex or either is possible? The

image formed by a convex mirror is always smaller than the object; therefore the mirror

must be concave.

The next question arises: how many positions are there in front of a concave mirror,

where the ring can be placed and produce an image that is twice the size of the object?

There are two places: (1) when the object is placed between centre of curvature and the

focal point, the magnified image is real and inverted. (2) When the object is between the

focal point and the mirror, the magnified image is virtual and upright.

In first case the image is inverted, so the magnification is m = -2, in the second case the

image is upright, so the magnification is m = +2

From mirror equation and magnification equation, (1/v) + (1/u) = (1/f) and m = -v/u

On solving magnification equation we have v = -mu; substituting this expression for v in

mirror equation, we get

(1/u) + [1/(-mu)] = (1/f) or u = [f(m-1)]/m

Applying this result, we obtain

m = -2, u = [f(m-1)]/m = [(-12)(-2-1)]/(-2) = -18 cm

m =+2, u = [f(m-1)]/m = [(-12)(+2-1)]/(+2) = -6 cm

negative signs for object distances indicate that the object is real, lies in front of the

mirror.

Problem 9(IIT – JEE 2007):

Statement 1: The formula connecting u, v and f for a spherical mirror is valid only for

mirrors whose sizes are very small compared to their radii of curvature.

Statement 2: Laws of reflection are strictly valid for plane surfaces, but not for large

spherical surfaces.

(a) Statement 1 is true, statement 2 is true; statement 2 is a correct explanation for

statement 1

(b) Statement 1 is true, statement 2 is true; statement 2 is not a correct explanation for

statement 1

(c) Statement 1 is true, statement 2 is false

(d) Statement 1 is false, statement 2 is true

Solution: Laws of reflection can be applied to any type of surface.

Page 53: Geometrical Optics Book With Special Lecture

ANUJ PHYSICAL SCIENCES PHYSICS FOR IIT-JEE /AIEEE/AIPMT

Office: 128/652 “Y” Block Kidwai Nagar Kanpur -11 UP, Website: www.apskanpur.in Mob: 09795647531, 09889787531, 0848664308 E-mail:[email protected]

53

Problem 10: When the position of an object reflected in a concave mirror of 0.25 m

focal length is varied, the position of the image varies. Plot the image distance as a

function of the object distance, taking the object distance from 0 to +∞

Solution:

Y

Real image

1.0

0.5

0

0.5 1/4

1.0 virtual image

Problem 11: An object is located 6 cm in front of a mirror. The virtual image is located

4 cm away from the mirror and diminished. Find the focal length of the mirror?

Solution:

The first question arises: is the mirror concave or convex or either is possible? Since

both the mirrors form virtual image if the object is within the focal point of the mirror.

A convex mirror always forms a virtual image.

The second question arises that the image is diminished in size as well as virtual; do

these characteristics tighter indicate a concave or convex mirror? A concave mirror

produces a diminished image only when the object is located beyond the centre of

curvature of the mirror. However, the image in this case is real not virtual. A convex

mirror always produces an image that is virtual and smaller than the object.

The focal length of a convex mirror is positive.

The given parameters are u = - 6 cm, v = +4 cm

From mirror equation,

(1/f) = (1/v) + (1/u)

(1/f) = (1/4) + (1/-6) = (1/12)

f = + 12 cm

As expected, focal length is positive.

Page 54: Geometrical Optics Book With Special Lecture

ANUJ PHYSICAL SCIENCES PHYSICS FOR IIT-JEE /AIEEE/AIPMT

Office: 128/652 “Y” Block Kidwai Nagar Kanpur -11 UP, Website: www.apskanpur.in Mob: 09795647531, 09889787531, 0848664308 E-mail:[email protected]

54

Problem 12 (IIT-JEE 2005): A container is filled with water (μ= 1.33) up to a height

of 33.25 cm. A concave mirror is placed 15 cm above the water level and the image of an

object placed at the bottom is formed 25 cm below the water level. The focal length of

the mirror is

(a) 10 cm (b) 15 cm (c) 20 cm (d) 25 cm

15 cm

33.25cm 25cm

μ= 1.33

Solution: Distance of object from mirror

= 15+(33.25/1.33)= 40 cm

distance of image from mirror

= 15+(25/1.33) = 33.8 cm

for the mirror, 1/v + 1/u = 1/f

[1/(-33.8)] +[1/(-40)] =1/f

f = -18.3 cm so most suitable answer is (c)

Problem 13: A small plane mirror is placed 21 cm in front of a concave mirror of focal

length 21 cm. An object is placed 42 cm in front of the concave mirror. If light from the

concave mirror strikes the plane mirror, where is the final image?

Solution: First we will obtain image position from concave mirror, by mirror equation.

The given parameters are

f = -21 cm

u = -42 cm

(1/v) = (1/f) – (1/u) = (1/-21) - (1/-42) = -(1/42) cm

v = -42 cm

The image is 42 cm in front of the mirror. The image formed by the concave mirror is

object for the plane mirror. Now we use the mirror equation for the plane mirror with

f = ∞, u = -[42-21] cm

(1/v) + (1/-21)= (1/∞)

v = 21 cm

The position of the final image is 21 cm in front of the plane mirror.

Page 55: Geometrical Optics Book With Special Lecture

ANUJ PHYSICAL SCIENCES PHYSICS FOR IIT-JEE /AIEEE/AIPMT

Office: 128/652 “Y” Block Kidwai Nagar Kanpur -11 UP, Website: www.apskanpur.in Mob: 09795647531, 09889787531, 0848664308 E-mail:[email protected]

55

Problem 14(IIT- JEE 2007): In an experiment to determine the focal length (f) of a

concave mirror by the u-v method, a student places the object pin A on the principal axis

at a distance x from the pole p. The student looks at the pin and its inverted image from

a distance keeping his/her eye in line with PA. When the student shifts his/her eye

towards left, the image appears to the right of the object pin. Then

(a) x < f (b) f< x< 2f (c) x = 2f (d) x > 2f

Solution:

Image

Object Right

Since object and image move in opposite directions. The positioning should be as shown

in the figure. Object lies between focus and centre of curvature f < x < 2f

Problem 15:

y

P (40 cm, 3 cm)

O

The Co-ordinates of the image of point object P formed by a concave mirror of radius of

curvature 20 cm (consider paraxial rays only) as shown in the figure is

(a) 13.33 cm, -1 cm (b) 13.33 cm, +1 cm

(c) -13.33 cm, +1 cm (d) -13.33 cm, -1 cm

Solution: 1/v-1/u=1/f

1/v-1/40=1/-10

1/v=1/40-1/10

v=-40/3

u=-40, f=-10

v = uf/u-f = (-40)(-10)/-40-(10)=-40/3 cm

m= h1/3=-(40/3)/-40=-1/3

h1=-1cm

Page 56: Geometrical Optics Book With Special Lecture

ANUJ PHYSICAL SCIENCES PHYSICS FOR IIT-JEE /AIEEE/AIPMT

Office: 128/652 “Y” Block Kidwai Nagar Kanpur -11 UP, Website: www.apskanpur.in Mob: 09795647531, 09889787531, 0848664308 E-mail:[email protected]

56

Problem 16: A Large convex spherical mirror in an amusement park is facing a plane

mirror 10 m away a boy of height 1m standing midway between the two sees himself

twice as tall as in plane mirror as in spherical one. In other words, the angle subtended at

the observer by the image in the plane mirror is twice the angle subtended by the image in

the spherical mirror. What is the focal length of convex mirror?

Solution: 1/10 = θ1= 0.1 rad.

θ2

1m

(+5f)/(+5+f) 5 cm 5 cm 5cm 5 cm

(h1 /1) = 5f/ (5+f) ⇒ h1 = f/ (5+f)

θ2= [f/ (5+f)]/5+ [5f/ (5+f)] = f/ [25+10f] = 0.1/2

2f = 2.5 +f

f = 2.5m

Problem 17: A bright point S is on the principal axis of a concave mirror of radius R =

40 cm at d = 30 cm from its pole. At what distance (in cm) in front of the mirror should a

plane mirror be placed so that after two reflections, the rays converge back at point S.

Solution: 1

2 R = 40

x

d=30

For mirror 1

(1/v) + (1/u) = (1/f) y = -30

(1/v) – (1/30) = - (1/20) f = -20

(1/v) = (1/30) – (1/20)

v = -60 cm

so x = 15cm

so distance between two mirrors = 45 cm

Page 57: Geometrical Optics Book With Special Lecture

ANUJ PHYSICAL SCIENCES PHYSICS FOR IIT-JEE /AIEEE/AIPMT

Office: 128/652 “Y” Block Kidwai Nagar Kanpur -11 UP, Website: www.apskanpur.in Mob: 09795647531, 09889787531, 0848664308 E-mail:[email protected]

57

Problem 18:

F M

O O F

N

(a) (b)

(a) A ray of light falls on a concave spherical mirror, as shown in figure (a). Trace the

path of the ray further

(b) A ray of light falls on a convex mirror, as shown in figure (b). Trace the path of the

ray further

Solution: (a) consider a beam of rays parallel to the ray MN. The beam reflected from

the mirror will converge at the secondary focus F, which lies in the focal plane. Drawing

the ray DO ∥ MN parallel to the centre of the mirror, we find the secondary focus F’. The

ray NK is the one we are looking for fig. (c)

F M M

O F K

N F’ K M’

N

(c) (d)

Method 2:

F’

O F

(e)

Choose an arbitrary point M on the ray MN and with the aid of characteristic rays

construct its image M’. The required ray NK passes through this point (see in fig. (d))

(b) the paths of the rays are shown in fig. (e)

Page 58: Geometrical Optics Book With Special Lecture

ANUJ PHYSICAL SCIENCES PHYSICS FOR IIT-JEE /AIEEE/AIPMT

Office: 128/652 “Y” Block Kidwai Nagar Kanpur -11 UP, Website: www.apskanpur.in Mob: 09795647531, 09889787531, 0848664308 E-mail:[email protected]

58

Problem 19: Point S’ is the image of a point source of light S in a spherical mirror

whose optical axis is N1N2. Find by construction the position of the centre of the mirror

and its focus.

S

N1 S’ N2

Solution: Since the ray incident on the mirror at its pole is reflected symmetrically

with respect to the major optical axis, let us plot point S1 symmetrical to S’ and draw ray

SS1 until it intersects the axis at point P. This point will be the pole of the mirror.

The optical centres C of the mirror can obviously be found as the point of intersection of

ray SS’ with axis NN’.

S M

S1

N C F P N’

S’

The focus can be found by the usual construction of ray SM parallel to the axis. The

reflected ray must pass through focus F (lying on the optical axis of the mirror) and

through S’.

Problem 20: A point source S is placed midway between two converging mirrors

having equal focal length f as shown in figure. find the values of d for which only one

image is formed.

Solution:

S S

d d

When S is placed at the common focus of mirrors, the rays after reflection from one

mirror incident parallel on to the second mirror, which finally intersect at focus of the

mirror. Thus there will be only one image. In this case the value of d will be 2f. When S is

placed at the centre of curvature, the image will form at the same point, so in this case

the value of d will be 2f +2f = 4f

Page 59: Geometrical Optics Book With Special Lecture

ANUJ PHYSICAL SCIENCES PHYSICS FOR IIT-JEE /AIEEE/AIPMT

Office: 128/652 “Y” Block Kidwai Nagar Kanpur -11 UP, Website: www.apskanpur.in Mob: 09795647531, 09889787531, 0848664308 E-mail:[email protected]

59

Problem 21: The positions of optical axis N1N2 of a spherical mirror, the source and

the image are known (fig. (a)) find by construction the positions of the centre of the

mirror, its focus and the pole for the cases: (a) A- source, B-image; (b) B- source, A-

image.

B

A

N1 N2

(a)

Solution:

B B

A M A M

N1 C F P N2 N1 C F P N2

A’ A’

(b) (c)

(a) Let us construct, as in the previous example, the ray BAC and find point C (optical

centre of the mirror) [fig. (b)]. Pole P can be found by constructing the path of the ray

APA’ reflected in the pole with the aid of symmetrical point A’. The position of the

mirror focus F is determined by means of the usual construction of ray AMF parallel to

the axis.

(b) The construction can also be used to find the centre C of the mirror and pole P

[fig. (c)]. The reflected ray BM will pass parallel to the optical axis of the mirror.

For this reason, to find the focus, let us first determine point M at which straight line

AM, parallel to the optical axis, intersects the mirror and then extend BM to the point of

intersection with the axis at the focus F.

Page 60: Geometrical Optics Book With Special Lecture

ANUJ PHYSICAL SCIENCES PHYSICS FOR IIT-JEE /AIEEE/AIPMT

Office: 128/652 “Y” Block Kidwai Nagar Kanpur -11 UP, Website: www.apskanpur.in Mob: 09795647531, 09889787531, 0848664308 E-mail:[email protected]

60

Problem 22: A converging mirror M1, a point source S and a diverging mirror M2 are

arranged as shown in the figure. The source is placed at a distance of 30 cm from M1.

The focal length of each of the mirror is 20 cm. Consider only the images formed by a

maximum of two reflections. It is found that one image is formed on the source itself.

(a) Find the distance between the mirrors

(b) Find the location of the image formed by the single reflection from M2.

Solution: For mirror M1: u = -30 cm, f = -20 cm

By mirror formula, (1/u) + (1/v) = (1/f), we have

(1/-30) + (1/v) = (1/-30)

which on solving gives, v = -60 cm

For mirror M2: The image formed by mirror M1is a virtual object for mirror M2. Let it is

a distance x from the pole of mirror M2.

Thus u2 = +x, v2 = - (30-x)

Again by mirror formula. We have

(1/x) + (1/-(30-x)) = (1/20)

which on solving gives x = 10 cm or 60 cm,

x = 60 cm is not possible, thus x = 10 cm.

(a) Thus the separation between the mirrors = 60-10= 50 cm

(b) The image formed by mirror M2 is at a distance 10 cm

M1 M2

P1 S P2 I

30 cm (30-x) x

60 cm

Page 61: Geometrical Optics Book With Special Lecture

ANUJ PHYSICAL SCIENCES PHYSICS FOR IIT-JEE /AIEEE/AIPMT

Office: 128/652 “Y” Block Kidwai Nagar Kanpur -11 UP, Website: www.apskanpur.in Mob: 09795647531, 09889787531, 0848664308 E-mail:[email protected]

61

Problem 23:

A B

M

2R O 2R

m m

Two concave mirrors equal radii of curvature R is fixed on a stand facing opposite

directions. The whole system has a mass m and is kept on a frictionless horizontal table.

Two blocks A and B, each of mass m, are placed on the two sides of the stand. At t =0, the

separation between A and the mirror is 2R and also the separation between B and the

mirror is 2R. The block B moves towards the mirror at a speed v. All collisions which

take place are elastic. Taking the original position of the mirrors standard system to be x

= 0 and x-axis along AB, find the position of the image of A and B at;

(a) t = (R/v) (b) t = (3R/v) (c) t = 5R/v

Solution: (a) At t = R/v

For block A, u = -2R

∴ (1/v) + (1/-2R) = 2/-R

or v = -2R/3

For block B: The distance travels by block B in time (R/v) is R

Thus u = -R

(1/v) + (1/-R) = (2/-R)

or v= -R

The x- coordinate of the image of the block with respect to the mirror will be +R

(b) At t = 3R/v

The block B will collide with the stand after time 2R/v.

After collision block B becomes at rest and mirror starts moving with the same velocity

v. In the remaining time R/v, the distance moved by the mirror is R.

The position of blocks and mirror are shown in fig. (b)

Page 62: Geometrical Optics Book With Special Lecture

ANUJ PHYSICAL SCIENCES PHYSICS FOR IIT-JEE /AIEEE/AIPMT

Office: 128/652 “Y” Block Kidwai Nagar Kanpur -11 UP, Website: www.apskanpur.in Mob: 09795647531, 09889787531, 0848664308 E-mail:[email protected]

62

A B

M

R O R

(b)

m m

At this time the blocks lie at the centre of curvature of the respective mirrors. Their

images will form at the centres of curvature. So their coordinates are:

For block A, x = -R

For block B, x = +R

(c) At t = 5R/v

The block B will collide to the mirror after a time (2R/v), thereafter mirror starts

moving towards block A with velocity v, At t = 4R/v, the mirror will collide with block A

and stops after collision. The positions of blocks and mirror are shown in fig. (c)

A B

M

X

R O 2R 2R

(c)

m m

For block A: Its image will form on the same place.

Therefore the positions of the blocks are xA = -3R

For block B: u = -2R

(1/v) + (1/-2R) = (2/-R)

v = -2R/3

The co-ordinates of B : – (2R-(2R/3)) = -4R/3

Page 63: Geometrical Optics Book With Special Lecture

ANUJ PHYSICAL SCIENCES PHYSICS FOR IIT-JEE /AIEEE/AIPMT

Office: 128/652 “Y” Block Kidwai Nagar Kanpur -11 UP, Website: www.apskanpur.in Mob: 09795647531, 09889787531, 0848664308 E-mail:[email protected]

63

Problem 24: An object is placed in front of a convex mirror at a distance of 50 cm. A

plane mirror is introduced converging lower half of the mirror is 30 cm, it is found that

there is no parallax between the images formed by two mirrors.

The radius of curvature of the convex mirror is

a) 15 cm b) 20 cm c) 25 cm d) none

O I

20 cm

30 cm 30 cm

Solution:

The distance of the object from the plane mirror is 30 cm and so the distance of its

image is also 30 cm from the mirror.

As image formed by both the mirrors coincide, so distance of image for convex mirror is

=10 cm

By mirror formula (1/u) + (1/v) = (1/f)

(1/f) = (1/10) + (1/-50)

Which on solving give f = 12.5 cm

∴ Radius of curvature R = 2f = 25 cm

Page 64: Geometrical Optics Book With Special Lecture

ANUJ PHYSICAL SCIENCES PHYSICS FOR IIT-JEE /AIEEE/AIPMT

Office: 128/652 “Y” Block Kidwai Nagar Kanpur -11 UP, Website: www.apskanpur.in Mob: 09795647531, 09889787531, 0848664308 E-mail:[email protected]

64

Problem 25:

Y

60 cm 60 cm axis of M 1

O 1 cm x axis of M 2

M1

M2

(a)

Two concave mirrors each of radius of curvature 40 cm are placed such that their

principal axis are parallel to each other and at a distance of 1 cm to each other. Both the

mirrors are at a distance of 100 cm to each other. Consider first reflection at M1 and

then at M2, find the coordinates of the image thus formed. Take location of object as the

origin.

Solution:

Y

60 cm 40 cm

I1 I2 x

30 cm O 12 cm 28 cm

M1 70 cm M 2

Using mirror formula for first reflection:

(1/f) = (1/v) + (1/u) ⇒ (1/-20) = (1/v) + (1/-70)

⇒ (1/v) = (1/60) –(1/20) ⇒ v = -30 cm

Using mirror formula for first reflection:

(1/f) = (1/v) + (1/u) ⇒ (1/-20) = (1/v) + (1/-70)

(1/v) = (1/70) – (1/20) = [(2-7)/140]

⇒ v = -(140/5) = -28 cm

Height of I2 ⇒ m = (-30/-60) = I2/-1 ⇒ I1 = (1/2) cm

Height of first image from x-axis = 1 + (1/2) = 3/2 cm

Height of I2 ⇒ m = (-28/-70) = 2I2/3

Page 65: Geometrical Optics Book With Special Lecture

ANUJ PHYSICAL SCIENCES PHYSICS FOR IIT-JEE /AIEEE/AIPMT

Office: 128/652 “Y” Block Kidwai Nagar Kanpur -11 UP, Website: www.apskanpur.in Mob: 09795647531, 09889787531, 0848664308 E-mail:[email protected]

65

⇒ I2 = [3*28]/[2*70]

I2= -0.6 cm co-ordinate of I2= (12 - 0.6)

Problem 26:

Y

910

O x

20 cm

A point object is placed at the centre of curvature of a concave mirror (take as origin). A

plane mirror is also placed at a distance of 10 cm from the object as shown. Consider

two reflection first at plane mirror and then at concave mirror. Find the coordinate of

the image thus formed.

Solution: 20 cm Y

A 910 I2

20 tan 10 10 O 20/3 40/3 x

I1 B 10 cos 10 10cos10

20 cm

Distance of I1 from mirror = 40 cm

(1/f) = (1/v) + (1/u)

⇒ (1/-10) = (1/v) + (-1/-140)

⇒ (1/v) = (1/40) – (1/10)

⇒ (1/v) = [(1-4)/40]

⇒ v = -40/3 cm

Using magnification formula m = I/O = -v/u

Page 66: Geometrical Optics Book With Special Lecture

ANUJ PHYSICAL SCIENCES PHYSICS FOR IIT-JEE /AIEEE/AIPMT

Office: 128/652 “Y” Block Kidwai Nagar Kanpur -11 UP, Website: www.apskanpur.in Mob: 09795647531, 09889787531, 0848664308 E-mail:[email protected]

66

⇒ I/-20tan 10 = -[(-40/30)/-40]

⇒ I = 20 tan 10/3 = (20/3) tan (π/180) ≈ (20/3)( π/180) = π/27

co-ordinate of image = [(20/3), (π/27)]

Problem 27:

(0, 1)

C

The image of an object kept at a distance 30 cm in front of a concave mirror is found to

coincide with itself. If a glass slab (μ = 1.5) of thickness 3 cm is introduced between the

mirror and the object, then (i) Identity, in which direction the mirror should be

displaced so that the final image may again coincide with the object itself.

(ii) Find the magnitude of displacement

Solution:

3 cm

O C

1cm 30 cm

μ

(i) Since the apparent shift occurs in the direction of incident light, therefore, the

mirror should be displaced away from the objects.

(ii) (ii) The magnitude of displacement is equal to the apparent shift, i.e.

s = t [1- (1/μ)] = 3[1-(1/(3/2))] =1 cm

Page 67: Geometrical Optics Book With Special Lecture

ANUJ PHYSICAL SCIENCES PHYSICS FOR IIT-JEE /AIEEE/AIPMT

Office: 128/652 “Y” Block Kidwai Nagar Kanpur -11 UP, Website: www.apskanpur.in Mob: 09795647531, 09889787531, 0848664308 E-mail:[email protected]

67

Lecture-4

Geometrical Optics

Physics for IIT - JEE

Velocity of image in the plane mirror

Velocity of image in spherical mirror

Page 68: Geometrical Optics Book With Special Lecture

ANUJ PHYSICAL SCIENCES PHYSICS FOR IIT-JEE /AIEEE/AIPMT

Office: 128/652 “Y” Block Kidwai Nagar Kanpur -11 UP, Website: www.apskanpur.in Mob: 09795647531, 09889787531, 0848664308 E-mail:[email protected]

68

Velocity of image in plane mirror:

XOM = x coordinate of object with respect to mirror

XIM = x coordinate of image with respect to mirror

YOM = y coordinate of object with respect to mirror

YIM = y coordinate of image with respect to mirror For plane mirror:

y

O I

x

X Coordinate: XOM = - xIM

(d/dt) XOM=-(d/dt) XIM

(Vx) OM= - (Vx ) IM

[V0-VM]x = - [VI-VM]x

VOX – VMX = -VXI +VXM

VIX = 2 VMX - VOX

(VI)X = 2(VM)X - (V0)X

Y Coordinate: YOM = YIM

(VOM)Y = (VIM)Y

Note: In nutshell, for solving numerical problems involving calculation of velocity of

image of object with respect to any observer, always calculate velocity of image first

with respect to mirror using following points:

(VIM) ‖ = VOM ‖ & VIM) += - (VOM) +

(VIM) = (VIM ‖+ VIM) +

Velocity of image with respect to required observer is then calculated by using equation

VAB==VA-VB.

Page 69: Geometrical Optics Book With Special Lecture

ANUJ PHYSICAL SCIENCES PHYSICS FOR IIT-JEE /AIEEE/AIPMT

Office: 128/652 “Y” Block Kidwai Nagar Kanpur -11 UP, Website: www.apskanpur.in Mob: 09795647531, 09889787531, 0848664308 E-mail:[email protected]

69

Problem 1: A point object is moving with a speed v before an arrangement of two

mirrors as shown in figure find the velocity of image in mirror M1with respect to image

in mirror M2

a)2v sin θ b) 2v cos θ c)2v tan θ d) v sin θ

V M1

M2 θ

Solution:

Velocity of image in mirror M1& M2 is as shown in fig.

θ V cos θ M1

O V

M2 V sin θ θ

V cos θ

Image in mirror 2

Image in mirror 1

V

I2 V sin θ

V sin θ V

V cos θ

I1

V12=velocity of I1with respect to I2 = v1-v2 = 2v sin θ

Page 70: Geometrical Optics Book With Special Lecture

ANUJ PHYSICAL SCIENCES PHYSICS FOR IIT-JEE /AIEEE/AIPMT

Office: 128/652 “Y” Block Kidwai Nagar Kanpur -11 UP, Website: www.apskanpur.in Mob: 09795647531, 09889787531, 0848664308 E-mail:[email protected]

70

Problem 2: A point object is moving with speed of 10m/s in front of a mirror moving

with a speed of 3 m/s as shown in figure. Find the velocity of image of the object with

respect to mirror, object & ground.

300 3 m/s

10 m/s

(1) The velocity of image of the object with respect to mirror

(a) (5√3+3)î-5

(b) (5√3+3)î+5

(c) (5√3+3) -5î

(d) None

(2) Velocity of image of the object with respect to object

(a) (10√3+6)î

(b) (10√3+6)

(c) (10√3-6)î

(d) None

(3)The velocity of image of the object with respect to ground

(a) (5√3+6)î-5

(b) (5√3+6)î+5

(c) (5√3-6)î+5

(d) None

Page 71: Geometrical Optics Book With Special Lecture

ANUJ PHYSICAL SCIENCES PHYSICS FOR IIT-JEE /AIEEE/AIPMT

Office: 128/652 “Y” Block Kidwai Nagar Kanpur -11 UP, Website: www.apskanpur.in Mob: 09795647531, 09889787531, 0848664308 E-mail:[email protected]

71

Solution: VIM=? , VIO =? , VIG =?

Given: V0=-5√3î-5 = VO + + V0‖

10 cos 30 y -

300 3 m/s +

10 10 sin 30 x

5√3=10 cos 30

Vm=3î

(VIM)+ =(VI-VM) + = - (VOM) + =(VM-VO) +

(VIM) += - (VOM)

(VIM) += (3î+5√3î)

(VIM) ‖= (VOM)‖ =(V0-VM)‖

(VIM) ‖= -5

(VIM) =(VIM) ‖ + (VIM) +

(VIM) =-5 +3i+5√3i = 3+5√3 i=5

VIO =VI – VO for this we need first velocity of image VIM =VI – VM VIM + VM =VI

(5√3+3)î-5 +3î=VI

⇒ VI = (5√3+6)î-5

VIO =VI –VO VIO = (5√3+6)î-5 -(-5√3î- ) = 10√3î +6î+0

VIO =(10√3+6)î

VIG =VI - VG If ground is supposed to be at rest: VG =0

VIG =VI = (5√3+6)î-5

Page 72: Geometrical Optics Book With Special Lecture

ANUJ PHYSICAL SCIENCES PHYSICS FOR IIT-JEE /AIEEE/AIPMT

Office: 128/652 “Y” Block Kidwai Nagar Kanpur -11 UP, Website: www.apskanpur.in Mob: 09795647531, 09889787531, 0848664308 E-mail:[email protected]

72

Problem 3: Find the velocity of image of a moving particle shown in figure.

10 m/s

530 2m/s

Given: 10 cos 53=6, 10 sin 53=8

Solution:

8m/s

θ

10 m/s

(VI)X =2(VM)X –(VO)X

(VI)+ =2(VM)+ -VO+

(VI)X = 2(-2)-6 =-10m/s î

(VI)Y =(VO)y

(VI)Y =8

VI =-10î+8

VI =√82+62 =√164

θ= tan-1(4/5)

Page 73: Geometrical Optics Book With Special Lecture

ANUJ PHYSICAL SCIENCES PHYSICS FOR IIT-JEE /AIEEE/AIPMT

Office: 128/652 “Y” Block Kidwai Nagar Kanpur -11 UP, Website: www.apskanpur.in Mob: 09795647531, 09889787531, 0848664308 E-mail:[email protected]

73

Problem 4:A plane mirror is placed at origin parallel to y-axis facing the positive x-

axis .an object starts from (2m,0,0) with a velocity(2î+2 )m/s. The relative velocity of

image with respect to object is

along

(a) Positive x-axis

(b)Negative x-axis

(c) Positive y-axis

(d) Negative y-axis

Solution:

V = 2 +2

VI V0

450 450

VI =2√2

VI0

-VI0 = 2√2

Towards negative x- axis

VI –VO =VIO

VO =2√2m/s u=2√2√2

VI =2√2m/s VIo=4m/s

Hence the relative velocity of image with respect to object along –ive x-axis.

Page 74: Geometrical Optics Book With Special Lecture

ANUJ PHYSICAL SCIENCES PHYSICS FOR IIT-JEE /AIEEE/AIPMT

Office: 128/652 “Y” Block Kidwai Nagar Kanpur -11 UP, Website: www.apskanpur.in Mob: 09795647531, 09889787531, 0848664308 E-mail:[email protected]

74

Problem 5: Two blocks each of mass m lie on a smooth table. They are attached to

two other masses as shown in the figure. The pulleys and strings are light. An object O is

kept at rest on the table. The sides AB &CD of the two blocks are made reflecting .the

acceleration of two images formed in those two reflecting surface with respect to each

other is

(a) 5g/6 (b) 5g/3 (c) g/3 (d) 17g/6

A C

O

B D

m m

2m

3m

Solution: VI=2VM+VO

aI= 2aM+aO → zero

aA =acceleration of mass AB

T = m aA

3mg-T=3maA

3mg=4maA

aA=(3/4)g

AC =acceleration of mass CD

T = m aC

2mg-T=2maC

2mg=3mac

ac= (2/3)g

Acceleration of image in AB=2aA= 2(3/4) g = (3/2)g

Acceleration of image in CD=2aC = 2(2/3)g = (4/3)g

Acceleration of image in AB with respect to CD is = (3/2)g + (4/3)g

=(17/6)g m/s

Page 75: Geometrical Optics Book With Special Lecture

ANUJ PHYSICAL SCIENCES PHYSICS FOR IIT-JEE /AIEEE/AIPMT

Office: 128/652 “Y” Block Kidwai Nagar Kanpur -11 UP, Website: www.apskanpur.in Mob: 09795647531, 09889787531, 0848664308 E-mail:[email protected]

75

Problem 6: A plane mirror is placed along the xz-plane and an object P is placed at

point (0, a) the mirror rotates about z-axis with constant angular velocity ω.

(1) The position of image as function of time t < (π/2ω)

(a) r = a sin 2ωt î - a cos 2ωt

(b) r =- a sin 2ωt + a cos 2ωt

(c) r = a sin ωt + a cos ωt

(d) r = a sin ωt – a cos ωt

(2) the velocity of image as function of time t<

2

(a) V=2ωa[cos 2ωt î +sin 2ωt ]

(b) V=2ωa[cos ωt î - sin ωt ]

(c) V=ωa [-cos ωt î +sin ωt ]

(d) None

Solution: Y

P (0,a)

X ω

Mirror rotates about z-axis

ω =θ/t θ =ωt

(0,a) (90-θ)

Y=2a cos θ cos θ a θ a cos θ

θ 90 a cos θ

y

X = 2a cos θ sin θ

a cos θ (90-θ) 2a cos θ

θ

sin θ = a cos θ/x

Page 76: Geometrical Optics Book With Special Lecture

ANUJ PHYSICAL SCIENCES PHYSICS FOR IIT-JEE /AIEEE/AIPMT

Office: 128/652 “Y” Block Kidwai Nagar Kanpur -11 UP, Website: www.apskanpur.in Mob: 09795647531, 09889787531, 0848664308 E-mail:[email protected]

76

x= 2a cos θ sin θ

y=2a cos θ cos θ

x=2a cos θ sin θ

y=-(2a cos2θ -a) = -a[2 cos2θ-1] = -a cos 2θ

X=a sin 2θ

Y=-a cos 2θ

(r1/v) =a sin 2θ –a cos 2θ = a sin 2ωt -a cos 2ωt

= 2aω cos 2ωt +2aω sin 2ωt

Problem 7: The mirror of length 2l makes 10 revolutions per minute about the axis

crossing its midpoint O and perpendicular to the plane of the figure. there is a light

source in point A and an observer in point B of the circle of radius R drawn around

centre O (AOB = 900)

(a) Along what curve does the virtual image of the point-like light source A move?

(b) At what speed does the virtual image of A move?

(c) What is the ratio l/R if the observer B first sees the light source when the angle of

the mirror is ϕ = 150?

Solution:

A A

300

R 450

150 900

O R B

(a) circle

(b) ωM = 10(2π/60) = π/3 rad./s

ω1 = 2 ωM = 2(π/3) rad./s

v1= ω1R =(2π/3)R m/s

(c) In ∆OA’C sin 300/l = sin 1350/R

⇒ 1/2l = 1/R√2

⇒ l/R = 1/√2

Page 77: Geometrical Optics Book With Special Lecture

ANUJ PHYSICAL SCIENCES PHYSICS FOR IIT-JEE /AIEEE/AIPMT

Office: 128/652 “Y” Block Kidwai Nagar Kanpur -11 UP, Website: www.apskanpur.in Mob: 09795647531, 09889787531, 0848664308 E-mail:[email protected]

77

Velocity of image in spherical mirror:

(1/v) + (1/u) = (1/f) (1/v) + (1/u) = (1/f)

(u/u)+ (u/v) = (u/f) (v/v)+ (v/u) = (v/f)

1+(- 1/m) = (u/f) 1+(-m)= (v/f)

1-(u/f)= 1/m 1 – m =(v/f)

(f-u)/f = 1/m 1-(v/f) = m

m = f/(f-u) (f-v)/f =m

Let pole of mirror be origin of co-ordinate system X-axis be the principle axis of mirror

Y-axis is ⏊ to principle axis.

Object is placed such that incident rays travels along + X-axis.

(Vom)y Y

Vom

o (Vom)x

P X

X →∥, y → ⏊

From mirror equation:(1/v) + (1/u) = (1/f)

[1/xIm] +[1/xom] = (1/f)

Diff both sides with respect to t

-(1/x2IM)(dxIM/dt) – (1/x2OM)(dxOM/dt) =0

- (1/x2IM)(VIM)x = (1/x2OM) (VOM)x

(VIM)x = -[(x2IM/x2OM)] VOM

(VIM)x = - m2 (VOM)x (1)

As we know that m=f/(f-u)=[yIM/yOM] → m = I/O = v/u

yIM = f/(f-u) yOM

diff with respect to t:

(d/dt)(yIM) = (d/dt)( f/(f-u)yOM

(VIM)Y = (f/(f-u)=(VOM)y + (YOM)(d/dt)( f/(f-u))

(VIM)Y = (f/(f-u))(VOM)y + (YOM)[-f(d/dt)(f-u)+(f-u)(df/dt)]/[(f-u)2]

(VIM)Y = (f/(f-u)(VOM)y + (YOM)[f/(f-u)2 (du/dt)]

(VIM)Y = f/(f-u)(VOM)y + (YOM)[ f/(f-u)2 (du/dt)] (2)

Page 78: Geometrical Optics Book With Special Lecture

ANUJ PHYSICAL SCIENCES PHYSICS FOR IIT-JEE /AIEEE/AIPMT

Office: 128/652 “Y” Block Kidwai Nagar Kanpur -11 UP, Website: www.apskanpur.in Mob: 09795647531, 09889787531, 0848664308 E-mail:[email protected]

78

Note:

(du/dt)→ Zero, If u is constant with time.

(du/dt) = +ive, If u is increasing with time.

(du/dt) = -ive, If u is decreasing with time

Particular Case: 1st if object is placed on the principle axis YOM = 0

(VIM)Y = f/(f-u)(VOM)y (2a)

Case 2nd: if object is not on principle axis but moving parallel to principle axis

(VOM)y =0 (VIM)Y = - (YOM)[(f/(f-u)2)(du/dt)] (2b)

Case 3rd: If object is on principle axis but moving along it then (VIM)x =0

In general for any situation, use mirror equation for component of velocity parallel to

principle axis & magnification formula for component of velocity⏊ to principle axis.

Problem 8: find the velocity of image in situation as shown in fig.

(a) Image will move with speed of 2m/s towards mirror

(b) Image will move with speed of 4m/s towards mirror

(c) Image will move with speed of 2m/s opposite to mirror

(d) None

2m/s

10 m/s

10 m

f = 10cm

Solution:

m=f/(f-u) = 10/(10-(-10)) = ½

(VIM)x = -m2(VOM)x

(VOM)=V0-Vm = 10 -2 = 8î

(VIM)x = -(1/2)2(8)î

(VIM)x = -2i m/s

Hence the image will appear to be moving with a speed of 2m/s towards mirror.

Page 79: Geometrical Optics Book With Special Lecture

ANUJ PHYSICAL SCIENCES PHYSICS FOR IIT-JEE /AIEEE/AIPMT

Office: 128/652 “Y” Block Kidwai Nagar Kanpur -11 UP, Website: www.apskanpur.in Mob: 09795647531, 09889787531, 0848664308 E-mail:[email protected]

79

Problem 9: find the velocity of image in situation as shown in fig.

2m/s

15m/s

530

30 cm

f = 20 cm

(VIM) ‖ =? ⇒ - 44î (VIM) ⏊ =? ⇒ - 24

(VIM) =? ⇒ -44î-24 (VIG) =? ⇒ -46î – 24

Solution:

Y

12 m/s 15 m/s

530 X

9 m/s 2 m/s

V0 =velocity of object = (9î+12 ) m/s

Vm = -2i m/s

m= f/(f-u) =-20/[-20-(-30)]= -2

for velocity component parallel to optical axis

VIM ‖= - m2 (VOM) ‖

= -4[(9î+0)-(-2î)]= -4[11î] = -44 î m/s

For velocity component perpendicular to optical axis (VIM ) + = (VIM)y=f/(f-u)(VOM)y -YOM[f/(f-u)]2 (du/dt)→ zero(u is const with time)

(VIM ) + = (-2) (12 )

(VIM ) + = -24

(VIM ) = (VIM ) ‖+ (VIM ) +

(VIM ) = -44 i-24 m/s

(VIM ) = VI - VM

VI = (VIM ) + VM = -44î-24 +(-2î)

VIG=(-44î-24 )m/s

Page 80: Geometrical Optics Book With Special Lecture

ANUJ PHYSICAL SCIENCES PHYSICS FOR IIT-JEE /AIEEE/AIPMT

Office: 128/652 “Y” Block Kidwai Nagar Kanpur -11 UP, Website: www.apskanpur.in Mob: 09795647531, 09889787531, 0848664308 E-mail:[email protected]

80

Problem 10: A particle is dropped along the axis from height

2 on a concave mirror of

focal length f as shown in figure the acceleration due to gravity is g. Find the maximum

speed of image.

(a) (3/4) √3fg (b) (3/2) √3fg

(c) (3/4) √fg (d) (3/2) √fg

Solution:

t = 0

f/2 t=t (f/2)-(gt2/2)

v = gt

f

VIM ‖= -m2 (VOM) ‖

m=f/(f-u) =-f/[-f-{-(f/2)-(gt2/2)}] =-f/-f+[(f/2)-(gt2/2)]

m=-f/[-(f/2)-(gt2/2)] = -f/[-(f/2)+(gt2/2)] =2f/(f+gt2)

m = 2f/[f +gt2],

VOM =gt

VIM ‖= -[2f/f+gt2] (gt)

VIM ‖= -[4f2gt]/[f+gt2]2 (1)

For obtaining the maximum speed we have to differentiate equation(1) with respect

to t VIM ‖ is function of time

(d VIM/dt)‖ =0 for velocity of image maximum.

We get t = √f/3g

(VIM)max =-[4f2gt/(f+gt2)2

(VIM)max =-(3/4)√3fg

Page 81: Geometrical Optics Book With Special Lecture

ANUJ PHYSICAL SCIENCES PHYSICS FOR IIT-JEE /AIEEE/AIPMT

Office: 128/652 “Y” Block Kidwai Nagar Kanpur -11 UP, Website: www.apskanpur.in Mob: 09795647531, 09889787531, 0848664308 E-mail:[email protected]

81

Problem 11: A gun of mass M fires a bullet of mass m with a horizontal speed V. The

gun is fitted with a concave mirror of focal length f facing towards the receding bullet.

Find the speed of separation of the bullet just after the gun was fired.

(a) 2V[1+(m/M)] (b) 2V [1-(m/M) ] (c) V[1+m/M] (d) V[1-m/M]

Solution:

M

V’

m

V

Bullet object

Gun Concave

m v bullet

M V’ Gun(concave)

when the bullet moves in forward direction gun with concave mirror moves in backward

direction.

Applying conservation of linear momentum

MV’ +mV =0

V’ = - mV/M (1)

(VIM ‖) = - (m2) VOM

m = f/(f-u) when just after gun was fired, u=0 , hence m=1

(VIM ‖) = - (1)2 VOM = -[ V0 - Vm]

(VIM ‖) =- [Vî – (-m/M)Vȋ)] =-V[1+(m/M)]î

(VIM ‖) = -V [1+(m/M)] î

As here m=1 (VIM ‖) = - (VOM)

Hence the speed of separation between bullet & gun are

(VIM ) - (VOM) =( VIM ) - (-VIM) = 2VIM

Speed of separation =-2v [1+(m/M)]

Hence speed of separation between bullets as image will be 2V [1+(m/M)]

Page 82: Geometrical Optics Book With Special Lecture

ANUJ PHYSICAL SCIENCES PHYSICS FOR IIT-JEE /AIEEE/AIPMT

Office: 128/652 “Y” Block Kidwai Nagar Kanpur -11 UP, Website: www.apskanpur.in Mob: 09795647531, 09889787531, 0848664308 E-mail:[email protected]

82

Problem 12:

Mirror

(a)

u =√2m/s

450

An elevator at rest which is at 10th floor of a building is having a plane mirror fixed to its

floor. A particle is projected with a speed √2 m/s and at 450 with the horizontal as

shown in the figure. at the very instant of projection, the cable of the elevator breaks

and the elevator starts falling freely. What will be the separation between the particle

and its image 0.5 second after instant of projection?

Solution:

P

P’

(b)

1/√2m

450 y

vE/g =-gt vp/g = +(1-gt)

u sin θ = √2 sin 450= 1m/s

u cos θ = √2 cos 450= 1m/s

vP/E = + ∣ vP/E∣ = √2m/s

in 0.5 sec = √2(0.5) = 1/√2m

sin 450= y/(1/√2) y = ½ = 0.5m

PP’=1m

Page 83: Geometrical Optics Book With Special Lecture

ANUJ PHYSICAL SCIENCES PHYSICS FOR IIT-JEE /AIEEE/AIPMT

Office: 128/652 “Y” Block Kidwai Nagar Kanpur -11 UP, Website: www.apskanpur.in Mob: 09795647531, 09889787531, 0848664308 E-mail:[email protected]

83

Problem 13:

A

B a = 2.00 m/s2

M

m

m

Consider the situation shown in fig. The elevator is going up with an acceleration of 2

m/s2 and the focal length of the mirror is 12 cm. All the surfaces are smooth and the

pulley is light. The mass-pulley system is released from rest (with respect to the

elevator) at t = 0 when the distance of B from the mirror is 42 cm. Find the distance

between the image of the block B and the mirror at t = 0.2s. Take g = 10m/s2 .

Solution: Let us assume that the acceleration of blocks A and B is ‘a’ with respect to lift

and a1 is acceleration of lift.

Consider block B : System: Block (B) Frame of reference: Lift

mg + maL –T = ma (1)

Now, consider block A: System: Block (A) Frame of reference: Lift

N = mg + maL (2)

T = ma (3)

on adding eq.(1) and (2), we get

a = (g +aL)/2 = (10+2)/2 = 6 m/s2

T N

a T a

maL mg

maL mg Force diagram Acceleration diagram

Force diagram Acceleration diagram

∴ Distance fallen by block (B) = ½ at2 = ½ *6(0.2)2 = 0.12m or 12 cm

Now, consider reflection at convex mirror u = -(42-12) = -40 cm

f = +12 cm

v = uf/ (u-f) = [(-40)(12)]/[-40-12]= 8.75cm

Therefore, the distance between the image of block (B) and mirror is 8.57 cm.

Page 84: Geometrical Optics Book With Special Lecture

ANUJ PHYSICAL SCIENCES PHYSICS FOR IIT-JEE /AIEEE/AIPMT

Office: 128/652 “Y” Block Kidwai Nagar Kanpur -11 UP, Website: www.apskanpur.in Mob: 09795647531, 09889787531, 0848664308 E-mail:[email protected]

84

Problem 14: A small block of mass m and concave mirror of radius R fitted with a

stand, lie on a smooth, horizontal table with a separation d between them. The mirror

together with its stand has a mass m. The block is pushed at t=0 towards the mirror so

that it starts moving towards the mirror at speed V and collides with it. The collision is

perfectly elastic.

Find the velocity of the image

1) at a time t< d/V

(a)R2 V/[2(d-Vt)-R]2 (b) R2 V/[2(d+ Vt)-R]2

(c)R2 V/[2(d-Vt)-R]2 (d) RV [1+ R2/[2(Vt-d)-R]2

2) at a time t>d/v

(a)R2 V/[2(d-Vt)-R]2 (b) R2 V/[2(d+ Vt)-R]2

(c)R2 V/[2(d-Vt)-R]2 (d) RV[1+ R2/[2(Vt-d)-R]

Solution: (1) t < (d/V)

m

V m

d

u = - (d- Vt)

f = -R/2

We know that VI/m = -m2 VO/m

Here, m = f/ (f-u)

m = (-R/2)/[(-R/2)+(d-Vt)]

m = (-R)/[2(d-Vt)-R]

Velocity of image= VI/m = -m2 VO/m

VI= R2 V/[2(d-vt)-R]2

Page 85: Geometrical Optics Book With Special Lecture

ANUJ PHYSICAL SCIENCES PHYSICS FOR IIT-JEE /AIEEE/AIPMT

Office: 128/652 “Y” Block Kidwai Nagar Kanpur -11 UP, Website: www.apskanpur.in Mob: 09795647531, 09889787531, 0848664308 E-mail:[email protected]

85

(2) t > (d/v)

m

m v

u

Block will collide with mirror assembly after time T = d/V. From conservation of linear

momentum, block and mirror assembly will exchange their momentum i.e., block will

stop and mirror starts moving with velocity V.

U = -V [t-(d/V)]

Also, m = f/(f- u) = [-R/2]/[(-R/2)+V(t-(d/V))]

We know that VI/m = -m2 VO/m

VI –Vm = -m2 VO/m

Let us assume rightward direction as positive

VI –Vm =-m2(-V) or VI = (1+m2)V

VI=V[1+{R2/2(Vt-d)-R2}]

Problem 15(AIEEE 2011): A car is fitted with a convex side view mirror of focal

length 20cm. A second car 2.8m behind the car is overtaking the first car is a relative

speed of 15m/s. The speed of the image of the second car as seen in the mirror of the

first one is?

(a) 1/15 m/s (b) 10 m/s (c) 15 m/s (d) 1/10 m/s

Solution: 1/u + 1/v = 1/f Gives - (1/u2)(du/dt) –(1/v2)(dv/dt) = 0

dv/dt = -(v2/u2)(du/dt) but v/u = f/(u-f)

dv/dt = -[f/(u-f)]2(du/dt)

= {0.2/(-2.8-0.2)}2* 15 = 1/15 ms-1 answer

Problem 16(IIT- JEE 2010): Image of an object approaching a convex mirror of

radius of curvature 20m along its optical axis is observed to move from 25/3 m to 50/7

m in 30 sec. What is the speed of the object in kmh-1?

(a) 3 (b) 4 (c) 5 (d) 6

Solution: using mirror formula, [1/(+25/3)] + [1/(-u1)] = 1/10

or 1/u1 = (3/25) – (1/10) or u1 = 50 m

and 1/(+50/7) +(1/-u2) = 1/10

1/u2 = (7/50) – (1/10) or u2= 25 m

speed of object = (u1 -u2) / time = 25/30 ms-1 = 3 kmh-1

Page 86: Geometrical Optics Book With Special Lecture

ANUJ PHYSICAL SCIENCES PHYSICS FOR IIT-JEE /AIEEE/AIPMT

Office: 128/652 “Y” Block Kidwai Nagar Kanpur -11 UP, Website: www.apskanpur.in Mob: 09795647531, 09889787531, 0848664308 E-mail:[email protected]

86

Lecture-5

Geometrical Optics

Physics for IIT - JEE

Refraction at Plane Surfaces:

Apparent Depth & Normal Shift

Page 87: Geometrical Optics Book With Special Lecture

ANUJ PHYSICAL SCIENCES PHYSICS FOR IIT-JEE /AIEEE/AIPMT

Office: 128/652 “Y” Block Kidwai Nagar Kanpur -11 UP, Website: www.apskanpur.in Mob: 09795647531, 09889787531, 0848664308 E-mail:[email protected]

87

Refraction at plane surfaces:

Snell’s law μ1 sin i = μ2 sin r Sin i/sin r =μ2/μ1 = 1μ2 (1) μ = μD/μR > 1→ relative refractive index (2) μ = μD/μa = Absolute Refractive index (3) 1μ2 = ½μ1 (4) In lens theory μ is used for the refractive index of material of lens relative to the

medium μ = μlens/μmedium

μ >1, μ =1, μ <1 (5) If an object is placed in a medium of refractive index μ1 at an actual distance dAc

from a plane boundary and when seen from a refractive index μ2, appears to be a

distance dAp 2 μ1 = μ1/μ2 =dAc/dAP

μ2

2

1 dAC I dAP μ1

O

Problem 1: Which of the followings statement gives the condition of no reflected

(a) If light is incident normally on the boundary

(b) If refractive index of two media are equal.

(c) a & b both

(d) None of these

μ1 μ i

μ2 μ r i = r

Page 88: Geometrical Optics Book With Special Lecture

ANUJ PHYSICAL SCIENCES PHYSICS FOR IIT-JEE /AIEEE/AIPMT

Office: 128/652 “Y” Block Kidwai Nagar Kanpur -11 UP, Website: www.apskanpur.in Mob: 09795647531, 09889787531, 0848664308 E-mail:[email protected]

88

Apparent Depth & Normal shift:

If a point object is in one medium is observed from other medium and boundary is

plane. Snell’s law: μ1 sin i = μ2 sin r Case Ist : If object is in denser medium seen from Rarer medium

If μ1 > μ2, μ2 = μR Sin i = tan i If the rays OA& OB are close Sin r = tan r enough to see the edge μD sin i = μR sin r μD / μR = μ = dAc /dAp μD tan i = μR tan r dAc = μ dAp μD [P/dAc] = μR [P/dAP]

μD / μR = dAc /dAp >1 dAc > dAp

μ1 = μD P

water

dAc dAP r B

i

This is why an underwater object like stone or fish appears to be at depth than in reality. Normal shift: The distance between object & its image called normal shift if dAc =t dAc / dAp =μ ⇒ dAp =t/μ

Normal Shift: = dAc - dAp = t-t/μ =t(1-1/μ)

Normal Shift =t(1-(1/μ))

Case 2nd: If object in rarer medium is seen from denser medium

I

μ1 = μR r dAp

O r

dAc i i

μ2 = μD P

Page 89: Geometrical Optics Book With Special Lecture

ANUJ PHYSICAL SCIENCES PHYSICS FOR IIT-JEE /AIEEE/AIPMT

Office: 128/652 “Y” Block Kidwai Nagar Kanpur -11 UP, Website: www.apskanpur.in Mob: 09795647531, 09889787531, 0848664308 E-mail:[email protected]

89

μ1 sin i = μ2 sin r

μR sin i = μD sin

sin i/sin r = μD/μR = μ2 /μ1 > 1

μR sin i = μD sin r

μR tan i = μD tan r

μR[P/dAc]=μD [P/dAP]

dAp / dAc = μD / μR >1

dAp > dAc

if dAc =t , dAp / dAc = μ

Normal shift: x = dAp - dAc = μ dAc –dAc = (μ-1) dAc =(μ-1)t

x = (μ-1)t = Normal Shift dAp >dAc

If object is in denser medium seen from If object is in rarer medium seen

Rarer medium from denser medium

dAp < dAc dAp > dAc

Normal Shift = t(1-1/μ) Normal Shift = (μ-1) t

Where t= dAc where t = dAc

If there are number of liquids of different depth one over the other:

d1 μ1

d2 μ2

d3 μ3

dAc = d1+d2+d3

dAp =(d1/μ1) +( d2/μ2)+( d3/μ3)

μ =dAc/dAp = [(d1+d2+d3)/ {(d1/μ1) +( d2/μ2) +( d3/μ3)}]

μ =[Σdi/(Σdi/ui)]

In case of two liquids: d1 = d2

μ =(d +d)/[(d/μ1)+(d/μ2)]

μ =2d/d[(1/μ1)+(1/μ2)]

μ = (2μ1μ2)/(μ1+ μ2)

μ =Harmonic Mean

Page 90: Geometrical Optics Book With Special Lecture

ANUJ PHYSICAL SCIENCES PHYSICS FOR IIT-JEE /AIEEE/AIPMT

Office: 128/652 “Y” Block Kidwai Nagar Kanpur -11 UP, Website: www.apskanpur.in Mob: 09795647531, 09889787531, 0848664308 E-mail:[email protected]

90

Problem 2 : A fish rising vertically to the surface of water in a lake uniformly at the

rate of 3m/s. Observes a king fisher(bird) diving vertically towards the water at a rate

of 9m/s vertically above it. If the refractive index of water is 4/3.

The actual velocity of the dive of the bird is

(a) 4 m/s (b) 4.5 m/s (c) 3 m/s (d) none

Solution:

B’

B Bird μy h

Y

X

F fish

If at any instant the fish is at depth x below the water surface while the bird is at a

height y above the surface, then the apparent height of the bird above the surface as

seen by fish will be

dAp = μy (1)

so that total apparent distance of the bird as seen by fish in water at depth will be

h = x + μy (2)

dh/dt = dx/dt + μ (dy/dt) (3)

(dy/dt) =9 m/s

(dx/dt) =3 m/s

dy/dt=? μ = 4/3

Substituting the values in eq dh/dt = dx/dt + μ (dy/dt) (3)

9 = 3 + μ (dy/dt)

dy/dt = 6/μ =6/(4/a) = (6*3)/4 =9/2 = 4.5 m/s

The actual velocity of the dive of the bird is

dy/dt = 4.5 m/s

Page 91: Geometrical Optics Book With Special Lecture

ANUJ PHYSICAL SCIENCES PHYSICS FOR IIT-JEE /AIEEE/AIPMT

Office: 128/652 “Y” Block Kidwai Nagar Kanpur -11 UP, Website: www.apskanpur.in Mob: 09795647531, 09889787531, 0848664308 E-mail:[email protected]

91

Problem 3: A pole standing in a clear water pond stands in above the water surface;

the pond is 2 m deep. What are the lengths of the shadows thrown by the pole on the

surface and bottom of the pond if the sun is 300 over the horizontal refractive index of

water is 4/3.

(a)√3, √3 (b) √3, 2√3 (c) √2, √3/2 (d) none

Solution:

A

Y1 = 1m

i θ = 300

B

Y2 = 2m

F E x2 x1 C

D x1

r

According to rectilinear propagation of light, length of

Shadow on the surface will be BD = x1

tan θ = tan 30 = y1/x1 = 1/x11/√3 = 1/x1

X1= √3 m (1)

However due to refraction at the surface of water, the length Of shadow of pole at the

bottom will be

CE = √3 + x2 (2)

Now from Snell’s Law

μ sin r = 1 sin i = (90 - θ) = cos θ

μ sin r = cos θ

Sin i = cos θ/μ

x2/[√x22 +y22] = cos θ/μ

x22/[x22 +y22]= cos2θ/μ2

μ2/ cos2θ = x22/[x22 +y22] = 1+ (y2/x2)2

μ2/ cos2θ = 1+ (y2/x2)2

[(μ2/ cos2θ)-1] = (y2/x2)2

[(μ2- cos2θ)/ cos2θ] = (y2/x2)2

x2 =y2cos θ/[√μ2- cos2θ]

x2 = 2 cos 3θ/[√(4/3)2- cos23θ]

x2 =[2+(√3/2)]/[√(16/4)-(3/4)]= √3/[√(64-27)/36] = √3/[√37/36] =√3

X = x1+x2 = 2√3 m (Approx.)

Page 92: Geometrical Optics Book With Special Lecture

ANUJ PHYSICAL SCIENCES PHYSICS FOR IIT-JEE /AIEEE/AIPMT

Office: 128/652 “Y” Block Kidwai Nagar Kanpur -11 UP, Website: www.apskanpur.in Mob: 09795647531, 09889787531, 0848664308 E-mail:[email protected]

92

Problem 4: A person looking through the telescope T just sees the point A on the rim

at a cylindrical vessel when vessel is empty see figure. When the vessel is completely

filled with a liquid of refractive index μ , he observes a mark at the centre of bottom ,

without moving the telescope or vessel. What is the height of the vessel if the radius of

its cross section is R.

(a) h = 2R√[(μ2+1)/4-μ2] (b) h = 2R√[(μ2-1)/4-μ2]

(c) h = 2R√[(μ2)/4+μ2] (d) h = R√[(μ2+1)/μ2-4]

Solution: When filled with liquid the ray from B reaches the telescope so that

T

r

D

i h

A B C

R R

μ sin i = 1 sin r

μ = R/√[R2+h2] = 2R/√[h2+4R2]

μ2/4 = [R2+h2]/[ h2+4R2]

4/μ2 = [h2+4R2]/[ h2+R2] =[ h2+R2 +3R2]=1+[3R2/( h2+R2)]

4/μ2-1=[3R2/( h2+R2)]

(4-μ2)/μ2 = 3R2/( h2+R2)

μ2/ (4-μ2) = (h2+R2)/3R2 = (h2/3R2) +1/3

μ2/ (4-μ2)-(1/3) = h2/3R2

[3μ2-(4-μ2)]/ (4-μ2)3 = h2/3R2

[3μ2-4+μ2]/ (4-μ2) = (h2/R2)

4(μ2-1)/(4-μ2) = (h2/R2)

h2 = R24(μ2-1)/( 4-μ2)

h = 2R√[(μ2-1)/( 4-μ2)]

if 2R =10 cm , μ=1.5

h= 8.45cm

Page 93: Geometrical Optics Book With Special Lecture

ANUJ PHYSICAL SCIENCES PHYSICS FOR IIT-JEE /AIEEE/AIPMT

Office: 128/652 “Y” Block Kidwai Nagar Kanpur -11 UP, Website: www.apskanpur.in Mob: 09795647531, 09889787531, 0848664308 E-mail:[email protected]

93

Problem 5: An object O is locked at the bottom of a tank containing two immiscible

liquids and is seen vertically from above. The lower and upper liquids are of depth

h1&h2 respectively and of refractive indices μ1& μ2 respectively. Location of the position

of the image of the object o from the surface

(a) (h1/μ1)+ (h2/μ2) (b) (h1/μ2) + (h2/μ1)

(c) (h1+h2)/μ1+μ2) (d) none

Solution: D i2

h2 μ2 i2 i C

I2 r

B A

h1 r

μ1 I1

i1

As shown in the figure refraction at the 1st Surface

μ1 sin i = μ2 sin r

For small angle sin θ= tan θ = θ

μ1 tan i = μ2 tan r

μ1 [AB/OB]= μ2 [AB/BI1]

BI1 = μ2/μ1)h1 (1)

For refraction at the second surface

μ2 sin r = 1 sin i2

μ2 tan r = tan i2

μ2 [CD/DI1]= [CD/DI2]

DI2 = DI1/μ2 (2)

DI1 = DB +BI1 = h2 + ( μ2/μ1) h1 (3)

Substituting (3) in (2)

DI2 = [h2+(μ2/μ1)h1]/μ2 = (h2/μ2)+( h1+μ1)

DI2 =(h2/μ2)+( h1+μ1)

Page 94: Geometrical Optics Book With Special Lecture

ANUJ PHYSICAL SCIENCES PHYSICS FOR IIT-JEE /AIEEE/AIPMT

Office: 128/652 “Y” Block Kidwai Nagar Kanpur -11 UP, Website: www.apskanpur.in Mob: 09795647531, 09889787531, 0848664308 E-mail:[email protected]

94

Problem 6: An object o is kept at a depth H below the surface of a liquid of refractive

index μ. What is apparent depth when the angle of vision at the surface is θ.

(a) h=(H/μ)[(μ cos θ)/(√μ2-sin2θ)]3 (b) h=(H/μ)[(μ sin θ)/(√μ2-sin2θ)]3

(c) h=(μ/H)[(μ cos θ)/(√μ2-sin2θ)]3 (d) none

Solution:

b

a θ

h θ

H

μ

i

Here it must be noted that the object when viewed from an angle θ not only shifts

upwards by H-h but also laterally (a-b) with respect to its real position. If h is the

apparent depth

a= H tan i, b = h tan θ

a-b = H tan i – h tan θ

a, b , H , h are independent of small variation in θ & i

0 = H sec2 i di – h sec2 θ dθ

di/dθ = [h sec2θ/H sec2i] ⇒ h =(H sec2 i/sec2θ)(di/dθ) = H(cos2θ/cos2i)(di/dθ)

(1) But as at the surface Snell’s Law

μ sin i = 1 sin θ

μ cos i di = cos θ dθ

(di/dθ = cos θ/μ cos i

(2) Substituting (2) in (1) we get

h= H (cos2θ/cos2i)(di/dθ) = H (cos2θ/cos2i) (cos θ/ μ cos i)

h = H[cos3θ/μ cos3i] (3)

μ sin i = sin θ

sin i = (sinθ)/μ

√1-cos2 i = (sinθ)/μ

1-cos2 i =(sin2θ)/μ

1-(sin θ)/μ2 = cos2 i

(μ2- sin2θ)/μ2= cos2 i

√(μ2- sin2θ)/μ = cos i (4)

Substituting cos i in (3) we get

h= H (cos3θ/μ cos3i) =(H/μ)[(μ3 cos3θ)/(μ2- sin2θ)3/2]

h = H μ cos θ 3/μ μ2- sin2θ 3/2

Page 95: Geometrical Optics Book With Special Lecture

ANUJ PHYSICAL SCIENCES PHYSICS FOR IIT-JEE /AIEEE/AIPMT

Office: 128/652 “Y” Block Kidwai Nagar Kanpur -11 UP, Website: www.apskanpur.in Mob: 09795647531, 09889787531, 0848664308 E-mail:[email protected]

95

Problem 7: A tank contains three layers of immiscible liquids. The first layer is of

water with refractive index 4/3 and thickness 8cm. The second layer is an oil with

refractive index 3/2 and thickness 9cm. While the third layer is of glycerine with

refractive index 2 and thickness 4cm. find the apparent depth of the bottom of the

container.

(a) Final image is 1.4 cm below the glycerine.- air interface

(b) Final image is 1.4 cm below the oil.- glycerine interface

(c) Final image is 1.4 cm below the water.- oil interface

(d) None of the above

Air

Y 4cm

X 9 cm

8 cm

μ3 =2 (3)3 Gly

μ2 = 3/2 (2) Oil

μ1 = 4/3 (1) Water

Solution: A ray of light from the object undergoes refraction at three interfaces

(1) Water oil (2) oil glycerine (3) glycerine air

Water oil interface: μ1 =4/3, μ2= 3/2

(μ1/d1) = (μ2/d2) ⇒d1 = -8 cm

2 = (μ2/μ1)d1 =[(3/2)/(4/3)](-8) =(9/8)8 = -9 cm

Oil glycerine interface: (μ1/d1) = (μ2/d2), μ1 =3/2 , μ2= 2

(μ1/d1) = (μ2/d2), d1 = -(9 +9) = -18 cm

2 = (μ2/μ1)d1 = 2/(3/2)(-18)=-24cm

Glycerine air interface: μ1 = 2 , μ2= 1

d1 = - (24+4) = - 28 cm

d2 =( μ2/μ1)d1

d2= (1/2)(-28) = -14 cm

Thus the final image is 14 cm below the glycerine air interface.

Page 96: Geometrical Optics Book With Special Lecture

ANUJ PHYSICAL SCIENCES PHYSICS FOR IIT-JEE /AIEEE/AIPMT

Office: 128/652 “Y” Block Kidwai Nagar Kanpur -11 UP, Website: www.apskanpur.in Mob: 09795647531, 09889787531, 0848664308 E-mail:[email protected]

96

Problem 8:

Bird

36 cm Air ⇒ n=1

36 cm water n = 4/3

Fish

See the figure

(1) At what distance will the bird appear to fish

(a) 84 (b) 63 (c) 72 (d) none

(2) At what distance will the fish appear to bird

(a) 84 (b) 63 (c) 72 (d) none

Solution:

Concept: Apparent depth = μy

Fish observe Bird Bird observe Fish

μD/μR μR/μD

For fish:

dB = 36+ μ 36 = 36[1+ (4/3)] dB= 36(7/3) = 12* 7 = 84 cm

For bird:

df = 36+ 36 μ =36(1+μ)

= 36(1+ 3/4) = 36(7/4)= 63 cm

Page 97: Geometrical Optics Book With Special Lecture

ANUJ PHYSICAL SCIENCES PHYSICS FOR IIT-JEE /AIEEE/AIPMT

Office: 128/652 “Y” Block Kidwai Nagar Kanpur -11 UP, Website: www.apskanpur.in Mob: 09795647531, 09889787531, 0848664308 E-mail:[email protected]

97

Problem 9 (AIEEE 2011): A beaker contains water up to a height h1 and kerosene

of height h2 above water so that the total height of (water + kerosene) is (h1+ h2).

Refractive index of water is μ1 and that of kerosene is μ2. The apparent shift in the

position of the bottom of the beaker when viewed from above is

(a) [1-(1/μ1)] h2 + [1-(1/μ2)] h1 (b) [1+(1/μ1)]h1+[1+(1/μ2)]h2 (c) [1-(1/μ1)]h1 +[1-(1/μ2)]h2 (c) [1+(1/μ1)]h2-[1+(1/μ2)]h1

Solution: Apparent shift Δh = [1-(1/μ)] h

so apparent shift produced by kerosene Δh1 = [1-(1/μ1)] h1

and apparent shift produced by kerosene Δh2 = [1-(1/μ2)] h2

Δh = Δh1+Δh2 = [1-(1/μ1)] h1 + [1-(1/μ2)] h2

Problem 10(IIT – JEE 2009): A ball is dropped from a height of 20 m above the

surface of water in a lake. The refractive index of water is 4/3. A fish inside the lake, in

the line of fall of the ball, is looking at the ball. At an instant, when the ball is 12.8 m

above the water surface, the fish sees the speed of ball as

(a) 9 ms-1 (b) 12 ms-1 (c) 16 ms-1 (d) 21.33 ms-1

Solution: v = √2gh = √2* 10* 7 = 12ms-1

7.2 m

12.8 m v x

θ

In this case when eye is inside water Xapp = μx

d Xapp / dt = μ(dx/dt) or vapp = μv = (4/3)* 12 = 16 ms-1

Page 98: Geometrical Optics Book With Special Lecture

ANUJ PHYSICAL SCIENCES PHYSICS FOR IIT-JEE /AIEEE/AIPMT

Office: 128/652 “Y” Block Kidwai Nagar Kanpur -11 UP, Website: www.apskanpur.in Mob: 09795647531, 09889787531, 0848664308 E-mail:[email protected]

98

Lecture-6

Geometrical Optics

Physics for IIT - JEE

Refraction at Plane Surface:

Critical Angle

Total Internal Reflection

Page 99: Geometrical Optics Book With Special Lecture

ANUJ PHYSICAL SCIENCES PHYSICS FOR IIT-JEE /AIEEE/AIPMT

Office: 128/652 “Y” Block Kidwai Nagar Kanpur -11 UP, Website: www.apskanpur.in Mob: 09795647531, 09889787531, 0848664308 E-mail:[email protected]

99

Total Internal Reflection (TIR):

The phenomenon:

In case of refraction of light from Snell’s law μ1sin i = μ2sin r

If light is passing from denser to rarer medium through a plane boundary.

Then μ1=μD , μ2 =μr

μD/μr=μ μD sin i = μr sin r

μD sin i = μr sin r if r=90, i=ic

μD/μr = sin i/ sin r = μ>1 μD sin ic = μr sin 90

Sin i= sin r/μ μD sin ic = μr

Sin i α sin r μD/μr=1/ sin ic =μ

∠ i α ∠ r Sin ic = 1/μ

Sin-1 /μ = ic → Critical angle

* so as angle of incidence increases angle of refraction also increases.

* for certain value of i < 90, r will becomes 900 , the value of angle of incidence for which

angle of refraction becomes 900 is called critical angle.

Rarer μr

Denser μD

f Fig: illustrating critical angle & T.I.R. phenomenon

i i = ic i > ic

O

Fig. Illustrating critical angle & TIR phenomenon

If i < ic → refraction, i = ic →critical Angle, i > ic →total internal reflection

μD sin i = μr sin r

(μD/μr) sin i = sin r

μ sin i = sin r

sin i/ sin ic = sin r

* so if i > ic , sin r >1, which is no meaning, r is imaginary simply this situation implies

that refracted ray does not exist.

* so the total light incident on the boundary will be reflected back in to the same

medium from the boundary. This phenomenon is called total internal reflection. (T.I.R)

Page 100: Geometrical Optics Book With Special Lecture

ANUJ PHYSICAL SCIENCES PHYSICS FOR IIT-JEE /AIEEE/AIPMT

Office: 128/652 “Y” Block Kidwai Nagar Kanpur -11 UP, Website: www.apskanpur.in Mob: 09795647531, 09889787531, 0848664308 E-mail:[email protected]

100

Note:

1. For total Internal reflection to take place light must be propagating

from denser to rarer medium.

2. When light is passing from denser to rarer medium. Total internal

reflection will take place only if angle of incidence is greater than a

certain value called critical Angle. Given by θc= sin-1(µ) with μ = μD

3. In case of T.I.R i.e. (100 %) incident light is reflected back into the

same medium there is no loss of intensity. While in case of reflection

from mirrors or refraction from lances there is some loss of intensity

as all light can never be reflected or refracted.

4. Images formed by T.I.R. are much brighter than formed by mirrors or

lenses.

5. θc= sin-1(1/μ),μ=c/v ,μ α (1/v) α (1/λ)

for a given pair of media, critical angle depends on wavelength of

light used. i.e. greater is the wavelength of light, lesser will be the

value of μ & so greater will be the value of critical angle.

6. Critical angle is maximum for red & minimum for violet

sin θc = (1/μ), FR-FV=ω Fy ,1/f =(μ-1)[(1/R1)-(1/R2)],

δv – δR →deviation

7. For a given light, critical angle depends on the nature of pair of

media. Lesser is the value of μ, greater will be the critical angle & vice

versa. Glass-air → θc = 420

Water-air → θc = 490

Glass- water →θc = 630

more gap in refractive index less critical angle

Some examples of TIR:

(1)Shining of air bubble (2) Sparking of diamond

(3) Optical fibre (4) Mirage’s & looming

(5) Duration’s of sun’s visibility

Page 101: Geometrical Optics Book With Special Lecture

ANUJ PHYSICAL SCIENCES PHYSICS FOR IIT-JEE /AIEEE/AIPMT

Office: 128/652 “Y” Block Kidwai Nagar Kanpur -11 UP, Website: www.apskanpur.in Mob: 09795647531, 09889787531, 0848664308 E-mail:[email protected]

101

Problem 1: A beam of light consisting of red, green & blue colours is incident on a

right – angled prism as shown in figure for the above red, green & blue wavelength are

1.39,1.44 & 1.47 respectively.

Which colour will be separated from the other by prism

(a) red (b) green (c) blue (d) all (e) none

Solution: At face AB i=0, So r=0 i.e. no refraction will take place. So light will be

incidence on face AC at an angle of incidence of 450. The face AC will not transmit the

light for which i>θc

Sin i > sin θc, Sin 45 > (1/μ) ⇒ 1/√2 > (1/μ)

μ = √2 = 1.41

μR=1.39 < 1.41

μg=1.44> 1.44 will not transmitted

μb=1.47> 1.47

B

G

450

R 450

C

Now as μR < μ while μG & μB> μ , so red colour will be transmitted through the face AC

while green & blue will be reflected. So the prism will be separated red colour from

green & blue.

Problem 2: When light is incident at the glass and air as shown in the figure. If green

light is just T I. Reflected than the emergent ray in air contains

(a) Yellow, orange, red (b) Violet, indigo, blue

(c) All colours (d) None

Air Green

Glass

white

Page 102: Geometrical Optics Book With Special Lecture

ANUJ PHYSICAL SCIENCES PHYSICS FOR IIT-JEE /AIEEE/AIPMT

Office: 128/652 “Y” Block Kidwai Nagar Kanpur -11 UP, Website: www.apskanpur.in Mob: 09795647531, 09889787531, 0848664308 E-mail:[email protected]

102

Optical fibre:

Problem 3: A particular optical fibre consist of a glass core (index of refraction n1)

surrounded by a cladding (index of refraction n2). n1 > n2. Suppose a beam of light enters

the fibre from air at an angle θ with the fibre axis as shown in figure. Show that the

greatest possible value θ for which a ray can be propagated down the fibre is given by.

(a) θ = sin-1√n12 –n22 (b) θ = sin-1√n22 +n12

(c) θ = cos-1√n12 –n22 (d) θ = cos-1√n12 +n22

n2 n1

A i

r B

θ

Solution: The situation is shown in figure. The light will be propagated down the fibre

if it does not emerge from curved surface. i.e. at P.

i > θc , sin i > sin θc

μ = μD/μr= n1/n2, sin θc = (1/μ) = n2/n1

sin i > n2/n1 (1)

now from Snell’s law

1 sin θ =n1 sin r

r + i = 90 r= 90 – i

sin θ =n1 sin (90 - θ) = n1 cos i

cos i = sin θ /n1

sin i =√1- cos2i = √1- (sin2 θ/n12) (2)

Substituting the value of sin i from equation (2) in equation (1)

sin i = n2/n1 = √1- (sin2 θ/n12)

(n2/n1)2 = 1- (sin2 θ/n12)

[(n22 –n12)/n12] ≤ - (sin2 θ/n12)

(n22 –n12) ≤ - sin2θ

θ ≤ sin-1

√n12 – n2

2 (3)

Page 103: Geometrical Optics Book With Special Lecture

ANUJ PHYSICAL SCIENCES PHYSICS FOR IIT-JEE /AIEEE/AIPMT

Office: 128/652 “Y” Block Kidwai Nagar Kanpur -11 UP, Website: www.apskanpur.in Mob: 09795647531, 09889787531, 0848664308 E-mail:[email protected]

103

Problem 4: A bulb is placed at a depth of 2√7 m in water and floating opaque disc is

placed over the bulb so that the bulb is not visible from surface. What is the minimum

diameter of the disc μ= 4/3 (water)

(a) 6m (b) 12m (c) 10m (d) none

D A

i > θc

h water θ

μ

B

C R

Solution:

As shown in figure light from bulb will not emerge out of water at the edge of the disc

i > θc , sin i > sin θc (1)

Now if R is the radius of disc & h is the height/depth of bulb from it.

Sin i =[R/√R2 +n2],

sin θc = 1/μ

[R/√R2 +n2]= 1/μ

or R >[μ/√μ2-1],

h=2√7, μ=4/3

Rmin > [(2√7)/(4/3)2 -1] = 6m

Hence diameter of the disc = 2R=2*6=12m

Problem 5: A ray of light in a transparent medium falls on a surface separating the

medium from air atom angle of incidence 450 the ray undergoes TIR. If n is the refractive

index of the medium with respect to air, select the possible values of n from the following.

(a) 1.3 (b) 1.4 (c) 1.5 (d) 1.6

Solution: For T.I.R. to take place sin i > sin θc

Sin 450 > sin θc

1/√2 > 1/n

√2 = 1.414

n > 1.414

the possible values of n can be 1.5 or 1.6

Page 104: Geometrical Optics Book With Special Lecture

ANUJ PHYSICAL SCIENCES PHYSICS FOR IIT-JEE /AIEEE/AIPMT

Office: 128/652 “Y” Block Kidwai Nagar Kanpur -11 UP, Website: www.apskanpur.in Mob: 09795647531, 09889787531, 0848664308 E-mail:[email protected]

104

Problem 6: A point source of light is placed at a distance h below the surface of a

large and deep lake. Show that the fraction f of the light energy that escapes directly

from the water surface is independent of n and is given by

(a) f =

2[1- √1- (1/n)2] (b) f =

2[1+ √1- (1/n)2]

(c) f =

2[1- √1+ (1/n)2] (d) f =

2[1+ √1+ (1/n)2]

Solution:

As due to total internal reflection, light will be reflected back into water if i > θc , so

only the position of incident light will escape which pass through the core of angle

θ =2θc .

C

A P y

h

R

Cos θc = h/r

D

O θc

So the fraction of light Escaping

f = area of cap /area of sphere

f =2πRy/4πR2

f = 2πy/4πR = y/2R

f = [(1/2) (R-h)/R]

f = (1/2)[1-(h/R)]

f = (1/2) [1-(h/R)]

f =(1/2) [1- cos θc]

f = (1/2)[1- √1- sin2θc]

Sin θc = 1/n

f = (1/2) [1- √1-(1/n)2]

f = ½[1-√1- (1/n)2]

Page 105: Geometrical Optics Book With Special Lecture

ANUJ PHYSICAL SCIENCES PHYSICS FOR IIT-JEE /AIEEE/AIPMT

Office: 128/652 “Y” Block Kidwai Nagar Kanpur -11 UP, Website: www.apskanpur.in Mob: 09795647531, 09889787531, 0848664308 E-mail:[email protected]

105

Problem 7: A ray of light from a denser medium strikes a rarer medium at an angle

of incidence i, if the reflected and refracted rays are mutually + to each other. What is

the value of critical angle

(a) sin-1 tan i (b) sin-1 cot i (c) sin-1 cos i (d) none

Rarer μR

A C r’ B

Denser μD i r 90

Solution: The situation in accordance with given problem is shown in figure applying

Snell’s law at the boundary at c.

μD sin i = μr sin r’ r’+ r =90

μD sin i = μr sin (90-r) incident < reflected & refracted

μD sin i = μr cos r but i=r reflected ray & refracted ray are + to

from reflection each other r’+ r =90

μD sin i = μr cos i

μD /μr =cos i/ sin i = cot i

1/μ = tan i but by definition θc =sin-1(1/μ) ⇒ θc =sin-1(1/μ) θc =sin-1(tan i)

Problem 8(IIT –JEE 2010): A large glass slab (μ = 5/3) of thickness 8cm is placed

over a point source of light on a plane surface. It is seen that light emerges out of the top

surface of the slab from a circular area of radius R cm. What is the value of R?

(a) 6 cm (b) 7 cm (c) 8 cm (d) 9 cm

Solution: (R/t) = tan θc or R = t (tan θc)

R θc

r

θc

But sin θc = 1/μ = 3/5

tan θc = ¾ so R = ¾ t = ¾ (8cm) = 6 cm

Hence the answer is 6.

Page 106: Geometrical Optics Book With Special Lecture

ANUJ PHYSICAL SCIENCES PHYSICS FOR IIT-JEE /AIEEE/AIPMT

Office: 128/652 “Y” Block Kidwai Nagar Kanpur -11 UP, Website: www.apskanpur.in Mob: 09795647531, 09889787531, 0848664308 E-mail:[email protected]

106

Problem 9(IIT-JEE 2011): A light ray travelling in glass medium on glass-air

interface at an angle of incidence θ. The reflected (R) and transmitted (T) intensities,

both as function of θ, are plotted. The correct sketch is?

100% T 100% T

Intensity

R R

0 θ 900 0 θ 90

0

100% T 100% T

R R

0 θ 900 0 θ 90

0

Solution: After critical angle reflection will be 100% and transmission is 0%. Options

(b) and (c) satisfy this condition. But option (c) is the correct option. Because in option

(b) transmission is given 100% at θ = 00, which is not true.

So correct answer is(c).

Problem 10(IIT – JEE 2008): A light beam is travelling from region 1 to region 4

(refer figure). The refractive indexes in region 1, 2, 3, 4 are n0. n0 / 2, n0/6, n0/8

respectively. The angle of incidence θ for which the beam just misses entering region 4

is

Region 1 Region 2 Region 3 Region 4

θ n0 / 2 n0 / 6 n0 / 8

n0

0 0.2 m 0.6 m

(a) sin-1 (3/4) (b) sin-1 (1/8) (c) sin-1(1/4) (d) sin -1 (1/3)

Solution: Critical angle from region 3 to region 4 sin θc = (n0/8)/ (n0/6) = ¾

now applying Snell’s law in region 1 and region 3 n0 sin θ = (n0/6) sin θc

sin θ = 1/6 sin θc = (1/6)(3/4) = 1/8

so θ = sin-1 (1/8)

Page 107: Geometrical Optics Book With Special Lecture

ANUJ PHYSICAL SCIENCES PHYSICS FOR IIT-JEE /AIEEE/AIPMT

Office: 128/652 “Y” Block Kidwai Nagar Kanpur -11 UP, Website: www.apskanpur.in Mob: 09795647531, 09889787531, 0848664308 E-mail:[email protected]

107

Problem 11(IIT – JEE 2010):

O B

600 135

0 C

P 900

A 750 D

A ray OP of monochromatic light is incident on the face AB of prism ABCD near vertex B

at an incident angle of 600. If the refractive index of the material of the prism is√3,

which of the following is (are) correct?

(a) The ray gets totally internally reflected at face CD

(b) The ray comes out through face AD

(c) The angle between the incident ray and the emergent ray is 900

(d) The angle between the incident ray and the emergent ray is 1200

Solution: √3 = sin 600/ sin r so r=300 θc = sin-1(1/√3) sin θc = 1/√3 = 0.577

At point Q, angle of incidence inside the prism is i = 450

Since sin i= 1/√2 is greater than sin θc = 1/√2 , Ray gets totally internally reflected at

face CD. Path of ray of light after point Q is shown in figure.

600 135

0

O p 450 Q

600 r=30

0 45

0

450

900 30

0 75

0

R 600

From the figure, we can see that angle between incident ray OP and emergent ray RS is

900 therefore, correct options are (a), (b) and (c).

Page 108: Geometrical Optics Book With Special Lecture

ANUJ PHYSICAL SCIENCES PHYSICS FOR IIT-JEE /AIEEE/AIPMT

Office: 128/652 “Y” Block Kidwai Nagar Kanpur -11 UP, Website: www.apskanpur.in Mob: 09795647531, 09889787531, 0848664308 E-mail:[email protected]

108

Problem 12(AIEEE 2009):

θ

A transparent solid cylindrical rod has a refractive index of 2/√3. It is surrounded by

air. A light ray is incident at the mid-point of one end of the rod as shown in the figure.

The incident angle θ for which the light ray grazes along the wall of the rod is

(a) sin-1(1/2) (b) sin-1(√3/2) (c) sin-1(2/√3) (d) sin-1(1/√3)

Solution: sin c = √3/2 (1) sin r= sin (900 - c) = cos c = ½

sin θ/ sin r = μ2/μ1

sin θ =(2/√3)*(1/2)

θ = sin-1(1/√3) :

Problem 13(AIEEE 2004): A light ray is incident perpendicular to one face of a 900

prism and is totally internally reflected at the glass-air interface. If the angle of

reflection is 450, we conclude that the refractive index n

(a) n< 1/√2 (b) n > √2 (c) n> 1/√2 (d) n< √2

450

450

Solution: For total internal reflection from glass-air interface, critical angle c must be

less than angle of incidence. C< i or C < 450 (∵∠i= 450)

but n= 1/ sin c ⇒ c = sin -1(1/n) ∴ sin-1 (1/n) < 450

1/n < sin 450 ⇒ n > 1/ sin 450 n > 1/(1/√2) ⇒ n > √2

Page 109: Geometrical Optics Book With Special Lecture

ANUJ PHYSICAL SCIENCES PHYSICS FOR IIT-JEE /AIEEE/AIPMT

Office: 128/652 “Y” Block Kidwai Nagar Kanpur -11 UP, Website: www.apskanpur.in Mob: 09795647531, 09889787531, 0848664308 E-mail:[email protected]

109

Lecture-7

Geometrical Optics

Physics for IIT - JEE

Prism Theory:

Refraction through prism

Deviation in prism

Maximum deviation in prism

Minimum deviation in prism

Condition of no emergence: (TIR) in prism

Dispersion of light & Causes of dispersion

Angular dispersion & Dispersive power

Dispersion without deviation

Deviation without dispersion

Page 110: Geometrical Optics Book With Special Lecture

ANUJ PHYSICAL SCIENCES PHYSICS FOR IIT-JEE /AIEEE/AIPMT

Office: 128/652 “Y” Block Kidwai Nagar Kanpur -11 UP, Website: www.apskanpur.in Mob: 09795647531, 09889787531, 0848664308 E-mail:[email protected]

110

Prism: Basic facts:

Prism is a transparent medium bounded by any number of surfaces in such a way that

the surfaces from which light emerges are plane and non-parallel.

S

(A) (B) (c) (d)

Generally equilateral, right angled isosceles or right angled prisms are used

A A

450

δ δ

A 900 450 900

C B C B

C B

Equivalent prism Right angled isosceles prism Right angle prism

(A) (B) (c)

Note:

(1) Angle of prism or refractive angle of prism means the angle between the faces on

which light is incident and from which it emerges in above fig. A is the angle of prism.

(2) Angle of deviation means the angle between emergent and incident rays i.e. the

angle through which incident ray turns in passing through a prism. It is represented by

δ in figure.

Page 111: Geometrical Optics Book With Special Lecture

ANUJ PHYSICAL SCIENCES PHYSICS FOR IIT-JEE /AIEEE/AIPMT

Office: 128/652 “Y” Block Kidwai Nagar Kanpur -11 UP, Website: www.apskanpur.in Mob: 09795647531, 09889787531, 0848664308 E-mail:[email protected]

111

(3) If the faces of a prism on which light is incident and from which it emerges are

parallel as in figure then the angle of prism will be zero and as incident ray will emerges

parallel to itself deviation will also be zero. i.e. The prism will act as transparent plate.

A =0

ii

δ = 0

(4) If μ of the material of the prism is equal to that of surroundings, no refraction at its

face will take place & light will pass through it an deviated i.e. δ = 0

i1=r1 i2=r2

r μ1

Hence i1=r1 & i2=r2

r1 r2

μ2 = μ1

μμ

Page 112: Geometrical Optics Book With Special Lecture

ANUJ PHYSICAL SCIENCES PHYSICS FOR IIT-JEE /AIEEE/AIPMT

Office: 128/652 “Y” Block Kidwai Nagar Kanpur -11 UP, Website: www.apskanpur.in Mob: 09795647531, 09889787531, 0848664308 E-mail:[email protected]

112

Deviation in prism:

Refraction through prism: Consider a mono chromatic ray of light incident an

angle i on the face AB of the prism. It gets refracted at an angle r into the prism. After

this the ray incident on other face AC of the prism at an angle r’ and then finally

emerges from this face with angle i’ see figure. A

S

(i-r) P δ

i i’

incident ray

B C emergent ray

E

r i’-r’

r’

From □ APEQ

A +E + 90 + 90 = 360

A+E = 180 (1)

r + r’+ E = 80 (2)

From Δ PQE

from (1) & (2) we get

A= r + r’ (3)

From Δ PQS i – r + i’- r’ = δ

(i + i) – r + r’ = δ deviation angle (4)

For minimum deviation i=i’, r=r’, δ =δm

A = 2r ⇒ r =A/2 from equation (3)

2i -2r = δm from equation (4)

2i – A = δm

= (A+ δm)/2

from Snell’s law

μ = sin i/ sin r = sin {(A+ δm)/2}/ sin (A/2)

Deviation produced by small angled prism or

thin prism. Sin i/ sin r = μ = i/r

μ = {(A+ δm)/2}/ (A/2)

μ = (A+ δm)/A

μ A= A+ δm

(μ - 1) A = δm

δv =(μv -1) A

δR =(μR -1) A for deviation of different colours in white light

δy =(μy -1) A

Page 113: Geometrical Optics Book With Special Lecture

ANUJ PHYSICAL SCIENCES PHYSICS FOR IIT-JEE /AIEEE/AIPMT

Office: 128/652 “Y” Block Kidwai Nagar Kanpur -11 UP, Website: www.apskanpur.in Mob: 09795647531, 09889787531, 0848664308 E-mail:[email protected]

113

δ

δ

δm

i = i i = i’ i = i’ i

Maximum deviation in prism: r1+r2 =A, r2 =A – r1= A-θc A

δ N2

N1

i1=90

B C

r1 r2

δ = i1+i2-(r1+r2) (1)

δ = i1+i2- A

δ = δmax where i1= 900 (Grazing incidence)

δmax = 90+i2-A (2)

However when i1=90, Snell’s law (At AB)

1 sin 90 = μ sin r1

1/ μ = sin r1

r1 = sin-1(1/μ) which is critical angle

r1= θc = sin-1 /μ

so at surface AC μ sin r2 = 1. Sin i2

Sin i2 = μ sin (A-θc)

i2= sin-1[μ sin (A-θc)]

This is the value of angle of emergence in maximum deviation condition.

Minimum deviation in prism: 1 sin i= μ sin r

i= sin-1 μ sin r

i = sin-1 μ sin (A/2)

In this situation angle of emergence or incidence will be obtained by applying formula.

i = sin-1 μ sin (A/2)

Page 114: Geometrical Optics Book With Special Lecture

ANUJ PHYSICAL SCIENCES PHYSICS FOR IIT-JEE /AIEEE/AIPMT

Office: 128/652 “Y” Block Kidwai Nagar Kanpur -11 UP, Website: www.apskanpur.in Mob: 09795647531, 09889787531, 0848664308 E-mail:[email protected]

114

Condition of no emergence: (TIR) in prism: A

P

i1 R

B C

r1 r2

Q

The light will not emerge out of a prism for all values of angle of incidence if at face AB

for i1= max =90,at face AC r2 > θc (1)

now Snell’s law at face AB

1sin 90 = μ sin r1

sin r1= (1/μ)

r1= sin-1(1/μ) = θc (2)

r2 > θc

r1+ r2 > r2 + θc

r1+ r2 > θc + θc

r1+ r2 >2θc

A > 2θc (3)

A/2 > θc

sin A/2 > sin θc

sin A/2 > 1/μ ⇒

μ > 1/ [sin (A/2)]⇒

μ > cosec (A/2)

A/2 = sin-1(1/μ)

A > 2 sin-1(1/μ) (4)

A ray of light will not emerge out of a prism whatever be the angle of incidence if

A>2θc i.e. if μ> cosec A/2.

Page 115: Geometrical Optics Book With Special Lecture

ANUJ PHYSICAL SCIENCES PHYSICS FOR IIT-JEE /AIEEE/AIPMT

Office: 128/652 “Y” Block Kidwai Nagar Kanpur -11 UP, Website: www.apskanpur.in Mob: 09795647531, 09889787531, 0848664308 E-mail:[email protected]

115

Condition of grazing Emergence:

If a ray can emerge out of a prism, the value of angle ii for which angle of emergence i2 =

90 is called condition of grazing emergences out of face AC i.e. TIR does not take place at

it

A

Ii P

r i2 = 90

B C

r2

r2 < θcritical (1)

but as in prism r1+r2 =A

r1= A-(∠θc)

i.e. r1 > A-θc (2)

Now from Snell’s law at face AB we have

i sin i1 = μ sin r1

which in light of equation (2) gives

sin i1 > μ sin(A-θc)

i.e. sin i1 > μ[sin A cos θc – cos A sin θc]

sin i1 > μ[sin A √1-sin2 θc – cos A sin θc]

sin i1 > √μ2-1 sin A – cos A

as sin θc = 1/μ

i1 > sin-1[√μ2-1 sin A – cos A]

(i1)min = sin-1[√μ2-1 sin A – cos A] (3)

i.e. light will emerge out of a prism only if angle of incidence is greater than (i1)min

given by equation (3) in this situation deviation will be given by

δ = (i1+90-A) with i1 given by equation (3)

Page 116: Geometrical Optics Book With Special Lecture

ANUJ PHYSICAL SCIENCES PHYSICS FOR IIT-JEE /AIEEE/AIPMT

Office: 128/652 “Y” Block Kidwai Nagar Kanpur -11 UP, Website: www.apskanpur.in Mob: 09795647531, 09889787531, 0848664308 E-mail:[email protected]

116

Problem 1(IIT – JEE 2008): Two beams of red and violet colours are made to pass

separately through a prism (angle of the prism is 600). In the position of minimum

deviation, the angle of refraction will be

(a) 300 for both the colours (b) greater for the violet colour

(c) greater for the red colour (d) equal but not 300 for both the colours

Solution: At minimum deviation (δ=δm)

r1=r2 =A/2=600/2 =300 (For both colours)

Problem 2: A ray of light is incident at an angle of 600 on one face of a prism which

has an angle of 300. The ray emerging out of the prism makes an angle of 300 with the

incident ray. Show that the emergent ray is perpendicular to the face through which it

emerges and calculate the refractive index of material of the prism.

(a) μ = √2 (b) μ = √3 (c) μ = √4/3 (d) none

Solution:

A 300

300

600 i2 =0

B C

r1 r2 =0

According to given problem A = 300, i1 = 600, δ = 300

In a prism δ = (i1 + i2) – (r1 + r2)

δ = (i1 + i2) – A

30 = 60 + i2 -30

i2 =0

So the emergent ray is ⏊ to the face from which it emerges

Now as i2 =0, r2 =0 but r1 + r2 = A

r1 +0 = 30

r1 = 30

So at first face 1 sin 60 = μ sin 30

√3/2 = μ(1/2)

μ = √3 Answer

Page 117: Geometrical Optics Book With Special Lecture

ANUJ PHYSICAL SCIENCES PHYSICS FOR IIT-JEE /AIEEE/AIPMT

Office: 128/652 “Y” Block Kidwai Nagar Kanpur -11 UP, Website: www.apskanpur.in Mob: 09795647531, 09889787531, 0848664308 E-mail:[email protected]

117

Problem 3 (IIT – JEE 2004):

Q R

P S

A ray of light is incident on an equilateral glass prism placed on a horizontal table. For

minimum deviation which of the following is true?

(a) PQ is horizontal (b) QR is horizontal

(c) RS is horizontal (d) Either PQ or RS is horizontal

Solution: during minimum deviation the ray inside the prism is parallel to the base of

the prism in case of an equilateral prism.

Problem 4: A ray of light falls normally on a refracting face of a prism of refractive

index 1.5. Find the angle of the prism if the ray just fails to emerge from the prism

(a) sin-1 (2/3) = 420 (b) sin-1 (3/2) (c) cos-1 (2/3) (d) none

Solution:

A

i1 =0

r 900

δ

B C

r1 =0 r2

At first face of the prism as i1=0, sin 0 =0

Snell’s law 1 sin i1 = 1.5 sin r1 i.e. r1 =0

and as for a prism r1 + r2 = A

0 + r2 = A so r2 = A (1)

But as second face as the ray just fails to emerge i.e. r2 = θc (2)

A = r2 = θc

θc = sin-1(1/μ) critical angle

θc = sin-1(1/1.5) = sin-1 (2/3) = 420

A = 420

Note: Here δ = i1 + i2 –A = 0 + 90 -42 =480

δ = 480

Page 118: Geometrical Optics Book With Special Lecture

ANUJ PHYSICAL SCIENCES PHYSICS FOR IIT-JEE /AIEEE/AIPMT

Office: 128/652 “Y” Block Kidwai Nagar Kanpur -11 UP, Website: www.apskanpur.in Mob: 09795647531, 09889787531, 0848664308 E-mail:[email protected]

118

Problem 5:

A

300

R.I. = √2

B C

A light ray is incident on face AB of a right angled prism as shown in fig. The refractive

index of prism is √2. Now, the second face AC is rotated to increase the angle of prism.

Plot deviation versus angle of prism graph.

Solution:

A θ

r

β

B C

θ

Critical angle (C) = sin-1(1/μ) = sin-1(1/√2) = 450

Now, at any instant, angle of incidence is same as prism angle, initially θ is 300, as θ is

increased so the angle of incidence at second face will increase and ray is transmitted

out till θ < 450

Applying Snell’s law at interface AC, we get

√2 sin θ = 1. Sin r or r = sin-1 (√2 sin θ)

Deviation (β) = r – θ = sin-1 (√2 sin θ) – θ

This is a non-linear function Also θ → 450

β = sin-1 (√2 sin 450)-(π/4) = [(π/2)-( π/4)]= π/4

When prism angle is increased above 450, TIR will take place and deviation is given by

β = π -2θ , As θ → (π/2), β =0

The graph between deviation and angle of prism is as shown in fig.

Deviation β

π/2

π/4

O π/4 prism angle (θ)

Page 119: Geometrical Optics Book With Special Lecture

ANUJ PHYSICAL SCIENCES PHYSICS FOR IIT-JEE /AIEEE/AIPMT

Office: 128/652 “Y” Block Kidwai Nagar Kanpur -11 UP, Website: www.apskanpur.in Mob: 09795647531, 09889787531, 0848664308 E-mail:[email protected]

119

Problem 6: A ray of light undergoes deviation of 300 when incident on an equilateral

prism of refractive index √2. What is the angle subtended by the ray inside the prism

with the base of the prism.

(a) 300 (b) 600

(c) 900 (d) zero

Solution:

A

90 –r1 600

δ

i1 i2

600 600

B C

r1 r2

θ

Here δ = 300, A = 600

so if the prism had been in a minimum deviation

μ = sin {(A+δm)/2}/ sin (A/2)

μ = sin {(60+30)/2}/ sin (60/2)

μ = sin 450/ sin 300

μ = (1/√2)*2 = √2

Refractive index of prism is also given as √2,

the prism is in the position of minimum deviation.

This in turn implies that r1 = r2=r = A/2 = 60/2 = 300

So the angle subtended by the ray inside the prism with surface AB is equal to

90-r1 = 90-30 = 600

As base also subtends an angle of 600 with the face AB.

Hence the ray inside the prism is parallel to the base.

i.e. the angle subtended by the ray inside the prism with the base is zero.

Page 120: Geometrical Optics Book With Special Lecture

ANUJ PHYSICAL SCIENCES PHYSICS FOR IIT-JEE /AIEEE/AIPMT

Office: 128/652 “Y” Block Kidwai Nagar Kanpur -11 UP, Website: www.apskanpur.in Mob: 09795647531, 09889787531, 0848664308 E-mail:[email protected]

120

Problem 7: Find the co-ordinates of image of the point object ‘o’ formed after

reflection from concave mirror as shown in fig. Assuming prism to be thin and small in

size of prism angle 20 Refractive index of prism material is (3/2).

Y

f= 30cm

O (0, 0) x

5cm 20 cm

Solution:

I δ

d

O δ

5cm

Consider image formation through prism. All incident rays will be deviated by

δ = (μ-1)A = [(3/2)-1]20= 10 = (π/180) rad

Now as prism is thin so object and image will be in same plan as shown in fig.

It is clear d/5 = tan δ = δ (∵ δ is very small)

or d = (π/36) cm

Now this image will act as an object for concave mirror

u = -25 cm, f = -30 cm

v = uf/(u-f) = 150 cm

Also, m = -v/u = +6

∴ Distance of image from principal axis = (π/36)6 = π/6 cm

Hence, co-ordinates of image formed after reflection from concave mirror are

(175cm, π/6 cm)

Page 121: Geometrical Optics Book With Special Lecture

ANUJ PHYSICAL SCIENCES PHYSICS FOR IIT-JEE /AIEEE/AIPMT

Office: 128/652 “Y” Block Kidwai Nagar Kanpur -11 UP, Website: www.apskanpur.in Mob: 09795647531, 09889787531, 0848664308 E-mail:[email protected]

121

Problem 8: IIT-JEE Based on causes of dispersion (TIR): A beam of light

which contains radiations of wavelengths 5000 Ȧ and 4000 Ȧ is incident normally on a

prism as shown in figure. The refractive index of the prism as a function of wavelength

is given by μ (λ) = 1.20 + (b/√2)

where λ is in Ȧ and b is a +ive constant. The value of b is such that the condition for total

internal reflection at the face AC is just satisfied for one wavelength and not for other.

Find the value of constant b and deviation produced by the prism.

Solution:

A sin A = 0.8

i1 = 0 r1 =0

4000 Ȧ r2 = A δ 5000 Ȧ

5000 Ȧ

B C

According to given problem with increase in wavelength, μ decreases and as

θc = sin-1 (1/μ) longer wavelength will have shorter μ and greater critical angle.

Now as condition of TIR is satisfied only for one wavelength (and not for other),

the wavelength must be shorter one i.e. 4000 Ȧ

[As if the condition of TIR is satisfied for longer wavelength, i.e. r2 > (θc)longer, it will

automatically be satisfied for shorter wavelength also as (θc)shorter <(θc)longer and no light

will be transmitted] and hence μ = 1.20 + b/√2 (1)

Now as in prism r1 + r2 = A & as here i1 = r1 = 0, r2 = A (2)

but condition of TIR for λ = 4000 Ȧ at face AC required r2 = θc (3)

so from equation (2) & (3)

A = θc i.e. sin A = sin θc (4)

0.8 = sin θc = 1/μ , μ = /0 8 = 25

Substituting the value of μ from eq. (4) in eq. (1)

μ = 1.20 + (b/√2) = 1.20 + (b/ (4000)2)

i.e. b = 8*105 (Ȧ)2

μ = 1.20 +[8*105 / (500)2] = 1.20 + 0.032 = 1.232

And hence for light of wavelength 5000 Ȧ at face AC from Snell’s law

μ sin r2 = 1. Sin i2,

1.232 sin A = sin i2 (∵ r2 = A, μ= 1.232 )

sin i2 = 1.232*0.8 (∴ sin A = 0.8)

i2 = sin-1 (0.9856) = 80.2650

Now as in prism δ = i1 + i2 –A here i1 =0, A = sin-1(0.8) = 53.130

δ = 27 350

Page 122: Geometrical Optics Book With Special Lecture

ANUJ PHYSICAL SCIENCES PHYSICS FOR IIT-JEE /AIEEE/AIPMT

Office: 128/652 “Y” Block Kidwai Nagar Kanpur -11 UP, Website: www.apskanpur.in Mob: 09795647531, 09889787531, 0848664308 E-mail:[email protected]

122

Dispersion of light:

VIBGYOR

White light from narrow slit

Glass prism

* In Rig-Veda it is mentioned that light is made of many colours.

* In 1665 Sir Isaac Newton showed that natural light actually consists of seven colours.

All the colours of light mixed together appear white.

* The splitting of light (white) in to its constitutions is called dispersion of light

VIBGYOR The band of colours formed on a screen due to dispersion is called spectrum.

* Theoretically each wavelength is associated with its own colour; there are infinite

numbers of colours in natural light. Our eyes can differentiate only six colours indigo &

violet cannot be differentiable. So in further study we consider only six colours in the

spectrum of white light in wavelength range (4000 – 7000 Ȧ).

Causes of dispersion: μ = A+ (B/λ

2) Cauchy’s equation

Where μ= refractive index of any medium

λ = wavelength of light, μ depends on λ , A& B are constants. λR > λv hence μv >μR

hence δV > δR by δ = (μ-1) Hence deviation of violet colour is maximum is at the lower end &

red colour is the upper end of spectrum.

δR

δV

Page 123: Geometrical Optics Book With Special Lecture

ANUJ PHYSICAL SCIENCES PHYSICS FOR IIT-JEE /AIEEE/AIPMT

Office: 128/652 “Y” Block Kidwai Nagar Kanpur -11 UP, Website: www.apskanpur.in Mob: 09795647531, 09889787531, 0848664308 E-mail:[email protected]

123

Angular dispersion: θ= δv - δR

Dispersive power:

ω=θ/δy= angular dispersion/mean deviation ω = (δv - δR)/ δy = μv –μR / μy-1)

Combination of prisms: prism produces both

Deviation Dispersion

prism cannot give deviation without dispersion and vice versa when white light is

incident on it. So combination of prism (two) can do so...

Dispersion without deviation Deviation without dispersion

Dispersion without deviation:

A μ’

White light μ δy’

δy

A’

Consider two prisms of angles A& A’ ,

index μ &μ’ ,deviation by intermediate colours δy & δy’

δy = (μy-1)A , δy’= (μy’-1)A’

Total deviation produced by two prism’s is zero.

δy + δy’ = 0

(μy-1)A + (μy’-1)A’=0

A’= - [( μy-1) A) / μy’-1)] (1)

The negative sign indicates the two prisms must be placed with their angles oppositely.

But the combination will produce some dispersion.

By 1st prism dispersion produced δv - δR

By 2nd prism dispersion produced δv’ - δR’

Total dispersion produced D= δv - δR + δv’ - δR’

= (μv –μR) A+( μv‘–μR’)A’ (2)

D = (μv –μR) A+( μv’ –μR’)[- (μy-1)/( μy’-1)]A substituting (1)

D = μv –μR) A-[{ μv’ –μR’ / μy’- } μy-1)A] 3(a)

D= [{(μv –μR) / ( μy-1)} ( μy-1)A] – [{(μv’ –μR’)/ (μy’-1)} ( μy-1)A]

D= ωA( μy-1) – ω’A( μy-1)

D= (ω-ω’ A μy-1) 3(b)

Page 124: Geometrical Optics Book With Special Lecture

ANUJ PHYSICAL SCIENCES PHYSICS FOR IIT-JEE /AIEEE/AIPMT

Office: 128/652 “Y” Block Kidwai Nagar Kanpur -11 UP, Website: www.apskanpur.in Mob: 09795647531, 09889787531, 0848664308 E-mail:[email protected]

124

Deviation without dispersion: Total dispersion D= (δv - δR) + ( δv’ - δR’) =0

(μv –μR) A+( μv‘–μR’)A’ = 0

A’ = - [ μv –μR / μv’ –μR’ ] A’=0 (4)

A

A’

Negative sign indicates two prisms must be placed with their angle

oppositely. But deviation : δ = δy + δy’

δ= (μy-1)A+ (μy’-1)[-( μv –μR)/( μv’ –μR’)]A

δ= (μy-1)A[1- {( μy’-1)/ (μv’ –μR’)}*{( μv –μR)/( μy-1)}] 6(a)

δ= μy-1) A [1- ω/ω’ ] Two identical prisms of same material Placed in contact will

give light without deviation & dispersion.

Problem 9: A thin prism P1with angle 40 & made from glass of refractive index 1.54 is

combined with another prism P2 made from glass of refractive index 1.72 to produce

dispersion without deviation. What is the angle of prism P2.

(a) +30 (b) + 40 (c) 50 (d) none

Solution: given A1= 40, μ1 = 1.54, μ2 = 1.72

the angle of second prism for no total deviation:

A2 = -[(μ1-1)/(μ2-1)]A1 = [(1.54-1)/(1.72-1)]*4= -3*

Hence, the angle of the second prism should be 30 & it should be placed opposite to the

first.

Problem 10: White light is passed through a prism of angle 50. If the angle refractive

indices for red and blue colours are 1.641 & 1.659 respectively .Find

(1) The angle of dispersion between them (2) dispersive power of prism

Solution: (1) As for small angle of prism δ = (μ-1) A

δv = (1.659-1) *50 = 3.2950 δR = (1.641-1) *50 = 3.2050

θ = δv – δR = 3.295 – 3.205 = 0.0900

(2) As μav = (μv – μR)/2 = (1.659 + 1.641)/2 = 1.650

so dispersive power of the prism

ω = (μv – μR)/ (μ-1) = (1.659 - 1.641)/(1.650-1) = 0.081/(1.650 - 1)

ω = 0.018/0.650

= 0.0277

Page 125: Geometrical Optics Book With Special Lecture

ANUJ PHYSICAL SCIENCES PHYSICS FOR IIT-JEE /AIEEE/AIPMT

Office: 128/652 “Y” Block Kidwai Nagar Kanpur -11 UP, Website: www.apskanpur.in Mob: 09795647531, 09889787531, 0848664308 E-mail:[email protected]

125

Problem 11 (IIT 2001): The refractive indices of crown glass for blue and red

lights are 1.51 and 1.49 respectively and those of the flint glass are 1.51 and 1.49

respectively. An isosceles prism of angle 60 is made of crown glass. A beam of white light

is incident at a small angle on this prism. The other flint glass isosceles prism is combine

with the crown glass prism such that there is no deviation of the incident light.

Determine the angle of the flint glass prism. Calculate the net dispersion of the

combined system.

Solution:

A

A’

For combination of prisms

net deviation = δ1 + δ2 = (ny -1)A +(n’y-1)A

where δ1 and δ2 are deviations produced by individual prisms.

Net dispersion = (nv - nR) A + (n’v –n’R) A

According to problem net deviation is zero

hence (ny -1)A = -(n’y-1)A’

A’ = - [(ny - A / n’y-1)]

Negative sign implies that second prism is inverted relative to first

ny = (nv + nR)/2, n’y = (n’v –n’R)/2

A’=-[(ny -1)A /(n’y-1)]

ny = (1.51 + 1.49)/2

n’y = (1.77 + 1.73)/2 , A = 60 on substituting numerical values of ny, n’y, A

we get A’ = -40

Net dispersion = (nv - nR)A – (n’v - n’R)A’ = 0.040 ans.

Page 126: Geometrical Optics Book With Special Lecture

ANUJ PHYSICAL SCIENCES PHYSICS FOR IIT-JEE /AIEEE/AIPMT

Office: 128/652 “Y” Block Kidwai Nagar Kanpur -11 UP, Website: www.apskanpur.in Mob: 09795647531, 09889787531, 0848664308 E-mail:[email protected]

126

Lecture-8

Geometrical Optics

Physics for IIT - JEE

Defects of images (Aberration)

Chromatic Abberation:

Achromatism The achromatic doublet

Achromatism by separated doublet

Monochromatic Abberation:

Spherical

Coma

Astigmatism

Curvature

Distortion

Page 127: Geometrical Optics Book With Special Lecture

ANUJ PHYSICAL SCIENCES PHYSICS FOR IIT-JEE /AIEEE/AIPMT

Office: 128/652 “Y” Block Kidwai Nagar Kanpur -11 UP, Website: www.apskanpur.in Mob: 09795647531, 09889787531, 0848664308 E-mail:[email protected]

127

Defects of images (Aberration):

The equation & relations derived for lenses hold for paraxial light rays or for the

rays making small angles with the optic axis.

In practise however lenses are used to form images of points. Which are off the

axis also if light coming from an object is not monochromatic; a number of

overlapped coloured images are formed by the lens.

Thus in actual practise the image of a point object is not sharp & white. This

defect of lens is called aberration.

The coloured object formed by a lens of white light is called chromatic

aberration.

Chromatic Aberration:

μ= A+ B/λ2)

λ is the different for different colours of white light.

μ is different for different colours of white light.

Hence the focal length of a lens is different for different colours. It is longest for

red & shortest for violet colour is called axial or longitudinal chromatic

aberration.

Fv FR

FR – Fv

δf = fR - fv

δf = longitudinal or axial chromatic aberration for thin lens, the expression for

chromatic aberration can be easily derived.

1st Approach: The focal length of a thin lens is

1/f = (μ-1)[(1/R1)-(1/R2)] (1)

-df/f2= δμ[(1/R1)-(1/R2)] (2)

dividing equation (2) & (1) -

(df/f2) / 1/f = δμ/(μ-1) If a small change in μ gives a small change in f.

- (df/f2) f = δμ/(μ-1)

[df/f] = - δμ/ μ- = ω

[df] = - δμ/ (μ-1) f

This represents the axial chromatic aberration of lens.

If μv & μR represents the refractive indices of the red & violet colours

Page 128: Geometrical Optics Book With Special Lecture

ANUJ PHYSICAL SCIENCES PHYSICS FOR IIT-JEE /AIEEE/AIPMT

Office: 128/652 “Y” Block Kidwai Nagar Kanpur -11 UP, Website: www.apskanpur.in Mob: 09795647531, 09889787531, 0848664308 E-mail:[email protected]

128

fR - fv = -[(μR-μv)/(μ-1)]fy

fR - fv = [(μv-μR)/(μy-1)]fy

[(μv-μR)/(μy-1)] = dispersive power of a lens material

fR - fv= ω fy Expression for chromatic aberration

2nd Approach:

1/fR = (μR-1)[(1/R1)-(1/R2)]

1/fv = (μv-1)[(1/R1)-(1/R2)]

1/fy = (μy-1)[(1/R1)-(1/R2)]

(1/fR)-( 1/fv) = ( μR- μv) [(1/R1)-(1/R2)]

(fv- fR)/ fR fv= ( μR- μv) [(1/R1)-(1/R2)]

(fR- fv )/fR fv=[( μv- μR)/( μy-1)] [(1/R1)-(1/R2)]

(fR- fv )/ fy2 = [( μv- μR)/( μy-1)][1/ fy] (fR fv = fy2)

(fR - fv = ω fy

Achromatism: Minimization or removal of chromatic aberration is called Achromatism.

This can be possible by using two lenses of opposite nature.

The system of two lenses which is free from chromatic aberration is called

achromatic doublet.

The achromatic doublet: Consider two lenses of focal lengths f1&f2 & dispersive powers ω1 & ω2 are put in

contact.

f is the focal length of the combination.

Then 1/f=1/f1+1/f2

-df/f2 = - df1/f12 – df2/f22 for Achromatism df = 0

df1/f12 + df2/f22 =0

ω1 /f1 + ω2 /f2) =0 This is the required condition of Achromatism.

Case1st: If ω1= ω2

[1/f1+1/f2] ω=0

ω≠0 , Properties of the material of the prism.

[1/f1+1/f2]=0

1/f =0

f = ∞

Lens behaves like a plane mirror.

Hence both lenses have opposite material.

Page 129: Geometrical Optics Book With Special Lecture

ANUJ PHYSICAL SCIENCES PHYSICS FOR IIT-JEE /AIEEE/AIPMT

Office: 128/652 “Y” Block Kidwai Nagar Kanpur -11 UP, Website: www.apskanpur.in Mob: 09795647531, 09889787531, 0848664308 E-mail:[email protected]

129

Crown glass flint glass

Case 2nd: (ω1 /f1) = - (ω2 /f2)

ω1 &ω2 are the properties of the material of the prism. Hence it must be positive. This

indicates both the lenses must be of opposite nature. If one is concave then other is

convex.

Concave: Diverging Flint glass Convex: Converging Crown glass

Achromatism by separated doublet:

Problem 1: Consider two convex lenses of focal lengths f1&f2 are made from same

material, are separated by a suitable distance d. The focal length of the combination is

given by 1/f = 1/f1+1/f2-d/f1f2, the value of d for which combination is free from

chromatic aberration is given by

a) (f1+f2)/2 b) (f1-f2)/2 c) (f1+f2) d) none

Solution: Consider two convex lenses of focal lengths f1&f2 separated by a suitable

distance d. The focal length of the combination is given by: 1/f1+1/f2-d/f1f2 = 1/f (1)

differentiate above equation

-df1/f12 –df2/f22 –d[1/f1(-df2/f22)+1/ f2(df1/f12)] = (-1/f2 )df {df=0 for Achromatism}

(df1/f1)1/f1+(df2/f2)1/f2 –d[(df2/f2)1/f1f2 +(df1/f1)1/f1f2]=0

(ω1/f1) + ( ω2/f2) – d[(ω2/f1f2) + (ω1/f1f2)] =0

(ω1f2+ ω2f1)/ f1f2 = d(ω2+ω1)/ f1f2

which gives d = (ω1f1+ ω2f2)/ ω1+ω2)

In case when lenses are of same material, ω1 = ω2 = ω Then d = (f1+f2)/2

Thus two lenses of same nature can be free from chromatic aberration if they are placed

at a separation d = (f1+f2)/2

Page 130: Geometrical Optics Book With Special Lecture

ANUJ PHYSICAL SCIENCES PHYSICS FOR IIT-JEE /AIEEE/AIPMT

Office: 128/652 “Y” Block Kidwai Nagar Kanpur -11 UP, Website: www.apskanpur.in Mob: 09795647531, 09889787531, 0848664308 E-mail:[email protected]

130

Problem 2: Achromatism of Hygen’s Eyepiece: Hygen’s Eyepiece consist of two Plano convex lenses L1&L2 of same material having focal length +3f & +f separated by a

distance d from each other. Their convex side face the incident light. Hygen’s’ Eyepiece

is achromatic when lenses are placed coaxially at distance

a) d=f /2 b) d=f c) d=2f d ) d=3f

Solution: As we know that in case of Achromatism by separated doublet separation

distances between two lenses is given by : d = (ω1f1+ ω2f2)/ ω1+ω2)

In case when lenses are of same material, ω1 = ω2 = ω Then d = (f1+f2)/2

in this problem f1=3f & f2=f

d = (f1+f2)/2=3f+f/2=2f

Hygen’s Eyepiece satisfies the condition of Achromatism and has the same focal length

for all colours. This makes Hygen’s Eyepiece apparently free from lateral chromatic

aberration.

Problem 3: The focal lengths of two convex lenses of same material are 20 cm & 30

cm. What should be the separation between them so that they form an achromatic

combination?

a) 25 cm b) 30 cm c) 40 cm d) None

Solution: d = (f1+f2)/2 = (20+30)/2 = 25 cm

Problem 4: The dispersive power of crown and flint glasses are 0.02&0.04

respectively. Find the focal length of the two components of an achromatic doublet of

focal length 20 cm.

Solution: An achromatic doublet (telescope objective) consists of two lenses in

contact, a convex lens of crown glass and a concave lens of flint glass. If f1 & f2 be the

focal length and ω1 & ω 2 the dispersive powers of the convex and concave components,

then the condition of Achromatism is

ω1 /f1 + ω2 /f2) =0

here ω1=0.02 & ω2=0.04

(0.02/f1) + (0.04/f2) =0

f2=-2f1

Focal length of combination of two thin lenses in contact 1/f1 + 1/f2 =1/F

here F=20 cm & f2=-2f1

1/f1 + 1/(-2f1 )=1/20

1/f1 - 1/2f1 =1/20

1/2f1 =1/20

f1 =10 cm

f2=-2f1=- 20 cm

Page 131: Geometrical Optics Book With Special Lecture

ANUJ PHYSICAL SCIENCES PHYSICS FOR IIT-JEE /AIEEE/AIPMT

Office: 128/652 “Y” Block Kidwai Nagar Kanpur -11 UP, Website: www.apskanpur.in Mob: 09795647531, 09889787531, 0848664308 E-mail:[email protected]

131

Problem 5: A convex lens made of material ‘A’ is combined with a concave lens made

of material ‘B’ so as to form an achromatic doublet. If an object of height 6 cm is placed

30 cm in front of the doublet, it forms an erect image of size 2 cm. Find the focal lengths

of the component lenses, given that the ratio of dispersive powers of materials A & B is

2:1.

(a) 15, -7.5 cm (b) -7.5, 15cm (c) 1.5, 15cm (d) none

Solution: The erect & small size of the image shows that doublet should be of

diverging nature. For erect image

m= v/u = I/O {m=-v/u}

m = (v)/(u) = I/O = 2/6

v/u =1/3 , v = u/3

u= -30cm, v = -10cm

By the lens formula 1/v -1/u = 1/f

[1/(-10)] – [1/(-30)] =1/f fcombination = -15 cm

1/fA + 1/fB =1/f

1/fA + 1/fB = -1/15

1/fA[1+(fA/fB)] = -1/15

From condition of Achromatism (ωA /fA) + (ωB /fB)=0 ,

ωA/ ωB=-- fA/ fB= 2 ( given in problem)

1/fA[1+(-2)] =-1/15

-1/fA = -1/15 gives fA=15

fA/fB = -2 , 15/fB = -2 gives fB = -7.5 cm

Hence option (a) 15; -7.5 cm is only correct.

Problem 6: An equviconvex lens of crown glass and an equviconcave lens of flint

glass make an achromatic system. The radius of curvature of convex lens is 0.54m. If the

focal length of the combination for mean colour is 1.50mand the refractive indices for

the crown glass are μR= 1.53 & μv= 1.55. Find the dispersive power of flint glass

(a) 1/18 = .055 (b) 2/18 (c) 3/18 (d) none

Solution: As for a lens 1/f =(μ-1) [(1/R1)-(1/R2)]

R1=R2 =R = 54cm μy = (μv + μR)/2 = 1.54 (given)

for convex lens 1/fc =(1.54-1) [(1/54)-(1/-54)

1/fc =(1.54-1) [(1/54)+(1/54)] gives fCrown = 50 cm (1)

now as combination of two thin lenses in contact 1/fc + 1/fF =1/f

1/50+1/fF=1/150 gives flint = -75cm (2)

As the system is achromatic ωf /fF + ωc/fc =0

ωc = (μv - μR) / μy-1 = 1.55-1.53/1.54-1 = 1/27 (3)

Substituting the values of fc , ff & ωc in ωf /fF + ωc/fc =0

ωf = - ωc (ff / fc) =-(1/27)(-75/50)=1/18=0.055

Page 132: Geometrical Optics Book With Special Lecture

ANUJ PHYSICAL SCIENCES PHYSICS FOR IIT-JEE /AIEEE/AIPMT

Office: 128/652 “Y” Block Kidwai Nagar Kanpur -11 UP, Website: www.apskanpur.in Mob: 09795647531, 09889787531, 0848664308 E-mail:[email protected]

132

Problem 7: Two lenses, one made of crown glass and the other of flint glass, are to be

combined so that the combination is achromatic for the blue & red light and acts as a

convex lens of focal length 35 cm.

Calculate the focal length of the components of it.

For crown glass μy = 1.5175 μβ – μR = 0.00856

flint glass μy = 1.6214 μβ – μR = 0.01722

Solution: According to given problem (1/fc) + (1/ff) = 1/35 (1)

the system will be achromatic if (ωc /fc) + (ωf/ff) =0 i.e. (fc/ff) = - (ωc /ωf) (2)

Now as by definition ω = dμ/μ-1 = (μβ – μR)/ (μy-1)

ωc = 0.00856/(1.5175-1) = 0.01654

ωf = 0.01722/(1.6214-1) = 0.02771

substituting these values of ωc & ωf in eq. (2)

fc / ff = -[0.01654/0.02771] fc = -[1654/2771] ff (3)

substituting fc from eq.(3) in eq. (1)

1/fc +1/ff =1/35

- [2771/1654ff]+1/1/ff = 1/35

ff = [-35*117]/1654 = -23.8cm

fc = -[1654/2771] fc = (-1654*-23.8)/2771 = 14.17 cm

fflint = -23.8 & fcrown = 14.17 cm

Problem 8: (IIT-JEE ) State whether the following statement is true or false giving reason in brief “ A parallel

light beam of white light is incident on a combination of a concave & convex lens both of

same material. Their focal lengths are 15cm and 30cm respectively for the mean

wavelength in white light on the other side of the lens system. One sees coloured

patterns with violet colours at the outer edge’’.

Solution:

The combination will behave as a single lens of focal length 1/f = 1/ (-15) +1/30

f = - 30 cm , As divergent lens of focal length 30cm.

Since for a lens fv is lesser than fR violet deviates more than red so on the other side we

will see coloured pattern with violet on the outer side i.e. the given statement is true. V

δv R

δR

FR Fv

Fv

V

Page 133: Geometrical Optics Book With Special Lecture

ANUJ PHYSICAL SCIENCES PHYSICS FOR IIT-JEE /AIEEE/AIPMT

Office: 128/652 “Y” Block Kidwai Nagar Kanpur -11 UP, Website: www.apskanpur.in Mob: 09795647531, 09889787531, 0848664308 E-mail:[email protected]

133

Monochromatic Aberration:

The size of the image as formed by a lens is not according to theoretical calculation even

using monochromatic light the image formed will spread both along & perpendicular to

the principle axis of the lens. Also the shape of the image is not according to the shape of

object.

Monochromatic Aberration can be divided:

1.Spherical 2.Coma 3.Astigmatism 4.Curvature 5.Distortion

1. Spherical Aberration:

Figure shows the image formed by different parts of a lens of a point object.

Paraxial rays of light form the image at longer distance from the lens than the marginal

rays. The image is not separate any point on the axis the effect is called spherical

aberration.

Marginal rays

Paraxial rays

Fm FP

Paraxial rays

Marginal rays

Minimization or removal of Spherical Aberration: Spherical aberration can be minimized by

Page 134: Geometrical Optics Book With Special Lecture

ANUJ PHYSICAL SCIENCES PHYSICS FOR IIT-JEE /AIEEE/AIPMT

Office: 128/652 “Y” Block Kidwai Nagar Kanpur -11 UP, Website: www.apskanpur.in Mob: 09795647531, 09889787531, 0848664308 E-mail:[email protected]

134

(1) Using stops or crossed lens.

(2) Using lens of large focal lengths

(3) Using Plano convex lens

(4) Using two thin lenses separated by a distance using crossed lens

(5) Using aplanatic lens

(6) Using parabolic reflectors

2. Coma: When object is situated off the axis, its image will spread obliquely

perpendicular to the principle axis. It looks like comet & so called coma.

Removal of Coma: (i) Coma may be reduced by placing a stop at a suitable distance from the lens.

(ii) Coma may be minimized by designing lenses of suitable shapes and materials.

For example, for an object at infinite distance, a lens with µ=1.5 and R1/R2= -1/9 forms

an image sufficiently free from coma.

3. Astigmatism: When object is situated off the axis, the spread of image along the

principle axis of the lens is known as astigmatism.

The object situated off the axis its image will spread along & perpendicular to the

principle axis.

Removal of Astigmatism: (i) For a single lens the astigmatic difference may be reduced by placing a stop in a

suitable position so that only less oblique rays are permitted to form the image.

(ii)For a system of several lenses astigmatism may be eliminated by adjusting their

relative positions. Such systems are widely used as photographic objectives on which

narrow pencils are incident at large angles.

4. Curvature: The image of an object (extended plane object) formed by lens is not a

flat but curved. This defect is called the curvature. This effect is even present it the

aperture of the lens is reduced by a suitable stop.

Removal of Curvature: (i) for a single lens, the curvature may be reduced by placing an aperture in a suitable

position in front of the lens.

(ii) For a combination of lenses, the condition for absence of curvature is sum of 1/µf

=0, where µ is refractive index & f is the focal length of a lens. For two lenses (whether

in contact or separated by a distance) the condition reduces to

1/µ1f1+1/µ2f2 =0 or µ1f1 + µ2f2=0 (Petzval condition)

Page 135: Geometrical Optics Book With Special Lecture

ANUJ PHYSICAL SCIENCES PHYSICS FOR IIT-JEE /AIEEE/AIPMT

Office: 128/652 “Y” Block Kidwai Nagar Kanpur -11 UP, Website: www.apskanpur.in Mob: 09795647531, 09889787531, 0848664308 E-mail:[email protected]

135

Since µ1, µ2 are positive, f1 & f2 should be of opposite signs. Hence by combining a

convex lens of a certain material with a concave lens of the suitable material and focal

length, a flat object is obtained.

5. Distortion: When a stop is used is used with a lens to reduce the various

aberrations, the image of a plane square like objects placed perpendicular to the axis is

not of the same shape as the object. This defect is called distortion.

Removal of Distortion: A combination of two similar meniscus convex lenses with their concave surfaces facing

each other and having an aperture stop in the middle is free from distortion, when the

object & image are symmetrically placed.

(a) (b) (c) (d)

Object Image like comet image Spread of the image Curved Image

situated spread perpendicular to along the principle axis

off axis principle axis

Problem 9(AIEEE 2011): When monochromatic red light is used instead of blue

light in a convex lens, its focal length will

(a) increase (b) decrease (c) remain same (a) does not depend on colours of light

Solution: 1/f =(μ-1)[(1/R1)-(1/R2)]

also, by Cauchy’s formula μ=A+(B/λ2)+(c/λ4)+.......

λblue < λred

μblue > μred

hence, fred > fblue

Page 136: Geometrical Optics Book With Special Lecture

ANUJ PHYSICAL SCIENCES PHYSICS FOR IIT-JEE /AIEEE/AIPMT

Office: 128/652 “Y” Block Kidwai Nagar Kanpur -11 UP, Website: www.apskanpur.in Mob: 09795647531, 09889787531, 0848664308 E-mail:[email protected]

136

Condition for Minimum Spherical Aberration of two lenses separated

by a distance d:

Problem 10: The condition for minimum spherical aberration for two thin lenses of

focal length f1&f2 separated by distance d is given by

a) d= f1+f2 b) d= f1-f2 c) d= f1-f2/2 d) None

Solution: When a beam of light passes through a system of two thin lenses placed

coaxially at a distance apart from each other, the refraction takes place at four surfaces.

The spherical aberration is Minimum when there is an equal deviation at all a surface.

d

A B C

h1 δ1 h2 δ2

L1 L2 F F’

f1

Let L1&L2 be two coaxial converging lenses of focal lengths f1&f2 and separated by a

distance d. A ray parallel to the axis meets L1 at a height h1and suffers deviation

δ1=h1/f1 and is directed towards F’, the second focal point of L1. The reflected ray BC

meets the lens L2 at height h2 and suffers deviation δ2=h2/f2. The final emergent ray

meets the axis at F which is the second focal length of the combination.

For minimum spherical aberration, the total deviation should be equally shared by two

lenses, that is

δ1= δ2

h1/f1= h2/f2 , h1/ h2= f1/f2 (1)

Now from similar triangles BL1F’ & CL2F’, we have

BL1/CL2=L1F’/L2F’=L1F’/ (L1F’-L2F’)

Here BL1=+h1, CL2=+h2, L1F’=+ f1 & L1L2=d

h1/ h2=f1 /(f1-d) (2)

comparing equation (1) &(2) we get

f1/f2 = f1 /(f1-d), f2 =(f1-d)

d =(f1- f2) This is required condition. (3)

Page 137: Geometrical Optics Book With Special Lecture

ANUJ PHYSICAL SCIENCES PHYSICS FOR IIT-JEE /AIEEE/AIPMT

Office: 128/652 “Y” Block Kidwai Nagar Kanpur -11 UP, Website: www.apskanpur.in Mob: 09795647531, 09889787531, 0848664308 E-mail:[email protected]

137

So the two lenses should be placed with light incident on the lens of grater focal

length and the distance between them should be equal to the difference between

their focal lengths.

Note: An optical device satisfying Condition for Minimum Spherical Aberration of two

lenses separated by a distance d: The condition of minimum spherical aberration is satisfied in Hygen’s eyepiece, which

consists of two Plano convex lenses of focal lengths3f&f respectively and separated by a

distance 2f.

Problem11: A convergent doublet of separated lenses, corrected for spherical

aberration, has an equivalent focal length of 10 cm. the lenses of doublet are separated

by 2 cm.

What are the focal lengths of its component lenses?

a) 20 cm , 18 cm

b) 20 cm , 15 cm

c) 15 cm , 18 cm

d) None

Solution: Let f1&f2 be the focal length of two components.

Since the doublet is corrected for spherical aberration, it satisfies the following

condition;

d =(f1- f2)=2cm

f1-= f2 +2 cm (1)

The equivalent focal length 1/F = 1/f1 + 1/f2 – d/(f1f2)

F= (f1f2) /( f1+ f2-d)=10 cm (2)

Substituting the value of f1 from equation (1) in equation (2) we get

F= (f2 + 2) f2/( f2+2+ f2-2)=10 cm

(f2 + 2) f2/( 2f2)=10

(f2 + 2) /2=10

(f2 + 2) =20

f2 =18 cm (3)

substituting value of f1 in equation (1) f1-= f2 +2 cm

f1= 18+2=20 cm

Page 138: Geometrical Optics Book With Special Lecture

ANUJ PHYSICAL SCIENCES PHYSICS FOR IIT-JEE /AIEEE/AIPMT

Office: 128/652 “Y” Block Kidwai Nagar Kanpur -11 UP, Website: www.apskanpur.in Mob: 09795647531, 09889787531, 0848664308 E-mail:[email protected]

138

Problem 12: From the condition for no chromatic aberration and minimum spherical

aberration of a combination of two separated thin lenses, design a combination of

equivalent focal length 5.0 cm. To design the required condition,

a) Two Plano convex lenses of focal lengths 10 cm & 3.3 cm must be kept separated by a

distance of 6.7 cm

b) Two Plano concave lenses of focal lengths 10 cm & 3.3 cm must be kept separated by

a distance of 6.7 cm

c) A Plano convex lens of focal lengths 10 cm & A Plano concave lens of focal length 5cm

must be kept separated by a distance of 6.7 cm

d) A Plano concave lens of focal lengths 10 cm & A Plano convex lens of focal length 5cm

must be kept separated by a distance of 6.7 cm

Solution: Let f1&f2 be the focal length of two lenses of the same material and d the

separation between them.

From the condition of the Achromatism, we have d= (f1+f2)/2 (1)

From the condition of minimum spherical aberration, we have d=(f1-f2) (2)

Solving equation (1) & (2),

d= (f1+f2)/2 =(f1-f2)

(f1+f2) =2(f1-f2)

f1+f2 =2f1-2f2

3f2 =f1 (3)

Putting the value from equation (3)in equation (2)

d=(f1-f2)

d=3f2 –f2=2f2 (4)

The equivalent focal length of combination is given by 1/F = 1/f1 + 1/f2 – d/(f1f2)

F= (f1f2) /( f1+ f2-d) (5)

Here F=+5.0 cm (the combination is converging), f1= 3f2, d=2f2

substituting these values in equation (5) F= (f1f2) /( f1+ f2-d)

5= (3f2 f2) /( 3f2+ f2-2f2)

5= (3f2 f2) /( 2f2)

5= 3f2/2

f2=10/3= 3.33 cm (6)

substituting the value of f2 in equation (3) & (4), we get

f1=3f2 =10 cm

d =2f2 +6.7 cm

Hence to design the required combination, two Plano convex lenses of focal lengths 10

cm & 3.3 cm must be kept separated by a distance of 6.7 cm.

Page 139: Geometrical Optics Book With Special Lecture

ANUJ PHYSICAL SCIENCES PHYSICS FOR IIT-JEE /AIEEE/AIPMT

Office: 128/652 “Y” Block Kidwai Nagar Kanpur -11 UP, Website: www.apskanpur.in Mob: 09795647531, 09889787531, 0848664308 E-mail:[email protected]

139

Lecture-9

Geometrical Optics

Physics for IIT - JEE

Refraction from curved surfaces:

Refraction from single curved surface

Lens makers formula

Lense formula

Displacement methods

Page 140: Geometrical Optics Book With Special Lecture

ANUJ PHYSICAL SCIENCES PHYSICS FOR IIT-JEE /AIEEE/AIPMT

Office: 128/652 “Y” Block Kidwai Nagar Kanpur -11 UP, Website: www.apskanpur.in Mob: 09795647531, 09889787531, 0848664308 E-mail:[email protected]

140

Refraction from curved surfaces: If the boundary between two transparent media is

not plane and an object O in medium of refractive index μ1 forms an image I in medium of

refractive index μ2 as shown in figure.

Applying Snell’s law at the boundary AB

A

μ1 i D μ2

α β δ

o P C I

u v

B

μ1 sin i = μ2 sin r

for small angles

μ1 tan i = μ2 tan r

μ1 i = μ2 r (1)

μ1 (α + β) = μ2 (β - δ)

μ1 α + μ1 β = μ2 β – μ2δ

μ1 α + μ2δ = (μ2 – μ1)β

μ1 tan α + μ2 tan δ = (μ2 – μ1) tan β

μ1 (y/-u) + μ2 (y/v) = (μ2 – μ1) (y/R)

(μ1 /-u) + (μ2 /v) = ((μ2 – μ1)/R)

μ2 /v) – μ1 /u = μ2 – μ1)/R) (2)

This is desired result. If we compare it with mirror formula (1/v) + (1/u) = (1/f)

u → (-u/ μ1), v → (v/ μ2), f → (R/ (μ2 – μ1))

so transverse magnification in this case

m = I/O = [(-v/μ2) /(-u/μ1)] = (v/u)( μ1 / μ2)

m = v/u μ1 / μ2) (3)

While using eq. (1) and (3) keep in mind that

(1) these are valid for all single refractive surfaces, concave, convex or plane.

In case of plane refractive surface f = ∞, R = ∞

(μ2 /v) – (μ1 /u) = 0 ⇒ u/v = μ1 / μ2

dAc /dAp = μ1 / μ2

(2) The rules for signs for single refracting surfaces are same as spherical mirrors.

(3) If object or image is itself present at refracting surface, refraction at that surface is

not considered.

Page 141: Geometrical Optics Book With Special Lecture

ANUJ PHYSICAL SCIENCES PHYSICS FOR IIT-JEE /AIEEE/AIPMT

Office: 128/652 “Y” Block Kidwai Nagar Kanpur -11 UP, Website: www.apskanpur.in Mob: 09795647531, 09889787531, 0848664308 E-mail:[email protected]

141

Problem 1(IIT.R -1997): If a mark of size 0.2 cm on the surface of a glass sphere of

diameter 10 cm and μ = 1.5 is viewed through diametrically opposite point, where will

the image be seen and of what size.

(a) 20 cm towards object from point of observation & size is 0.6 cm

I O P

μ2

10 cm

C

μ1 = 1.5

Solution: As mark is an surface, refraction will take place on the other surface only

(which is curved)

(μ2 /v) – (μ1 /u) = ((μ2 – μ1)/R)

(1/v) –(1.5/(-10)) = (1.5-1)/-5

1/v = (.5/-5) – (1.5/10)

1/v = (5/50) – (15/100)

1/v = (10/100) – (15/100)

1/v = (-5/100)

v = -20 cm

The image is at a distance of 20 cm from P towards O as shown in figure.

Now as in case of refraction through curved surface

m = I/O = (v/u) (μ1 / μ2) = (-20/-10) (15/1) = 3.0 cm

Size of image: m = I/O = 3.0 cm

I = 3.00 =3*0.2 = 0.6 cm

so image is erect, virtual & enlarged i.e. of size 0.6 cm

If refracting surface had been plane.

(μ2 /v) – (μ1 /u) = ((μ2 – μ1)/R)

(μ2 /v) – (μ1 /u) = ((μ2 – μ1)/∞) =0

(μ2/μ1) = (v/u)

(1/ 1.5) = (v/-10) ⇒ v = (-10/1.5) = - 100/15 = -20/3

V= -20/3 cm

m = (μ2/μ1) (v/u) = (1.5/1) [(-20/3)/-10] = (15/10)(20/3)(1/10) = 1

m = I/O =1⇒ I = 0 i.e. image is at a distance of 20/3 cm from P towards O as shown in

the figure.

Page 142: Geometrical Optics Book With Special Lecture

ANUJ PHYSICAL SCIENCES PHYSICS FOR IIT-JEE /AIEEE/AIPMT

Office: 128/652 “Y” Block Kidwai Nagar Kanpur -11 UP, Website: www.apskanpur.in Mob: 09795647531, 09889787531, 0848664308 E-mail:[email protected]

142

Problem 2(IIT 1999): A quarter cylinders of radius R and refractive index 1.5 is

placed on a table. A point object P is kept at a distance of mR from it. Find the value of m

for which a ray from P will emerge parallel to the table as shown in figure.

(a) m = 4/3 (b) m = 4 (c) m = 2 (d) m = 3/4

P G

mR R

Solution:

R

P1 P mR R

1.5 mR

From refraction at plane surface

(μ2 /v) – (μ1 /u) = ((μ2 – μ1)/R)

μ2 = 1.5, μ1 = 1, R1 = R = ∞ (plane), u = -mR

(1.5/v)- (1/-mR) = (1.5 -1)/∞

(1.5/v) = (1/-mR)

V = -1.5 mR So image will be formed at point P1

for curved surface image of P at P1 will act as an object, for this surface

(μ2 /v) – (μ1 /u) = ((μ2 – μ1)/R)

μ2 =1, μ1 = 1.5, u = -(1.5mR +mR) = -R[1.5m + 1], v = ∞

(1/∞)-(1.5/-(1.5m+1)R) = (1-1.5)/(-R)

[1.5/(1.5m+1)R] = [.5/+R]

1.5/(1.5m+1) = +.5/1 ⇒ 1.5/(1.5m+1) = +.5

3/(1.5m+1) = Δ ⇒ 3 = 1.5m +1 ⇒ 2 = 1.5m

m = 2/1.5 = 20/15

m= 4/3

Page 143: Geometrical Optics Book With Special Lecture

ANUJ PHYSICAL SCIENCES PHYSICS FOR IIT-JEE /AIEEE/AIPMT

Office: 128/652 “Y” Block Kidwai Nagar Kanpur -11 UP, Website: www.apskanpur.in Mob: 09795647531, 09889787531, 0848664308 E-mail:[email protected]

143

Problem 3 (IIT 1991): The slab of material of refractive index 2 shown in figure

has a curved surface APB of radius of curvature 10 cm and a plane surface ID on the left

of APB is air and on the right of CD is water with refractive indices as given in the figure.

An object O is placed at a distance of 15 cm from the pole P as shown. Which of the

following statements are correct

(a) Distance of final image of O from P as viewed from the left is 30 cm.

(b) Curved surface will form virtual image I at a distance of 30 cm from P

(c) There will be no refraction at the plane surface CD

(as the rays are not actually passing through the boundary)

(d) All statements are correct.

A C

μ =1 μ =2 μ = 4/3

P C I

15 cm right

Left

B 20 cm D

30 cm

Solution:

In case of refraction from a curved surface, we have

(μ2 /v) – (μ1 /u) = ((μ2 – μ1)/R)

μ1 = 2, μ2 =1, R = -10 cm, u = -15 cm

so (1/v) –(2/-15) = (1-2)/ -10

⇒ v = -30 cm

i.e. the curved surface will form virtual image I at a distance of 30 cm from P since the

image is virtual there will be no refraction at the plane surface CD (as the rays are not

actually passing through the boundary), the distance of final image I from P will remain

30 cm.

Page 144: Geometrical Optics Book With Special Lecture

ANUJ PHYSICAL SCIENCES PHYSICS FOR IIT-JEE /AIEEE/AIPMT

Office: 128/652 “Y” Block Kidwai Nagar Kanpur -11 UP, Website: www.apskanpur.in Mob: 09795647531, 09889787531, 0848664308 E-mail:[email protected]

144

Problem 4: An air bubble in glass (μ = 1.5) is situated at a distance 3 cm from a

spherical surface of diameter 10 cm as shown in figure (A,B). At what distance from the

surface will the bubble appear if the surface is (1) convex (2) concave (3) plane

(a) The bubble will appear at a distance 2.5 cm from the curved surface inside the glass.

(b) The bubble will appear at a distance 1.66 cm from the curved surface inside the

glass.

(c) The bubble will appear at a distance .2 cm from the curved surface inside the glass

(d) none

Solution:

μ = 1.5 μ = 1 μ = 1.5 μ =1

C O I P O I P C

3 cm 3 cm 5 cm

5 cm

(A) (B)

In case of refraction from curved surface (μ2 /v) – (μ1 /u) = ((μ2 – μ1)/R)

(a) In case convex surface, μ1 = 1.5, μ2 =1, u = -3cm, R = -5cm

(1/v) – (1.5/-3) = [(1-1.5)/-5] ⇒ v = -2.5 cm

i.e. the bubble will appear at a distance 2.5 cm from the curved surface inside the glass.

(b) In case concave surface, μ1 = 1.5, μ2 =1, u = -3cm, R = +5cm

(1/v) – (1.5/-3) = [(1-1.5)/5] ⇒ v = - 10/6 = -1.66 cm

i.e. the bubble will appear at a distance 1.66 cm from the curved surface inside the glass.

(c) In case plane surface, μ1 = 1.5, μ2 =1, u = -3cm, R = ∞

(1/v) – (1.5/-3) = [(1-1.5)/∞]=0

(1/v) = -[1.5/3] ⇒ v = .2cm

i.e. the bubble will appear at a distance .2 cm from the curved surface inside the glass.

Page 145: Geometrical Optics Book With Special Lecture

ANUJ PHYSICAL SCIENCES PHYSICS FOR IIT-JEE /AIEEE/AIPMT

Office: 128/652 “Y” Block Kidwai Nagar Kanpur -11 UP, Website: www.apskanpur.in Mob: 09795647531, 09889787531, 0848664308 E-mail:[email protected]

145

Problem 5: A parallel beam of light is incident normally on the flat surface of a

hemisphere of radius R = 6 cm and refractive index 1.56. Assuming paraxial ray

approximation,

(a) determine the point at which the beam is focussed as measured along the axis from

the curved surface;

(b) determine the new focal length measured from the flat surface if the rays are

incident at the curved surface

R

Solution: Rays pass without deviation at flat surfaces. From single surface refraction

equation for single surface refraction equation for surface S2, we have

(1/v) – (n/∞) = (1-n)/(-R)

v = R/(n-1) = 6/[1.56-1] = 6/0.56 = 10.7 cm

This focal point as rays travel from left.

From single surface refraction equation at S1, we have

(n/v) – (1/∞) = (n-1)/R

v = nR/(n-1)

f f I1

S1 S2 S1 S2

This is the first image, it acts as an object for refraction at the plane surface. Object

distance for refraction at S2, = [R/(n-1)]-R = R/(n-1)

so we have (1/v’)- [n/+R(n-1)] =0

v’ = R/n(n-1) = 6/1.56(1.56-1) = 6.9 cm

Hence focal point is at 6.9 cm from the plane surface.

Page 146: Geometrical Optics Book With Special Lecture

ANUJ PHYSICAL SCIENCES PHYSICS FOR IIT-JEE /AIEEE/AIPMT

Office: 128/652 “Y” Block Kidwai Nagar Kanpur -11 UP, Website: www.apskanpur.in Mob: 09795647531, 09889787531, 0848664308 E-mail:[email protected]

146

Problem 6: In fig. Light is incident on a thin lens as shown. The radius of curvature of

both surfaces is R. Determine the focal length of the system.

μ1 μ2 μ3

For refraction at first surface (μ2/v1)- (μ1/∞) = (μ2-μ1)/R (1)

for 2nd surface: u2 =v1, v2 = f

(μ3/f) –(μ2 /v1) =( μ3 –μ2)/R (2)

on adding (1) and (2)

(μ2 /v1) =( μ2 –μ1)/R

(μ3/f) –(μ2 /v1) =( μ3 –μ2)/R

(μ3/f) = [μ2 – μ1 + μ3 –μ2]/R

(μ3/f) =( μ3 –μ1)/R

f = μ3R / μ3 –μ1)

If we want nature as a convex then f > 0

[μ3R /( μ3 –μ1)] > 0

μ3 > μ3 –μ1)/R

Page 147: Geometrical Optics Book With Special Lecture

ANUJ PHYSICAL SCIENCES PHYSICS FOR IIT-JEE /AIEEE/AIPMT

Office: 128/652 “Y” Block Kidwai Nagar Kanpur -11 UP, Website: www.apskanpur.in Mob: 09795647531, 09889787531, 0848664308 E-mail:[email protected]

147

Lens maker formula:

In case of a mage formula by lens the incident ray is reflected twice at first surface &

second surface respectively. The image formed by the first surface acts as object for the

second

μ1 = μm μ2 = μL

O I2 I1

R1 R2

u v

v1

Formula for refraction at curved surfaces

(μ2 /v) – (μ1 /u) = ((μ2 – μ1)/R) (*)

for 1st surface (μ2 /v1) – (μ1 /u) = ((μ2 – μ1)/R1) (1)

for 2nd surface (μ1 /v) – (μ2 /v1) = ((μ1 – μ2)/R2) (2)

Adding (1) & *(2) we get

(μ2 /v1) – (μ2 /v1) +( μ1 /v) – (μ1 /u) = ((μ2 – μ1)/R1) +((μ1 – μ2)/R2)

μ1[(1/v) – (1/u)] = (μ2 – μ1)[1/R1 - 1/R2]

[(1/v) – (1/u)] =[(μ2 / μ1)-1][ 1/R1 - 1/R2] { μ2 / μ1 = μ = μ2 / μm}

[(1/v) – /u ] = [μ-1][ 1/R1 - 1/R2] (*1)

If an object is placed at infinity u = ∞, v = f

(1/f) – (1/∞) = [μ-1][ 1/R1 - 1/R2]

/f = [μ-1][ 1/R1 - 1/R2] →Lens makers formula (*2)

comparing (*1) and (*2) we get

1/f = [(1/v) – (1/u)] → Lens formula (*3)

Note: Lens formula 1/v -1/u =1/f

(i) The above formula is valid for convex as well as concave lens for all positions of

object and it is independent of nature of object (real or virtual).

(ii) In numerical problems, it is convenient to use following expressions:

v = uf/(u +f), u = vf/(f-v), f = vu/(u-v)

Page 148: Geometrical Optics Book With Special Lecture

ANUJ PHYSICAL SCIENCES PHYSICS FOR IIT-JEE /AIEEE/AIPMT

Office: 128/652 “Y” Block Kidwai Nagar Kanpur -11 UP, Website: www.apskanpur.in Mob: 09795647531, 09889787531, 0848664308 E-mail:[email protected]

148

Problem 7: In case of a thin lens of focal length f in f an object is placed at a distance x1

from first focus and its image is formed at a distance x2 from the second focus then show

that f2 = x1x2

(a) f = √ x1x2 (b) f = (x1 + x2)/2 (c) f = (2x1x2)/( x1 + x2) (d)none

Solution:

f f

O I

x1 x2

u = x2 +f , v = x1 + f

(1/v) + (1/u) = (1/f)

[1/( x1 + f)] + [1/( x2 + f)] = 1/f

[( x2 + f) + ( x1 + f)]/[( x1 + f)( x2 + f)] = 1/f

f(x1 + x2 +2f) = ( x1 + f)( x2 + f)

2f2 + fx1+fx2 = x1x2 + x1f + x2f + f2

f2 = x1 x2

f = √ x1x2 This is the Newton’s formula.

Problem 8: If the distance between the real objects a real image formed by a lens & f is

the focal length of lens then Lmin is

(a) f (b) 2f (c) 3f (d) 4f

Solution: L = u + v

L = (√u - √v)2 +2√uv

Lmin ⇒ (√u - √v)2 =0 ⇒ u =v

Lmin = u +v = v +v = 2v

(1/v) + (1/u) = (1/f)

(1/v) + (1/v) = (1/f)

2/v = 1/f

V = 2f

Lmin = u +v = 2f + 2f = 4f

Page 149: Geometrical Optics Book With Special Lecture

ANUJ PHYSICAL SCIENCES PHYSICS FOR IIT-JEE /AIEEE/AIPMT

Office: 128/652 “Y” Block Kidwai Nagar Kanpur -11 UP, Website: www.apskanpur.in Mob: 09795647531, 09889787531, 0848664308 E-mail:[email protected]

149

Problem 9: An object is placed at A (OA >f). Here f is the focal length of the lens. The

image is formed at B. A ⏊r erected at O & C is chooses such that ∠BCA = 900 let OA = a,

OB = b & OC = c then the values of f is

(a) (a + b)3/c2

(b) (a + b)c/(a+ c)

(c) c2/(a + b)

(d) a2/ (a +b +c)

C

C

A O B

a b

[(1/v) – (1/u)]= 1/f

[(1/b) - (1/-a)]= 1/f

1/b + 1/a = 1/f

(1/f) = 1/a + 1/b

f = ab/ (a + b) (1)

From Δ ABC

Ac2 + Bc2 = AB2

a2 + c2 + b2 + c2 = (a + b)2

a2 + b2 +2c2 = a2 +b2 + 2ab

c2 = ab (2)

substituting (3) in (2)

f = ab/ (a + b)

f = c2 / (a +b)

Page 150: Geometrical Optics Book With Special Lecture

ANUJ PHYSICAL SCIENCES PHYSICS FOR IIT-JEE /AIEEE/AIPMT

Office: 128/652 “Y” Block Kidwai Nagar Kanpur -11 UP, Website: www.apskanpur.in Mob: 09795647531, 09889787531, 0848664308 E-mail:[email protected]

150

Problem 10: A diverging lens of focal length 10 cm is placed 10 cm in front of a plane

mirror as shown in fig. Light from a very far away source falls on the lens what is the

distance of the final image?

d

Solution:

I1 I2

20 cm

10 cm 10 cm

(1/v) – (1/u) = (1/f)

(1/v) – (1/-30) = + (1/-10)

(1/v) = (-1/10) – (1/30) = -(4/30) ⇒ v = -7.5

The final image distance = 2.5 cm in front of the mirror.

Page 151: Geometrical Optics Book With Special Lecture

ANUJ PHYSICAL SCIENCES PHYSICS FOR IIT-JEE /AIEEE/AIPMT

Office: 128/652 “Y” Block Kidwai Nagar Kanpur -11 UP, Website: www.apskanpur.in Mob: 09795647531, 09889787531, 0848664308 E-mail:[email protected]

151

Problem 11:

10 cm 20 cm -5 cm

O

10 cm d1 d2

(a)

The values of d1 and d2 for final rays to be parallel to the principle axis are: (focal length

of the lenses are written on the lenses)

(a) d1 = 10 cm, d2 = 15 cm (b) d1 = 20 cm, d2 = 15 cm

(c) d1 = 30 cm, d2 = 15 cm (d) none of these

Solution:

10 cm d1 d2 5 cm

20 cm

(b)

d2 = 20 -5 = 15 cm

d1 can take any value.

Hence option (a), (b), (c) all are correct

Page 152: Geometrical Optics Book With Special Lecture

ANUJ PHYSICAL SCIENCES PHYSICS FOR IIT-JEE /AIEEE/AIPMT

Office: 128/652 “Y” Block Kidwai Nagar Kanpur -11 UP, Website: www.apskanpur.in Mob: 09795647531, 09889787531, 0848664308 E-mail:[email protected]

152

Problem 12:

O C

(a)

An object is placed at a distance 0f 15 cm from a convex lens of focal length 10 cm on the

other side of the lens, a convex mirror is placed at its focus such that the image formed

by combination coincides with the object itself. Find the focal length of concave mirror.

Solution:

For retracting of ray; ray must fall normally on mirror i.e., towards the centre of

curvature.

(1/v) – (1/u) = (1/f) u = -15

(1/v) + (1/15) = (1/10) f = 10 cm, v = 30 cm

For mirror (1/v) + (1/u) = (1/f) u = 2f

(1/v) + (1/2f) = (1/f) v = 2f

Hence from the ray diagram

2f = 30-10

f = 10 cm

O C

15 cm 10 cm 2f

30 cm

(b)

Page 153: Geometrical Optics Book With Special Lecture

ANUJ PHYSICAL SCIENCES PHYSICS FOR IIT-JEE /AIEEE/AIPMT

Office: 128/652 “Y” Block Kidwai Nagar Kanpur -11 UP, Website: www.apskanpur.in Mob: 09795647531, 09889787531, 0848664308 E-mail:[email protected]

153

Problem 13: 25 cm x

S 2 cm

f = 20 cm

(a)

A point source of light S is placed on the axis of a lens of focal length 20 cm as shown in

the figure. A screen is placed normal to the axis of lens at a distance x from it. Treat all

rays as paraxial.

(a) As x is increased from zero intensity continuously decreases

(b) As x is increased from zero intensity first increases then decreases

(c) Intensity at centre of screen for x =90cm and x = 110 cm is same

(d) Radius of bright circle obtained on screen is equal to 1 cm for x = 200 cm

Solution: (b, c, d)

90 cm 110 cm

S I

25 cm 100 cm

O

(1/v) – (1/u) = (1/f) ⇒ (1/v) – [1/(-25)] = (1/20)

⇒ (1/v) = (1/20) – (1/25) = (1/100)

⇒ v = 100 cm

From O to I intensity increases and then decreases at x = 90 cm and 110 cm intensity is

same.

(d) Radius at x = 200 cm is equal to radius of lens.

Page 154: Geometrical Optics Book With Special Lecture

ANUJ PHYSICAL SCIENCES PHYSICS FOR IIT-JEE /AIEEE/AIPMT

Office: 128/652 “Y” Block Kidwai Nagar Kanpur -11 UP, Website: www.apskanpur.in Mob: 09795647531, 09889787531, 0848664308 E-mail:[email protected]

154

Problem 14: A convex lens of focal length 20 cm is placed 10 cm in front of a convex

mirror of radius of curvature 15 cm. Where should a point object be placed in front of

the lens so that it images on to itself?

Solution:

X 10 cm 15 cm

The convex lens and the convex mirror are shown in fig. The combination behaves like a

concave mirror.

Let the distance of the object from the lens be x.

For the ray to retrace is path it should be incident normally on the convex mirror, or in

other words the rays should pass through the centre of curvature of the mirror.

From the diagram we see that for the lens

u = -x, f = +20 cm, v = +10+15 = +25 cm

From the lens equation we get

(1/v) – (1/u) = (1/f)

or (1/25) – (1/-x) = (1/20)

or x = 100 cm

Page 155: Geometrical Optics Book With Special Lecture

ANUJ PHYSICAL SCIENCES PHYSICS FOR IIT-JEE /AIEEE/AIPMT

Office: 128/652 “Y” Block Kidwai Nagar Kanpur -11 UP, Website: www.apskanpur.in Mob: 09795647531, 09889787531, 0848664308 E-mail:[email protected]

155

Problem 15:

Y y

I1

X X

L1 L2

X 30

A convex lens of focal length 10 cm is placed 30 cm in front of a second convex lens also

of the same focal length. A plane mirror is placed after the two lenses. Where should a

point object be placed in front of the first lens so that it images on to itself?

Solution: The convex lenses and the plane mirror are shown in figure. The

combination behaves like a concave mirror.

Let the distance of the object from the first lens be x.

For the ray to retrace its path it should be incident normally on the plane mirror.

From the diagram we see that for lens L2,

From the lens equation we get

(1/v) – (1/u) = (1/f)

or u = -10 cm

From the diagram we see that for lens L1,

v = 30-10 = 20 cm, f = +10cm, u =-x

From the lens equation we get (1/v) – (1/u) = (1/f)

or (1/20) – (1/-x) = (1/10)

or x = 20 cm

Page 156: Geometrical Optics Book With Special Lecture

ANUJ PHYSICAL SCIENCES PHYSICS FOR IIT-JEE /AIEEE/AIPMT

Office: 128/652 “Y” Block Kidwai Nagar Kanpur -11 UP, Website: www.apskanpur.in Mob: 09795647531, 09889787531, 0848664308 E-mail:[email protected]

156

Problem 16:

I

O C P M

70 cm 25 cm

40 cm

A diverging lens, focal length f1 = 20 cm, is separated by 5 cm from a converging mirror,

focal length f2 = 10 cm. Where an object should be placed so that a real image is formed

at the object itself?

Solution: Let the object be placed at a distance x to the left of the lens.

From the lens equation,

(1/v) = (1/-20) + (1/-x)

v = - [20x/(x+20)]

A virtual image is formed due to first refraction at the lens. This image is an object for

reflection from the concave mirror. Object distance is

– [5+ {20x/ (20+x)}] = - [(25x +100)/(x+20)]

From mirror equation, (1/v’) + [-(x+20)/(25x+100)] = (1/-10)

(1/v’) = (-1/10) +[(x+20)/(25x+100)]= [(10x +200-25x-100)/250(x+4)]

v’ = -[50(x+4)/(3x-20)]

This image is formed to the left of the mirror. Object distance for second refraction

through concave lens, u’’ =- [5-{50(x+4)/ (3x-20)}]

we assumed that second image lies between lens and mirror.

The final image is produced at the object itself hence v’’ = +x

From lens equation, (1/x) – 1/[5-{(50x+4)/(3x-20)}]= 1/-20

on solving for x, we get

25x2 -1400x -6000 = 0

x2 -56x -240 = 0

(x-60) (x+4) = 0

Hence x = 60 cm

The object must be placed at 60 cm to the left of the diverging lens.

Page 157: Geometrical Optics Book With Special Lecture

ANUJ PHYSICAL SCIENCES PHYSICS FOR IIT-JEE /AIEEE/AIPMT

Office: 128/652 “Y” Block Kidwai Nagar Kanpur -11 UP, Website: www.apskanpur.in Mob: 09795647531, 09889787531, 0848664308 E-mail:[email protected]

157

Problem 17: A diverging lens (f1=20 cm) is separated by 25 cm from a concave mirror

(f2 =20 cm). An object is placed 70 cm to the left of the lens. Find the focal length of the

lens if the image coincides with the object.

Solution: The light refracts through the lens, reflects at the mirror and finally passes

once again through the lens. The final image will coincide with the object if it retraces its

path after reflection from the mirror, i.e. the ray strikes the mirror normally. The

normal rays at the mirror after being extended must pass through the centre of

curvature of the mirror. Thus, the object for second refraction at the lens is at C and its

image is at I

I

O C P M

70 cm 25 cm

40 cm

From figure PC = (40-25) = 15 cm

from lens equation, (1/-15) – (1/-70) = (1/f)

f = -19.1 cm

Page 158: Geometrical Optics Book With Special Lecture

ANUJ PHYSICAL SCIENCES PHYSICS FOR IIT-JEE /AIEEE/AIPMT

Office: 128/652 “Y” Block Kidwai Nagar Kanpur -11 UP, Website: www.apskanpur.in Mob: 09795647531, 09889787531, 0848664308 E-mail:[email protected]

158

Displacement method:

Problem 18 (A):

(i) A thin converging lens of focal length f is placed between an object and a screen fixed

at a distance D apart, there are two positions of the lens at which a sharp image of the

object is formed on the screen if

(a) D = 4f (b) D > 4f (c) D < 4f (d) None

(ii) In above question if distance between the two positions of the lens is x then the

focus length of the convergent lens is

(a) f = (D2 – x2)/4D (b) f = x2 / 4D (c) (D2 + x2)/4D (d) None (iii) If m1 & m2 is the magnification in the above case then focal length f is

(a) f = x/ m1-m2 (b) f = m1-m2/x (c) f = x/ m1+m2 (d) None

Solution:

u v

O

Object

f screen

If the object is at distance D from the lens, the distance of image from the lens is

u + v = D, v = D – u

so from lens formula (1/v) – (1/u) = (1/f) for convergent lens u ⇒ -u

[1/(D-u)] + (1/u) = 1/f

f [u + (D-u)] = u(D-u)

fu + fD – fu = Du – u2

u2 – Du + f D =0

u = ± [√D2 -4(1)fD + D]/2

u = [D±√ D2 -4fD]/2 = [D ± √D(D-4f)]/2

Now there are 3 possibilities:

(1) if D – 4f >0 then two roots of the equation will be real i.e.

u1 = [D + √D(D-4f)]/2, u2 = [D - √D(D-4f)]/2

so there are two positions of lens at u1 & u2 from the object for which real image is

formed.

(2) If D – 4f =0 then only one roots of the equation which is real, u = D/2,

v = D-u = [D – (D/2)] = D/2, v = D/2 = 2f

Page 159: Geometrical Optics Book With Special Lecture

ANUJ PHYSICAL SCIENCES PHYSICS FOR IIT-JEE /AIEEE/AIPMT

Office: 128/652 “Y” Block Kidwai Nagar Kanpur -11 UP, Website: www.apskanpur.in Mob: 09795647531, 09889787531, 0848664308 E-mail:[email protected]

159

so only one position of image is possible.

(3) If D -4f <0, then roots are imaginary, so physically no position of lens is possible.

Note: this method is called displacement method & is used in lab to determine the focal

length of convergent lens.

In case of displacement method:

(1) If the distance between the two positions of the lens is x

x = u2 – u1 = ½[D +√D (D-4f)] -1/2 [D-√D (D-4f)] = √D (D-4f)

x2 = D (D -4f)

f = (D2 – x2)/4D

(2) The image distances corresponding to two positions of lens will be

v1 = D – u1 = D –(1/2)[D+√D(D-4f)] = (1/2)[D-√D(D-4f)] = u2

v2 = D – u2 = D –(1/2)[D-√D(D-4f)] = (1/2)[D+√D(D-4f)] = u1

i.e. for two positions of the lens object & image distances are interchangeable.

(3) As x = u2 – u1 & D = u1 + v1 = u1 + u2 as v1 = u2

x = u2 – u1 , D = u2 + u1 ⇒ D + x = 2u2 ⇒ u2 = (D + x)/2

D – x = 2u1 ⇒ u1 = (D - x)/2

so the magnification for the positions of the lens will be

m1 = u1/v1= u1/u2 = [(D-x)/2]/[ (D +x)/2] = (D-x)/(D +x)

m2= u2/v2 = u2/u1 = [(D +x)/2]/[ (D-x)/2] = (D+ x)/(D -x)

(a) m1m2 = [(D-x)/(D +x)][(D+ x)/(D -x)] = 1⇒ (I1/O1)( I2/O2) =1⇒ O = √ I1I2

(b) m1/m2 = [(D-x)/(D +x)]/[(D+ x)/(D -x)]= [(D-x)/(D +x)]2

(c) m1-m2 = [(D-x)/(D +x)]-[(D+ x)/(D -x)]= [(D-x)2 – (D+ x)2]/(D2 – X2) = 4Dx/(D2 – X2)

m1-m2 = x[4D/(D2 – X2)]

as we know f =(D2 – X2)/4D

m1-m2 = x/f

f = x / m1-m2

D

X v2

u2

u1

v1

Page 160: Geometrical Optics Book With Special Lecture

ANUJ PHYSICAL SCIENCES PHYSICS FOR IIT-JEE /AIEEE/AIPMT

Office: 128/652 “Y” Block Kidwai Nagar Kanpur -11 UP, Website: www.apskanpur.in Mob: 09795647531, 09889787531, 0848664308 E-mail:[email protected]

160

Problem 18(B): A luminous object and a screen are at a fixed distance D apart.

(a) show that a converging lens of focal length f, placed between object and screen will

form a real image on the screen for two lens positions that are separated by a distance

d = √D(D-4f)

(b) Show that [(D-d)/(D+d)]2 gives the ratio of the two image sizes for these two

positions of the lens.

(c) If the heights of two images are h1 and h2 respectively, the height of the object is

h =√h1h2

(d) If the distance between two positions of the lens is d,

f = (D2-d2)/4D and f = d/(m1-m2)

where m1 and m2 are magnifications in the two positions of the lens.

Solution:

(a) Let the object distance be x; then the image distance is D-x.

From lens equation, (1/x) + (1/(D-x)) = (1/f)

On algebraic rearrangement we get x2 –Dx +Df =0

On solving for x, we get x1 = [D-√D(D-4f)]/2

x2 = [D+√D(D-4f)]/2

The distance between the two object positions is d = x2 – x1 = √D(D-4f)

(b) If the object is at u = x1

m1 = I1/O = (D-x1)/x1

Now, x1 = (1/2)(D-d), where d = √D(D-f)

so, m1 = [D-(D-d)/2]/[(D+d)/2] = (D+d)/(D-d)

Similarly, when the object is at x2, the magnification is x2 = (1/2) (D+d)

m2= I2/O = (D-x2)/x2 =[D-(D+d)/2]/[(D+d)/2] = (D-d)/(D+d)

The ratio of magnification is (m2/m1)= [(D-d)/(D+d)]/[(D+d)/(D-d)] = [(D-d)/(D+d)]2

(c) As m1 = I1/O= v1/u1

and m2 = I2/O= v2/u2

m1*m2 = I1I2/O2 =(v1/u1)(v2/u2) =1

Hence O = √I1I2

(d) As d = √D(D-4f)

d2 = D2-4Df

f = (D2-d2)/4D

Also m1-m2 =[(D+d)/(D-d)]-[ (D-d)/(D+d)]= 4Dd/(D2-d2)

Hence m1-m2 = d/f

or f = d/(m1-m2)

Formula /f = μ-1) [(1/R1)-(1/R2)] for different shapes of thin lenses:

Page 161: Geometrical Optics Book With Special Lecture

ANUJ PHYSICAL SCIENCES PHYSICS FOR IIT-JEE /AIEEE/AIPMT

Office: 128/652 “Y” Block Kidwai Nagar Kanpur -11 UP, Website: www.apskanpur.in Mob: 09795647531, 09889787531, 0848664308 E-mail:[email protected]

161

For Convex lens: (1/f) = (μ-1) [(1/R1)-(1/R2)] = (μ-1) [(1/R1)-(1/-R2)]

⇒ /f = μ-1) [(1/R1) + (1/R2)]

For Concave lens: (1/f) = (μ-1) [(1/R1)-(1/R2)] = (μ-1) [(1/-R1)-(1/+R2)]

⇒ /f = μ-1) [(1/R1) +(1/R2)]

For Plano Convex lens:(1/f) = (μ-1) [(1/R1)-(1/R2)] = (μ-1) [(1/∞)-(1/-R)]

⇒ /f = μ-1) [1/R] OR

(1/f) = (μ-1) [(1/R1)-(1/R2)] = (μ-1) [(1/R)-(1/-∞)]

⇒ /f = μ-1)/R

For Plano Concave lens:(1/f) = (μ-1) [(1/R1)-(1/R2)] = (μ-1) [(1/-R)-(1/-∞)]

⇒ (1/f) =- [ μ-1)/R] OR

(1/f) = (μ-1) [(1/R1)-(1/R2)] = (μ-1) [(1/∞)-(1/ R)]

⇒ (1/f) =- [ μ-1)/R]

Note: (1) If surface is double convex or double concave then R1 = R2

for equviconvex (1/f) = 2[(μ-1)/R] for equviconcave (1/f) =- 2[(μ-1)/R]

(2) Sun goggles: In case of sum goggles radii of curvatures R1 = R2 = R

(1/f) = (μ-1) [(1/R1)-(1/R2)] = (μ-1) [(1/R)-(1/R)] =0

P=0, f = ∞ This is why sun goggles have no power or infinite focal length.

Glass slab: same is true for glass slab (transparent sheet) but with difference

R1 = ∞, R2 = ∞

(1/f) = (μ-1) [(1/R1)-(1/R2)] = (μ-1) [(1/∞)-(1/∞)] =0 , P=0, f = ∞

Problem 19: What is the refractive index of material of a Plano convex lens? If the

radius of curvature of convex surface is 10 cm & focal length of the lens is 30 cm.

Solution:

R1 = ∞ R2 = -R

Plano convex (1/f) = (μ-1) [(1/R1)-(1/R2)]= (μ-1) [(1/∞)-(1/-R)] = (μ-1)/R

f = [R/(μ-1)] ⇒ (μ-1) = R/f ⇒ μ = [1+(R/f)] = [1+ (10/30)]

μ = + /3 = 4/3

Page 162: Geometrical Optics Book With Special Lecture

ANUJ PHYSICAL SCIENCES PHYSICS FOR IIT-JEE /AIEEE/AIPMT

Office: 128/652 “Y” Block Kidwai Nagar Kanpur -11 UP, Website: www.apskanpur.in Mob: 09795647531, 09889787531, 0848664308 E-mail:[email protected]

162

Problem 20: Diameter of a Plano convex lens is 6 cm and its thickness at the centre is

3mm. What is the focal length of the lens? If the speed of light in the material of lens is

2 X 108 m/s

(a) 30cm (b) 20 cm (c) -30 cm (d) none

Solution:

r

R

(R-y)

According to lens maker’s formula

(1/f) = (μ-1) [(1/R1)-(1/R2)]

As the lens is Plano convex R1=R, R2=∞ [or R1 = ∞, R2 = -R]

f = R/(μ-1) (1)

Now as speed of light in vacuum = 3*108 m/s

Now as speed of light in medium = 2*108 m/s

μ = c/v = [(3*108 m/s)/( 2*108 m/s)] = 3/2 = 1.5 (2)

If r is the radius & y is the thickness of lens (at the centre) the radius of curvature R of

its curved surface in accordance to fig. Will be given by

R2 = r2 +(R -y)2

R2 = r2 +R2 +y2-2Ry

[r2 +y2]/2y = R

r = 6/2, y = 3mm

R = [32 + 32(1/100)]/[2(3/10)]

R=32[1+(1/100)]/(6/10)

R = 32[101/100][10/6]

R= (3/2)10 = 15 cm

so substituting the values μ & R in eq.(1)

f = R/(μ-1)

f= 15/(1.5-1) = 15/.5 = 150/5

f= 30 cm

Page 163: Geometrical Optics Book With Special Lecture

ANUJ PHYSICAL SCIENCES PHYSICS FOR IIT-JEE /AIEEE/AIPMT

Office: 128/652 “Y” Block Kidwai Nagar Kanpur -11 UP, Website: www.apskanpur.in Mob: 09795647531, 09889787531, 0848664308 E-mail:[email protected]

163

Problem 21: Lens immersed in water:

A glass convex lens of refractive index 3/2 has got a focal length of 0.3m. If it is

immersed in water of refractive index 4/3, then Find the focal length of the lens.

(1/f) = (μ-1) [(1/R1)-(1/R2)]

μ = μL/μm

(1/fa) = [μg/μa)-1] [(1/R1)-(1/R2)] (1)

(1/fω) = [μg/μω)-1] [(1/R1)-(1/R2)] (2)

Dividing eq. (1) and (2)

[(1/fa)/ (1/fω)] = {[μg/μa)-1][(1/R1)-(1/R2)]}/{ [μg/μω)-1] [(1/R1)-(1/R2)]}

fω /fa =[μg/μa)-1]/[μg/μω]

⇒ fω = [[μg/μa)-1]/[μg/μω]]fa

In this problem fa = 0.3, μa=1, μg=3/2

fω = [[μg/μa)-1]/[μg/μω]]fa

fω = (0.3) [{[(3/2)-1]/1}/{[(3/2)/(4/3)]-1}]

fω = [(0.3)(1/2)]/[(9/8)-1]

fω = [(0.3)(1/2)]/(1/8) = (0.3*1*8)/2

fω = 1.2m

Problem 22(AIEEE 2005): A thin glass (refractive index 1.5) lens has optical power

in a liquid medium with refractive index 1.6 will be

(a) 1D (b) -1D (c) 25D (d) -25D

Solution:

(1/fo) = (μ-1)[(1/R1)-(1/R2)]

(1/fo) =(1.5-1) [(1/R1)-(1/R2)] (1)

1/fm =[(μg – μm)/ μm] [(1/R1)-(1/R2)]

1/fm = [(1.5/1.6)-1] [(1/R1)-(1/R2)] (2)

Dividing eq. (1) and (2)

fm/f0 =(1.5-1) / [(1.5/1.6)-1] = -8

(∵ f0 = 1/p = (-1/5)m

fm = -8* f0 = -8*(-1/5) =1.6 m

Pm= μ/ fm

Pm = 1.6/1.6 = 1D

Page 164: Geometrical Optics Book With Special Lecture

ANUJ PHYSICAL SCIENCES PHYSICS FOR IIT-JEE /AIEEE/AIPMT

Office: 128/652 “Y” Block Kidwai Nagar Kanpur -11 UP, Website: www.apskanpur.in Mob: 09795647531, 09889787531, 0848664308 E-mail:[email protected]

164

Problem 23: A point object is places at 30 cm from a convex glass lens (μg = 3/2) of

focal length 20 cm. The final image will be formed at infinity if

(a) The whole system is immersed in liquid of refractive index 4/3.

(b) The whole system is immersed in liquid of refractive index 9/8.

(c) Another concave lens of focal length 60 cm is placed in contact with its previous lens.

(d) Another concave lens of focal length 60 cm is placed at a distance of 30 cm from the

first lens.

Solution:

30 cm

μg = 3/2, u = -30cm, f = 20cm, v=∞ (final image)

after contact of lens (1/v) – (1/u) = (1/f)

(1/∞) – (1/-30) = 1/f

f = + 30cm this is combined focal length

V =∞

30 cm

f =?

1/f = 1/f1 + 1/f2, 1/+30 = 1/20 + 1/f2

1/f2 = - 1/20 + 1/30 = (-3 +2)/60 = -1/60 f2 = -60

Hence another concave lens of focal length 60 cm is placed with contact in first lens.

1/f = (μ-1) [(1/R1)-(1/R2)] ⇒ 1 20 = [(μg μa)-1][(1/R1)-(1/R2)] (1)

If whole system is immersed in a liquid of refractive index μ & focal length becomes 30

cm.

1/30 =[(μg μω)-1][(1/R1)-(1/R2)] (2)

dividing eq.(1) and (2) we get

(3/2) = [(3/2)-1]/[{(3/2)/μ}-1] = (1/2)/[(3-2μ)/2μ] = (1/2)[2μ/(3-2μ)] ⇒ 3/2

=μ /(3-2μ) = 9-6μ = 2μ ⇒ μ = 9/8

Page 165: Geometrical Optics Book With Special Lecture

ANUJ PHYSICAL SCIENCES PHYSICS FOR IIT-JEE /AIEEE/AIPMT

Office: 128/652 “Y” Block Kidwai Nagar Kanpur -11 UP, Website: www.apskanpur.in Mob: 09795647531, 09889787531, 0848664308 E-mail:[email protected]

165

Problem 24: As shown in fig. a spherical lens of radii of curvatureR1=R2=10 cm is cut

in a glass cylinder. Determine the focal length and nature of air lens. If a liquid of

refractive index 2 is filled in the lens what will happen to its focal length and nature.

μ = 1.5

μ =1 μ = 1.5

μ = 2

Solution:

According to lens maker’s formula

(1/f) = (μ-1) [(1/R1)-(1/R2)] with μ = μL/μm

μ = μA/μg = 1/(3/2) = 2/3, R1 = +10, R2 = -10 cm

(1/f) = [(2/3)-1][(1/10)-(1/10)] = -2/30 ⇒ f = -15cm

i.e. the air lens in glass behaves as divergent lens of focal length 15 cm.

When the liquid μ = 2 is filled in the air cavity μ = μL/μm = 2/1.5 = 4/3

so that (1/f’)= [(4/3)-1][(1/10)-(1/-10)] = 2/30

f’= + 15 cm

i.e. the liquid lens in glass behave as a convergent lens of focal length 15 cm.

Problem 25: Which one of the following spherical lenses does not exhibit dispersion?

The radii of curvature of the surface of the lenses are as given in the diagrams.

(a) (b) (c) (d)

R1 R1 R ∞ R R R ∞

Solution: 1/f = (μ-1)[(1/R1) –(1/R2)]

d (1/f) =d(μ-1)[(1/R1) –(1/R2)]

for no dispersion d(1/f) =0,

dμ[(1/R1) –(1/R2)] = 0

dμ ≠0 then [(1/R1) –(1/R2)] = 0 R1=R2

Hence optics c is correct.

Page 166: Geometrical Optics Book With Special Lecture

ANUJ PHYSICAL SCIENCES PHYSICS FOR IIT-JEE /AIEEE/AIPMT

Office: 128/652 “Y” Block Kidwai Nagar Kanpur -11 UP, Website: www.apskanpur.in Mob: 09795647531, 09889787531, 0848664308 E-mail:[email protected]

166

Lecture-10

Geometrical Optics

Physics for IIT - JEE

Refraction from curved surfaces:

Magnification problems Lenses

Positions, nature and size of the

image for different positions of object

Analysis of graphs

Measurement of refractive index of

liquid by a convex lens:

Formula related to combinations of

thin lenses

Page 167: Geometrical Optics Book With Special Lecture

ANUJ PHYSICAL SCIENCES PHYSICS FOR IIT-JEE /AIEEE/AIPMT

Office: 128/652 “Y” Block Kidwai Nagar Kanpur -11 UP, Website: www.apskanpur.in Mob: 09795647531, 09889787531, 0848664308 E-mail:[email protected]

167

Magnification: This is defined as the ratio of size of image to the size of object

magnification (m) = Size of image (I)/Size of object (O)

(a) Transverse or lateral magnification (mT): It is defined as the ratio of any

transverse dimension of the image to the corresponding transverse dimension of the

object. Let the size of the object and image be denoted by ’O’ and ‘T’ respectively

mT = (I/O) = v/u

Note:

(i) the above formula is valid for convex as well as concave lens for all positions of

object and it is independent of nature of object (real or virtual)

(ii) linear magnification for a lens can also be expressed as

m = I/O =v/u =(f-v)/f = f/(f +u)

(b) Longitudinal or axial magnification (mL):

P Q P 1 Q1

C

It is defined as the ratio of the length of the image to the corresponding length of the

object ,

mL = (vP –vQ)/(up-uQ)

For small object, mL = dv/du

differentiating lens equation, we get –[dv/v2]–[-du/u2] =0

or [dv/v2] – [du/u2] = 0

and therefore mL = dv/du = (v2/u2)=(v/u)2 =m2

Therefore, for small object longitudinal magnification is square of transverse

magnification.

(c) Angular magnification: It is defined as the ratio tan α2/tan α1, where α1 and α2 are

the slope angles of the object ray and the image ray respectively as shown in the figure

it is denoted by γ.

∴ γ = tan α2/tan α1

Page 168: Geometrical Optics Book With Special Lecture

ANUJ PHYSICAL SCIENCES PHYSICS FOR IIT-JEE /AIEEE/AIPMT

Office: 128/652 “Y” Block Kidwai Nagar Kanpur -11 UP, Website: www.apskanpur.in Mob: 09795647531, 09889787531, 0848664308 E-mail:[email protected]

168

By Lagrange-Helmholtz equation

μ1h1sin α1 = μ2h2sin α2

μ1h1tan α1 = μ2h2tan α2 [ ∵α1 and α2 are small ]

u A v

α1 α2

P F1 O F2 Q

since μ1 = μ2

h1tan α1 = h2tan α2

∴ Angular magnification γ = tan α2/tan α1 = h1/h2 = 1/m

That is, the angular magnification is reciprocal of the lateral magnification produced by

a lens.

Problem 1(IIT – JEE 2003): The size of the image of an object, which is at infinity,

as formed by a convex lens of focal length 30 cm is 2 cm. If a concave lens of focal length

20 cm is placed between the convex lens and the image at a distance of 26 cm from the

convex lens, calculate the new size of the image.

(a) 1.25 cm (b) 2.5 cm (c) 1.05 cm (d) 2cm

Solution:

5 cm

l1

28cm 4 cm l 2

(1/v) – (1/u) = 1/f or (1/v) – ¼ = 1/ -20 or v = 5 cm

magnification for concave lens m = v/u = 5/4 = 1.25

As size of the image at I1 is 2 cm. Therefore, size of image at I2 will be 2* 1.25 = 2.5 cm

Page 169: Geometrical Optics Book With Special Lecture

ANUJ PHYSICAL SCIENCES PHYSICS FOR IIT-JEE /AIEEE/AIPMT

Office: 128/652 “Y” Block Kidwai Nagar Kanpur -11 UP, Website: www.apskanpur.in Mob: 09795647531, 09889787531, 0848664308 E-mail:[email protected]

169

Problem 2: When an object is at distance x &y from lens, a real image and a virtual

image is formed having same magnification. The focal length of the lens is

(a) (x +y)/2 (b) x- y (c) √xy (d) x +y

Solution: The given lens is a convex lens. Let the magnification be m.

(1/v) + (1/u) = (1/f)

convex lens (1/v) + (1/u) = (1/f)

m = v/u ⇒ real image m =+ve ⇒ v = mu

m = -v/u ⇒ virtual image m = -ve ⇒ v = -mu

ureal → x & vvirtual →y

(1/v) + (1/u) = (1/f) ⇒ v = mx for real image

(1/mx) + (1/x) = (1/f) (1)

(1/v) + (1/u) = (1/f) ⇒ v = -my for virtual image

(1/-my) + (1/y) = (1/f) (2)

Now equating eq.(1) & (2) we get

(1/mx)+ (1/x) = (1/-my) + (1/y) = (1/f)

(1/mx) +(1/my) = (1/y) – (1/x)

(1/m)[(1/x)+(1/y)] = [(1/y)- (1/x)]

(1/m) [(x + y)/(x- y)]=[(x -y)/xy]

m = [(x + y)/(x- y)] (3)

substituting values of m in eq. (1) (1/mx) + (1/x) = (1/f)

(1/x)[(1/m)+1]= 1/f

(1/x) [(x - y)/(x+ y)]+1=1/f

(1/x)[(x –y +x + y)/ (x +y)]= 1/f

2/(x +y) = 1/f

f= (x +y)/2 Hence option (a) is correct

Problem 3(IIT-JEE 2010): The focal length of a thin biconvex lens is 20 cm. When

an object is moved from a distance of 25 cm in front of it to 50 cm, the magnification of

its image changes from m25 to m50 The ratio m25 / m50 is

(a) 6 (b) 7 (c)8 (d) 9

Solution:

(1/v)-(1/u) = 1/f

(u/v)-1=u/f

(u/v) = (u+f)/f

m=v/u=f/ (u+f)

m25/m50 = [20/ (-25+20)]/ [20/ (-50+20)] =6

Page 170: Geometrical Optics Book With Special Lecture

ANUJ PHYSICAL SCIENCES PHYSICS FOR IIT-JEE /AIEEE/AIPMT

Office: 128/652 “Y” Block Kidwai Nagar Kanpur -11 UP, Website: www.apskanpur.in Mob: 09795647531, 09889787531, 0848664308 E-mail:[email protected]

170

Problem 4: A pencil of height 1 cm is placed 30 cm from an equviconvex lens,

refractive index n =3/2, radius of curvature for both the surfaces, R1 = R2 = R = 10 cm.

Find the location of the image and describe its characteristics.

F

F optical axis

f f

Solution: The ray of light diverges from the object; it is a real object. Focal length of a

convex lens is positive.

(1/f) =[(n2-n1)/n1](2/R) = [(3/2) -1]/1(2/10) = 1/10

The given parameters are

u = -30 cm

f= +10 cm

substituting these values in thin lens equation, we have

(1/v) – (1/u) = (1/f)

(1/v) – (1/-30) = (1/10)

which on solving gives v = +15 cm.

The image distance is positive, which implies that the image is real. The lateral

magnification is

m = v/u = 15/(-30) = -1/2

The image is half the size of the object; size of the image is 0.50 cm. The negative sign

with magnification shows inverted image.

Problem 5: If the pencil in the previous problem is kept at 6 cm from the lens, locate

and characterize the image.

Solution: from lens equation, (1/v) = (1/f) + (1/u)

= (1/10) + (1/-6) = -(1/15)

v = - 15 cm

Lateral magnification, m = v/u = -15/-6 = 2.50

the minus sign with distances shows that the image is located on the side of the object.

The image is upright, because magnification is positive.

Page 171: Geometrical Optics Book With Special Lecture

ANUJ PHYSICAL SCIENCES PHYSICS FOR IIT-JEE /AIEEE/AIPMT

Office: 128/652 “Y” Block Kidwai Nagar Kanpur -11 UP, Website: www.apskanpur.in Mob: 09795647531, 09889787531, 0848664308 E-mail:[email protected]

171

Problem 6:

F F

If in the previous example we use a diverging lens with a focal length 10 cm to form an

image of the pencil kept 15 cm in front of the lens, locate and characterise the image.

Solution: In accordance with Cartesian sign convention the given parameters are

f = -10 cm, u = -15 cm

From lens equation, we have (1/v) = (1/f) + (1/u)

= [1/(-10)] +[1/(-15)] = -(1/6)

v = -6 cm

Lateral magnification, m = (v/u) = -6/-15 = 0.40

The minus sign with image shows that the image is located on the side of the object. The

magnification is positive and m <1, which shows that the image is upright and

diminished.

Problem 7: A pencil with a height of 5 mm is placed 45 cm to the left of a converging

lens of focal length 20 cm. A diverging lens of focal length 15 cm is at a distance 10 cm

from the first lens. What is (a) the image distance v1 and (b) the height I1 of the image

produced by the first lens? (c) What is the object distance for the second lens? Find (d)

the image distance v2 and (e) the height I2 of the image formed by the second lens.

Solution: The image formed by a converging lens can be real or virtual. It depends on

where the object is located relative to the focal point. If the object is to the left of the

focal point as in this case, the image is real and formed on the right side of the lens. If on

the other hand the object is located between the focal point and the lens, the image is

virtual, located on the left of the lens.

The image produced by the first lens acts as an object for the second lens; this image is

called the first image. The first image can lie between the lenses or to the right of the

second lens. In the first case it will be a real object and in the second case it will be a

virtual object for the second lens.

Page 172: Geometrical Optics Book With Special Lecture

ANUJ PHYSICAL SCIENCES PHYSICS FOR IIT-JEE /AIEEE/AIPMT

Office: 128/652 “Y” Block Kidwai Nagar Kanpur -11 UP, Website: www.apskanpur.in Mob: 09795647531, 09889787531, 0848664308 E-mail:[email protected]

172

F1 I1

O1 F1

(a)

F2 I2 O2 F2

(b)

F1 F2 F1 F1

(c)

(a) For the image formed by the first lens,

(1/v1) = (1/f1) + (1/u1) = (1/20) + (1/-45) = (1/36)

v1 = +36 cm

We thus find that the first lens is attempting to forms a real image, 36 cm behind it.

However, before this image can be formed the rays are intercepted by the second lens

located 10 cm behind it, see fig. (a) The point of convergence of these rays is a virtual

object for the second lens.

(b) The height of the image, I1 = O1 (v1/u1) = (5*10-1) (36/-45) = -4mm

The minus sign indicates that the image is inverted.

(c) The object distance for the second lens = (36-10) cm = 26 cm

to the right of the second lens; hence v2 = +26 cm

(d) From lens equation for the second lens,

(1/v2) = (1/f2) + (1/u2) = (1/-15) + (1/+26) = - (1/35.5)

v2= -35.5 cm

The negative sign shows that it is on the left of the diverging lens (virtual image)

(e) The image height for the first lens is object height for the second lens. Height of the

image formed by the second lens

I2 = O2 (v2/u2) = (-4*10-1) [-35.5/+26] = 5.46mm

The positive sign for I2 means that the final image has the same orientation as the first

image.

Page 173: Geometrical Optics Book With Special Lecture

ANUJ PHYSICAL SCIENCES PHYSICS FOR IIT-JEE /AIEEE/AIPMT

Office: 128/652 “Y” Block Kidwai Nagar Kanpur -11 UP, Website: www.apskanpur.in Mob: 09795647531, 09889787531, 0848664308 E-mail:[email protected]

173

Problem 8: An equviconvex lens, f1 = 10 cm, is placed 40 cm in front of a concave

mirror, f2 = 7.50 cm, as shown in fig. An object 2 cm high is placed 20 cm to the left of

lens. Find the location of three images:

(a) the image formed by lens as rays travel to the right,

(b) the image formed after rays reflect from mirror and

(c) the final image after leftward travelling rays once again pass through lens.

Complete the ray diagram and characterise the image.

Solution: (a) from lens equation, (1/v) – [1/(-20)] = (1/10), v = +20cm magnification,

m1= v/u = [+20/-20] = -1

image is real and inverted, same size as object.

(b) The first image acts as object for concave mirror.

Object distance for mirror is (40-20)cm

From mirror equation, (1/v’) + (1/-20) = (1/-7.5)

v’ = - 12 cm

magnification, m2 = - (u’/v’) = -[12/-20] = -0.6

The second image acts as objects for the lens. The object distance for second refraction

at the lens,

F1 F2 F3

20 cm 40 cm

u’’ = +28 cm

From lens equation, (1/v’’) - (1/+28) = 1/(-10), v’’ =-15.6 cm

Note the sign convention for f and u

magnification,m3 = v’’/u’’ = (-15.6)/(+28) = -0.556

- +

Ray of light (F1 positive)

F2 F1

Ray of light (F2 negative)

Page 174: Geometrical Optics Book With Special Lecture

ANUJ PHYSICAL SCIENCES PHYSICS FOR IIT-JEE /AIEEE/AIPMT

Office: 128/652 “Y” Block Kidwai Nagar Kanpur -11 UP, Website: www.apskanpur.in Mob: 09795647531, 09889787531, 0848664308 E-mail:[email protected]

174

The final image is real, inverted and lies 15.6 cm to the left of the lens. Overall

magnification, m = m1*m2*m3 = (-1) (-0.6) (-0.556) = -0.333

Problem 9: A biconvex lens, f1 = 20 cm, is placed 5 cm infront of a convex mirror, f2 =

15 cm, an object of length 2 cm is placed at a distance 10 cm from the lens. Find the

location of three images: (a) the image formed by the lens as the rays travel to the right,

(b) the image formed after the rays reflect from the mirror and (c) the final image after

the leftward travelling rays once again pass through the lens.

I1 O I2 I3

Solution: (a) From lens equation, (1/v) – (1/-10) = -(1/20) , v = -20 cm

magnification m1 = (-20)/(-10) = +2

image is virtual, erect and magnified.

(b) the first image acts as an object for the convex mirror. Object distance for the mirror,

u’ = (20+5) = 25 cm.

From mirror equation, (1/v’) + (1/-25) = (1/15)

v’ = +75/8 cm

magnification, m2 = (+75/8)/(-25) = 3/8

image is virtual (to the right of the mirror), erect and diminished.

(c) the object distance for second refraction at the lens = (75/8) +5 = 115/8

From lens equation, (1/v’’) – 1/(+115/8) = 1/-20

v’’= 460/9 = +51.1 cm

magnification m3 = (+460/9)/(115/8) = 32/9

overall magnification m = m1*m2*m3 = 2(3/8)(32/9) = 8/3

hence size of image=(8/3)2 cm= 5.33 cm

Final image is to the right of the lens at a distance 51.1 cm from the lens, real, erect and

magnified.

Page 175: Geometrical Optics Book With Special Lecture

ANUJ PHYSICAL SCIENCES PHYSICS FOR IIT-JEE /AIEEE/AIPMT

Office: 128/652 “Y” Block Kidwai Nagar Kanpur -11 UP, Website: www.apskanpur.in Mob: 09795647531, 09889787531, 0848664308 E-mail:[email protected]

175

Positions, nature and size of the image for different positions of object:

Convex lens:

(i) Object at infinity:

F

C B1

A1

In this case, u → ∞

(1/f) = (1/v) – (1/∞) or (1/f) = (1/v)

or v = f

Hence

Position: At F

Nature: Real and inverted

Size: Diminished (very small)

(ii) Object lying beyond 2F:

A

B C F 2F

2F F A1

From lens equation, we have (1/v) = (1/f) + (1/u)

If u > 2f, 2f > v > f, i.e., the image lies between f and 2f

Also, m =v/u always is less than one

Position: Between f and 2f

Nature: Real and inverted

Size: Small

Page 176: Geometrical Optics Book With Special Lecture

ANUJ PHYSICAL SCIENCES PHYSICS FOR IIT-JEE /AIEEE/AIPMT

Office: 128/652 “Y” Block Kidwai Nagar Kanpur -11 UP, Website: www.apskanpur.in Mob: 09795647531, 09889787531, 0848664308 E-mail:[email protected]

176

(iii) Object at 2F:

A

B C F 2F

2F F A1

Here, u = -2f

From lens equation, we have v = uf/(u + f) =[(-2f)(f)]/[-2f + f] = 2f

Also, m = v/u = 2f/-2f = -1

Position: At 2f

Nature: Real and inverted

Size: Same as that of object

(iv) Object lying between F and 2F:

B

2F A F F 2F A 1

B1

C

If 2f > u > f, it can be seen from lens equation that, v > 2f

Also, m = v/u is greater than one

Position: Beyond 2F

Nature: Real and inverted

Size: Enlarged

Page 177: Geometrical Optics Book With Special Lecture

ANUJ PHYSICAL SCIENCES PHYSICS FOR IIT-JEE /AIEEE/AIPMT

Office: 128/652 “Y” Block Kidwai Nagar Kanpur -11 UP, Website: www.apskanpur.in Mob: 09795647531, 09889787531, 0848664308 E-mail:[email protected]

177

(v) Object at F:

A

F

B C

Object at F

Here, u = -f

From lens equation, we have v = uf/(u+ f) = [(-f)(-f)]/[-f+ f] = ∞

Also, m = v/u → ∞

Position: At infinity

Nature: Real and inverted

Size: Highly magnified

iv Object lying between ‘F’ and optical centre ‘C’:

Here, u = -f

From lens equation, we have

(1/f) = (1/v) – (1/-f) = (1/v) + (1/f)

or (1/v) = (1/f) – (1/f) =0

A1 A

2F B1 F B C F

Object between F and optical centre

At C, u = 0

(1/v) = (1/f) + (1/0) = ∞ or v = 0

Position: Same side of lens

Nature: Virtual and erect

Size: Magnified

Page 178: Geometrical Optics Book With Special Lecture

ANUJ PHYSICAL SCIENCES PHYSICS FOR IIT-JEE /AIEEE/AIPMT

Office: 128/652 “Y” Block Kidwai Nagar Kanpur -11 UP, Website: www.apskanpur.in Mob: 09795647531, 09889787531, 0848664308 E-mail:[email protected]

178

Concave lens:

A

A’

B F B1 C

In concave lens whatever be the position of object, image formed is always virtual, erect

and small in size. If object is virtual concave lens may form real or virtual image

depending on the position of object. If u < f, image is real and if u > f, then image is

virtual.

Note:

Y Real image of virtual object Real image of a real object

O f u f

Real image of a virtual object

-f virtual image of virtual -f O u

object

concave lens convex lens

real image of a real object virtual image of a real object

In general, all situations in lenses can be summarised in u-v graph as shown in fig.

While interpreting these graphs remember following points:

(i) u is negative for real object and u is positive for virtual object

(ii) v is positive for real image and v is negative for virtual image

Page 179: Geometrical Optics Book With Special Lecture

ANUJ PHYSICAL SCIENCES PHYSICS FOR IIT-JEE /AIEEE/AIPMT

Office: 128/652 “Y” Block Kidwai Nagar Kanpur -11 UP, Website: www.apskanpur.in Mob: 09795647531, 09889787531, 0848664308 E-mail:[email protected]

179

SNo. Type of lens Shape of lens The factor [(1/R1)- (1/R2)]

Reciprocal of the focal length in air or vacuum

Nature of lens

(1) Concavo -convex

or [(1/x)-(1/y)] (μ-1) [(1/x)-(1/y)] Converging

(2) Convexo-concave

or -[(1/x)-(1/y)] -(μ-1) [(1/x)-(1/y)] Diverging

(3) Biconvex (1/x)+(1/y) (μ-1) [(1/x)+(1/y)] Converging

(4) Biconcave -[(1/x)+(1/y)] -(μ-1) [(1/x)+(1/y)] Diverging

(5) Plano convex or

1/R (μ-1)/R Converging

(6) Plano concave or

-1/R -(μ-1)/R Diverging

(7) Symmetrical biconvex

2/R (μ-1)/(2/R) Converging

(8) Symmetrical biconcave

-2/R -(μ-1)(2/R) Diverging

Graph of u vs. v for a lens:

According to lens equation, it is hyperbola, as shown in fig.

Page 180: Geometrical Optics Book With Special Lecture

ANUJ PHYSICAL SCIENCES PHYSICS FOR IIT-JEE /AIEEE/AIPMT

Office: 128/652 “Y” Block Kidwai Nagar Kanpur -11 UP, Website: www.apskanpur.in Mob: 09795647531, 09889787531, 0848664308 E-mail:[email protected]

180

(a) Convex lens:

Real object, real image v

2f

f virtual object, Real image

- ∞←u -2f -f 0 f 2f →u u→+∞

-f

-2f

Real object, virtual image

u = -∞ -2f -f -(f/2) -(f/4) 0 +f +2f +∞ V = +f +2f +∞ -f -(f/3) 0 +f/2 +2f/3 +f

(b) Concave lens:

v virtual object, Real image

2f

f

- ∞←u -2f -f 0 f 2f →u u→+∞

Real object, virtual image -f virtual object, real image

-2f

u = -∞ -2f -f -(f/2) 0 +f/2 +f +f +2f +∞ V = -f -2f/3 -f/2 -f/3 0 +f +∞ -∞ -2f -f

Page 181: Geometrical Optics Book With Special Lecture

ANUJ PHYSICAL SCIENCES PHYSICS FOR IIT-JEE /AIEEE/AIPMT

Office: 128/652 “Y” Block Kidwai Nagar Kanpur -11 UP, Website: www.apskanpur.in Mob: 09795647531, 09889787531, 0848664308 E-mail:[email protected]

181

Analysis of graphs:

Convex lens:

3 2 1 4

2F F

v

u<0 R0 2 u>0 v0

v >0 R1 v > 0 v1

+2f

3 +f 4

-2f -f u

u <0 R0 u > 0 V0

v <0 V1 v < 0 V1

1

Region 1: object lies between optical centre and focus real object, virtual image ∣v∣ >

∣u∣.

m = v/u thus ∣m∣ > 1 image is enlarged is u < 0: v < 0 thus m >0 image is erect. As object

moves from focus towards optical centre magnification goes on decreasing.

Region 2: object lies between F and 2F object is real image is also real ∣v∣ > ∣u∣.

∣m∣ < 1 image is enlarged v> 0; u<0, m < 0 image is inverted as object moves from F to

2F magnification decreases and equal 1 at 2F.

Region 3: object lies between 2F and infinity object is real and image is also real. ∣v∣ <

∣u∣ ∣m∣ < 1 image is also real. ∣v∣ > ∣u∣

∣m∣ > 1 image is smaller in size v > 0 u < 0, m <0 image is inverted as object moves

farther away magnification goes on decreasing.

Region 4: object is virtual and it lies between optical centre and infinity. Object is

virtual image is real ∣v∣ > ∣u∣.

∣m∣ < 1 image is smaller in size v > 0, u > 0, m>0 image is erect as object moves away

magnification goes on decreasing.

Page 182: Geometrical Optics Book With Special Lecture

ANUJ PHYSICAL SCIENCES PHYSICS FOR IIT-JEE /AIEEE/AIPMT

Office: 128/652 “Y” Block Kidwai Nagar Kanpur -11 UP, Website: www.apskanpur.in Mob: 09795647531, 09889787531, 0848664308 E-mail:[email protected]

182

Analysis of graphs:

Concave lens:

u<0 R0 u >0 V0

v>0 R1 v >0 R1

2

+2+

U

1

u<0 R0 -2f u >0 V0

v>0 V1 v <0 V1

1 2 3 4

2F F F 2F

Region 1: For all the positions of a real object in front of a lens. Object is real image is

virtual ∣v∣ > ∣u∣.

∣m∣ < 1 image is smaller in size v<0, u<0 m>0 image is erect. As object is moved away

from lens image size i.e. magnification goes on decreasing

Region 2: Virtual object that lies between optical centre and focus. Object is virtual

and image is real ∣v∣ > ∣u∣ ∣m∣ > 1 image is enlarged v>0, u>0, m>0 image is erect. As

object is moved towards F image size goes on increasing i.e., m increases.

Region 3: virtual object lies between +F and +2F object is virtual and image is virtual

∣v∣ > ∣u∣ ∣m∣ > 1 image is inverted. As object moves from +F to+2F magnification goes on

decreasing.

Region 4: virtual object lies between 2F and infinity object is virtual and image is

virtual ∣v∣ < ∣u∣, ∣m∣ < 1 image is smaller in size u >0 v<0 m<0 image is inverted.

Page 183: Geometrical Optics Book With Special Lecture

ANUJ PHYSICAL SCIENCES PHYSICS FOR IIT-JEE /AIEEE/AIPMT

Office: 128/652 “Y” Block Kidwai Nagar Kanpur -11 UP, Website: www.apskanpur.in Mob: 09795647531, 09889787531, 0848664308 E-mail:[email protected]

183

Problem 10(AIEEE 2009): In an optics experiments, with the position of the object

fixed, a student varies the position of a convex lens and for each position; the screen is

adjusted to get a clear image of the object. A graph between the object distance u and

the image distance v, from the lens, is plotted using the same scale for the two axes. A

straight line passing through the origin and making an angle of 450 with the x-axis

meets the experimental curve at p. The coordinates of p will be

(a) (2f, 2f) (b) ((f/2),(f/2)) (c) (f, f) (d) (4f, 4f)

Solution:

V

V

U u(u>f)

It is possible when object kept at centre of curvature u=v,

u= 2f, v=2f.

Problem 11(IIT- JEE 2006): The graph between object distance u and image

V

+11

+10

+9 450

-9 - 10 - 11 u

Distance v for lens is given. The focal length of the lens is

(a) 5 ± 0.1 (b) 5 ± 0.05 (c) 0.5 ± 0.1 (d) 0.5 ± 0.05

Solution: from the lens formula: 1/f = (1/v) – (1/u) we have

1/f = (1/10) – (1/-10) or f = +5

further Δu = 0.1 and Δv = 0.1 (from the graph)

now differentiating the lens formula we have,

(Δf/f2) = (Δv/v2)+( Δu/u2) or Δf = [(Δv/v2)+( Δu/u2)]f2

substituting the value we have Δf = [(0.1/102)+(0.1/102)] (5)2 = 0.05

so f ± Δf = 5±0.05

Page 184: Geometrical Optics Book With Special Lecture

ANUJ PHYSICAL SCIENCES PHYSICS FOR IIT-JEE /AIEEE/AIPMT

Office: 128/652 “Y” Block Kidwai Nagar Kanpur -11 UP, Website: www.apskanpur.in Mob: 09795647531, 09889787531, 0848664308 E-mail:[email protected]

184

Measurement of refractive index of liquid by a convex lens:

I O I O

F0 F

(a) (b)

fig. Shows an equviconvex lens placed on a plane mirror. An object pin is moved up and

down. When the pin lies at the focus of the lens, there is no parallax between the object

and the image. When a liquid whose refractive index is to be obtained is placed between

a plane mirror and a convex lens, the object pin O has to be shifted downward so that no

parallax exists between it and its image. The position of object O from the lens is now

equal to the combined focal length of ‘lens and liquid’ combination. If f is the focal length

of the liquid lens, then the combined focal length is given by

(1/F) = (1/f0) + (1/f)

The liquid lens is Plano -concave type as its lower surface is the plane surface of the

mirror and the upper surface is the curved surface of the convex lens, then

(1/f) = (μ-1) [(1/-R) – (1/∞)]

or (1/f) = -[(μ-1)/R]

Thus, μ = 1+[R/f]

Formula related two combinations of thin lenses:

If lenses are placed in contact

Net Deviation: δnet = δ1 + δ2 + δ3 + . . .

Net Power: Pnet = P1 + P2 + P3

Net Magnification: mnet = m1 m2 m3. . . If many steps are involved

Magnification = (I1/O)(I2/I1)(I3/I2). . . (I/) = I/O

If two lenses are placed at a separation d apart

Pnet = P1 + P2 – dP1P2

1/feq =1/F1 + 1/F2 – d (1/F1F2)

Page 185: Geometrical Optics Book With Special Lecture

ANUJ PHYSICAL SCIENCES PHYSICS FOR IIT-JEE /AIEEE/AIPMT

Office: 128/652 “Y” Block Kidwai Nagar Kanpur -11 UP, Website: www.apskanpur.in Mob: 09795647531, 09889787531, 0848664308 E-mail:[email protected]

185

Problem 12(AIEEE 2007): Two lenses of Power - 15D and + 5D are in contact with

each other. The focal length of the combination is

(a) -20 cm (b) -10 cm (c) + 20 cm (d) + 10 cm

Solution: Power of a lens is reciprocal of its focal length. Power of combined lens is

p = p1+p2 = - 15 + 5 = -10 D so f = 1/p = 100/(-10) cm

f = - 10 cm

Problem 13(IIT – JEE 2005): A convex lens is in contact with concave lens. The

magnitude of the ratio of their focal length is 2/3. Their equivalent focal length is 30 cm.

What are their individual focal lengths?

(a) -75, 50 (b) -10, 15 (c) 75, 50 (d) -15, 10

Solution: Let focal length of convex lens is +f then focal length of concave lens would

be (-3/2) f.

1/Fnet = 1/F1 + 1/F2

from the given condition.,

1/30 = (1/f ) – (2/3f) = 1/3f

f = 10 cm

Therefore, focal length of convex lens=+10 cm and that of concave lens

= -15 cm.

Problem 14(IIT- JEE 2010): A biconvex lens of focal length 15 cm is in front of a

plane mirror. The distance between the lens and the mirror is 10cm. A small object is

kept at a distance of 30 cm from the lens. The final image is

(a) virtual and at a distance of 16 cm from the mirror

(b) real and at a distance of 16 cm from the mirror

(c)virtual and at a distance of 20 cm from the mirror

(d) none of the above

Solution:

10 cm

o l2 l3 l1

6cm

30 cm 10cm 20cm

Object is placed at distance 2f from the lens. So first image I will be formed at distance 2f

on other side. This image I1 will behave like a virtual object for mirror. The second

image I2 will be formed at distance 20 cm in front of the mirror.

Page 186: Geometrical Optics Book With Special Lecture

ANUJ PHYSICAL SCIENCES PHYSICS FOR IIT-JEE /AIEEE/AIPMT

Office: 128/652 “Y” Block Kidwai Nagar Kanpur -11 UP, Website: www.apskanpur.in Mob: 09795647531, 09889787531, 0848664308 E-mail:[email protected]

186

Problem 15(IIT-JEE 2006): A biconvex lens of focal length f forms a circular image

of radius r of sun in focal plane. Then which option is correct?

(a) πr2 =f

(b) πr2 =f2

(c) if lower half part is covered by black sheet, then area of the image is equal to πr2 /2

(d) if f is doubled, intensity will increase

Solution:

r = f tan θ

or r=f

so πr2 =f2

Problem 16: A reflecting surface is represented by the equation x2 +y2 = a2. A ray

travelling in negative x direction is directed towards positive y direction after refraction

from the surface at point P. Then the coordinates of point P are

(a) (0.8a, 0.6a) (b) (0.6a, 0.8a) (c) [(a/√2), (a/√2)]

(d) none (e) [(a/2), (a/2)]

Solution:

Y

P

x

y a

450

x

X = a/√2, y = a/√2

x = a cos 450 ⇒ a/√2 = x

y = a cos 450 ⇒ a/√2 = y

Hence coordinates [(a/√2), (a/√2)]

Page 187: Geometrical Optics Book With Special Lecture

ANUJ PHYSICAL SCIENCES PHYSICS FOR IIT-JEE /AIEEE/AIPMT

Office: 128/652 “Y” Block Kidwai Nagar Kanpur -11 UP, Website: www.apskanpur.in Mob: 09795647531, 09889787531, 0848664308 E-mail:[email protected]

187

Problem 17: A glass slab of thickness 3 cm and refractive index 1.5 is placed in front

of a concave mirror of focal length 20 cm. Where should a point object be placed if it is

to image on to itself? The glass slab and the concave mirror are shown in fig.

Slab

R

Solution:

Let the distance of the object from the mirror be x.

We know that the slab simply shifts the object.

The shift being equal to

s = t[1-(1/μ)] = 1 cm

The direction of shift is towards the concave mirror.

∴ The apparent distance of the object from the mirror is (x-1)

If the rays are to retrace their paths, the object should appear to be at the centre of

curvature of the mirror.

∴ (x-1) = 2f = 40 cm

or x = 41 cm from the mirror

Page 188: Geometrical Optics Book With Special Lecture

ANUJ PHYSICAL SCIENCES PHYSICS FOR IIT-JEE /AIEEE/AIPMT

Office: 128/652 “Y” Block Kidwai Nagar Kanpur -11 UP, Website: www.apskanpur.in Mob: 09795647531, 09889787531, 0848664308 E-mail:[email protected]

188

Problem 18:

20 cm

S P

60cm

A point source of light is placed 60 cm away from screen. Intensity detected at point P is

I. Now a diverging lens of focal length 20 cm is placed 20 cm away from S between S and

P. The lens transmits 75% of light incident on it. Find the new value of intensity at P.

Solution:

S r1 r2

10 cm

u = -20, f = -20,

gives v = -10

Let P = power of source

I = P/4π(60)2

Energy received by lens E2 = [P/4π(20)2]A1

∴ I2 = 0.75E2/A2

From similar triangles A2/A1 = 25 ∴ I2 = 0.271

Page 189: Geometrical Optics Book With Special Lecture

ANUJ PHYSICAL SCIENCES PHYSICS FOR IIT-JEE /AIEEE/AIPMT

Office: 128/652 “Y” Block Kidwai Nagar Kanpur -11 UP, Website: www.apskanpur.in Mob: 09795647531, 09889787531, 0848664308 E-mail:[email protected]

189

Lecture-11

Geometrical Optics

Physics for IIT - JEE

Refraction from curved surfaces:

Cut lense

Silvered lenses

Combination of lenses & mirrors

Page 190: Geometrical Optics Book With Special Lecture

ANUJ PHYSICAL SCIENCES PHYSICS FOR IIT-JEE /AIEEE/AIPMT

Office: 128/652 “Y” Block Kidwai Nagar Kanpur -11 UP, Website: www.apskanpur.in Mob: 09795647531, 09889787531, 0848664308 E-mail:[email protected]

190

Cut lens:

Depending upon how a lens is cut its behaviour gets changes in such a way that

either its power or brightness of image or both get changed.

Generally a lens is cut by a plane surface (It can be cut by curved surface as well)

in two possible ways.

(A) By a plane parallel to principle axis:

Power, focal length no change

= R R

Intensity, brightness change

In this situation power and focal length of the lens do not change (As radius of curvature

of both refracting surface & refractive index do not change) but amount of light energy

passing through lens decreases that is why brightness or intensity of image gets

reduced.

Problem 1: A convex lens of focal length f is cut by a plane parallel to principle axis.

The pieces are then placed side by side in such a way that light passes through both the

pieces. Find the power of such a combination.

Solution: As a focal length of both the pieces remain same as a whole

= +

f f f

Peff = 1/feff = (1/f) + (1/f) = (2/f)

Page 191: Geometrical Optics Book With Special Lecture

ANUJ PHYSICAL SCIENCES PHYSICS FOR IIT-JEE /AIEEE/AIPMT

Office: 128/652 “Y” Block Kidwai Nagar Kanpur -11 UP, Website: www.apskanpur.in Mob: 09795647531, 09889787531, 0848664308 E-mail:[email protected]

191

(B) By a plane perpendicular to its principle axis:

Power, focal change

= R R=∞ (change)

Intensity, brightness no change

In this situation power & focal length of the lens gets change

(As radius of curvature of one of the refracting surface changed)

Position nature & size of image may get changed.

But amount of light energy passing through remains unchanged and that is why

brightness of image remains same.

Problem 2: A convex lens of focal length f is cut by a plane perpendicular to principle

axis. The pieces are then placed side by side. In such a way that light passes through

both the pieces. Find the power of such a combination.

Solution:

= +

As power of each half becomes half or the focal length is twice so

Peff = 1/feff = [(1/2f) + (1/2f)]= 1/f

Page 192: Geometrical Optics Book With Special Lecture

ANUJ PHYSICAL SCIENCES PHYSICS FOR IIT-JEE /AIEEE/AIPMT

Office: 128/652 “Y” Block Kidwai Nagar Kanpur -11 UP, Website: www.apskanpur.in Mob: 09795647531, 09889787531, 0848664308 E-mail:[email protected]

192

Problem 3: A lens of focal length f is cut in to four pieces and placed as shown. Find

the power of combination.

a) Peff = 1/f

b) Peff = 2/f

c) Peff = 3/f

d) Peff = 5/f

1 2 3 4

Solution:

(1/f) = (μ-1)[(1/R1) – (1/R2)] for convex

(1/f) = (μ-1)[(1/R1) + (1/R2)]

(1/f1) = (μ-1)[(1/R1) – (1/∞)]

(1/f1) = (μ-1)/R1 = 1/f3 (1&3 are at ⏊ to principle axis radii of )

(1/f2) = (μ-1)[(1/R1) + (1/R2)]

(1/f2) = (μ-1)[(1/∞) - (1/-R2)]

(1/f2) = (μ-1)/R2 = 1/f4

Peff = 1/feff = (1/f1)+( 1/f2)+( 1/f3)+( 1/f4)

Peff = [(2/f1) + (2/f2)]

Peff = 2[(1/f1) + (1/f2)]

Peff = 2[[(μ-1)/R1]+[(μ-1)/R2]]

Peff = 2 (μ-1) [(1/R1) + (1/R2)]

Peff = 2/f

Page 193: Geometrical Optics Book With Special Lecture

ANUJ PHYSICAL SCIENCES PHYSICS FOR IIT-JEE /AIEEE/AIPMT

Office: 128/652 “Y” Block Kidwai Nagar Kanpur -11 UP, Website: www.apskanpur.in Mob: 09795647531, 09889787531, 0848664308 E-mail:[email protected]

193

Problem 4: A point object is placed at a distance of 30 cm along the principle axis of a

convex lens of focal length 20 cm & aperture 4 cm. It is cut by a plane parallel to its

principle axis and pieces are placed inverted as shown. Find distance between the

images formed

(a) 4 cm (b) 8 cm (c) 12 cm (d) none

f = 20 cm

R = 4 cm

O u = 30 cm

30 cm

Solution:

(1/f) = (1/v) + (1/u) Convex

(1/20)= (1/v) + (1/30) ⇒ (1/20) - (1/30) = (1/v)

(1/v) = (1/10)[(3-2)/6]= 1/60

V = 60 cm

m = v/u = 60/-30 = -2

4 cm

2cm

O

30 cm 2 cm

4 cm

so images are formed by each half, with twice magnification and as they are real, they

are formed 2*2 = 4 cm away from their respective principle axis.

m = m1, m2, m3, ------ magnification multiplied.

Thus distance between the image is 4 cm + 4 cm + 4 cm ⇒ d = 12 cm

Page 194: Geometrical Optics Book With Special Lecture

ANUJ PHYSICAL SCIENCES PHYSICS FOR IIT-JEE /AIEEE/AIPMT

Office: 128/652 “Y” Block Kidwai Nagar Kanpur -11 UP, Website: www.apskanpur.in Mob: 09795647531, 09889787531, 0848664308 E-mail:[email protected]

194

Problem 5: An equviconvex lens of focal length f is cut into four pieces by two planes

as shown and the pieces are places as shown and the pieces are placed as shown in

column I. Then match them with column II

Column I Column II

(a) p) f/2

(b) q) (2/3)f

(c) r) f

(d) s) None

Solution: focal length of each part becomes 2f

(a) 1/feq = 1/2f + 1/2f = 2/2f = 1/f

⇒ feq = f a → r

(b) 1/feq = 1/2f + 1/2f + 1/2f = 3/2f

⇒ feq = (2/3)f b → q

(c) 1/feq = 1/2f + 1/2f + 1/2f + 1/2f = 4/2f = (2/f)

⇒ feq = f/2 c → p

(d) As a paraxial ray for one lens is merging for other so focal length of such a

combination cannot be calculated. d → s

Column I Column II

(a) p) f/2

(b) q) (2/3)f

(c) r) f

(d) s) None

Page 195: Geometrical Optics Book With Special Lecture

ANUJ PHYSICAL SCIENCES PHYSICS FOR IIT-JEE /AIEEE/AIPMT

Office: 128/652 “Y” Block Kidwai Nagar Kanpur -11 UP, Website: www.apskanpur.in Mob: 09795647531, 09889787531, 0848664308 E-mail:[email protected]

195

Combination of cut lenses problems:

Problem 6: A liquid of μe = 1.62 is placed between two Plano convex identical lenses

of μ2 = 1.54. Two possible arrangements P & Q are shown. The system is

(a) divergent in P

(b) convergent in P

(c) divergent in Q

(d) convergent in Q

P Q

Solution:

For lens L:

1/FL = (1.54-1)[(1/R) – (1/-R)] = 1.08/R

FL =R/1.08

For P:

(1/Fl) = (1.62-1) ][(1/-R)-[1/+R]] = -1.24/R

fl = -R/1.24

1/fp = (1/fl) +(1/FL) = [(1.08/R)- (1.24/R) = -(0.16/R)

fp = -R/0.16

For Q:

1/fl’ = (1.62-1) [(-1/R)- 1/∞] = -(0.62/R)

1/fQ = 1/fl’ +1/FL = (-0.62/R) + (1.08/R) = 0.46/R

fQ = R/0.46

As focal length in P is negative & in Q is positive, so system is divergent in P and

convergent in Q.

Page 196: Geometrical Optics Book With Special Lecture

ANUJ PHYSICAL SCIENCES PHYSICS FOR IIT-JEE /AIEEE/AIPMT

Office: 128/652 “Y” Block Kidwai Nagar Kanpur -11 UP, Website: www.apskanpur.in Mob: 09795647531, 09889787531, 0848664308 E-mail:[email protected]

196

Problem 7: Two Plano concave lenses of glass of refractive index 1.5 have radii of

curvature of 20 & 30 cm. They are placed in contact with curved surfaces towards each

other & the space between them is filled with a liquid of refractive index 4/3. Find the

focal length of the system

(a) divergent lens of focal length 72cm (b) divergent lens of focal length 60cm

(c) convergent lens of focal length 72cm (d) convergent lens of focal length 60cm

Solution:

4/3

1 3

2

The system is equivalent to the combination of three thin lenses.

(1/f) = (1/f1) + (1/f2) + (1/f3)

but by lens maker’s formula 1/f = (μ -1)[(1/R1) – (1/R2)]

1/f1 = (1.5 -1)[(1/∞) – (1/+20)] = -1/40

1/f2 = (4/3 -1)[(1/20) + (1/30)] = 5/180

1/f3 = (1.5 -1)[(1/-∞) – (1/30)] = -1/60

(1/f) = (-1/40) + (5/180) –(1/60) ⇒ (1/f) = [(-9 +10-6)/360]

f = -72 cm i.e. the system will behave as a divergent lens of focal length 72 cm.

Problem 8(IIT 1985)(MNR 1992): A convex lens A of focal length 20 cm & a concave

lens B of focal length 5 cm are kept along the same axis with a distance d between them.

What is the values of d if a parallel beam of light incident on A leaves B as a parallel

beam

(a) 15 cm (b) 30 cm (c) 10 cm (d) none

Solution:

f = 20 cm f = 5 cm

I

A d 5 cm

As the incident beam is parallel in absence of concave lens it will form an image at a

distance v from it such that (1/v) – (1/-∞) = (1/20) ⇒ v = 20 cm = f

Now since d is the distance between convex & concave lens. The distance of image I

from concave lens will be (20-d). Since the image I will act as an object for concave lens

which forms its image at ∞, so (1/v)- (1/u) = (1/f)

(1/∞) – [1/(20-d)] = 1/(-5) , 20-d =5 ⇒ d = 15 cm

Page 197: Geometrical Optics Book With Special Lecture

ANUJ PHYSICAL SCIENCES PHYSICS FOR IIT-JEE /AIEEE/AIPMT

Office: 128/652 “Y” Block Kidwai Nagar Kanpur -11 UP, Website: www.apskanpur.in Mob: 09795647531, 09889787531, 0848664308 E-mail:[email protected]

197

Silvered lenses:

When one side of a lens is silvered such that it effectively behaves like a

mirror, then it is called a silvered lens.

Silvered lens

⇒ Combination of two lenses & mirror

⇒ Behaves like a mirror

A light ray entering into a silvered lens goes through

(1) Refraction

(2) Reflection

(3) Refraction

That means as bending ray takes place three times deviation

δeff = δ1 + δ2 + δ3

Peff = P1 +P2 +P3 = PL +PM +PL = 2PL +PM

-1/Feff = (2/FL) – (1/FM) Effective behaviour mirror,

PM =-1/f, PL = 1/f

1/Feff = -(2/FL) + (1/FM)

(1)

= + ⇒ δ1 + +

(2) δ2 δ3

(3)

Fig: silvered lens

Page 198: Geometrical Optics Book With Special Lecture

ANUJ PHYSICAL SCIENCES PHYSICS FOR IIT-JEE /AIEEE/AIPMT

Office: 128/652 “Y” Block Kidwai Nagar Kanpur -11 UP, Website: www.apskanpur.in Mob: 09795647531, 09889787531, 0848664308 E-mail:[email protected]

198

Working procedure for problems based on silvered lenses:

Step 1st:

Formula 1/Feff = (-2/FL) + (1/FM)

Step 2nd:

FM = R/2

(please take proper sign)

concave mirror → -R, convex mirror → +R, plane mirror → ∞

1/FM = 2/R

Step 3rd:

1/FL = (μ-1)[(1/R1)-(1/R2)]

for convex R1 → +ive, R2 → -ive

for concave R1 → -ive, R2 → +ive

R1 R2 R1 R2

+ - - +

Step 4th:

Use 1/FL and 1/FM in eq.

1/Feff = (1/FM) - (2/FL)

then find Feff → this is mirror

if Feff → +ive → convex mirror diverging

if Feff → -ive → convex mirror converging

Page 199: Geometrical Optics Book With Special Lecture

ANUJ PHYSICAL SCIENCES PHYSICS FOR IIT-JEE /AIEEE/AIPMT

Office: 128/652 “Y” Block Kidwai Nagar Kanpur -11 UP, Website: www.apskanpur.in Mob: 09795647531, 09889787531, 0848664308 E-mail:[email protected]

199

Problem 9: Find focal length & overall nature of the following silvered lenses. In all

cases refractive index of material of lens is greater than that of surroundings.

R R R ∞ R 2R R R R ∞ 2R R

(a) (b) (c) (d) (e) (f)

Answer: (a) feff = -[R/2(2μ-1)] (b) feff = -[R/2(μ-1)] (c) feff = [R/(2-μ)]

(d) feff = [R/2(2μ-1)] (e) feff = [R/2(μ-1)] (f) feff = -[R/(μ+1)]

Solutions:

(a)

= +

R

R R R R

1/Feff = (1/FM) – (2/FL), FM = R/2, FL =(μ-1)[(1/R1) - (1/R2)]

FM = -2/R

1/FL =(μ-1)[(1/R1) - (1/R2)]= (μ-1)[(1/R) - (1/-R)] = (μ-1)(2/R)

1/Feff = (1/FM) – (2/FL) = (-2/R)-2[2(μ-1)/R]= -2/R[1+2(μ-1)]

1/Feff = -2/R[2μ-1]

Feff = R/-2[2μ-1]

Feff →-ive silvered lens behaves like concave mirror & nature is converging.

Page 200: Geometrical Optics Book With Special Lecture

ANUJ PHYSICAL SCIENCES PHYSICS FOR IIT-JEE /AIEEE/AIPMT

Office: 128/652 “Y” Block Kidwai Nagar Kanpur -11 UP, Website: www.apskanpur.in Mob: 09795647531, 09889787531, 0848664308 E-mail:[email protected]

200

(b)

= +

R R R ∞ ∞

1/FL =(μ-1)[(1/R1) - (1/R2)]= (μ-1)[(1/R) - (1/-∞)] = (μ-1)/R

FM = R/L = ∞, 1/FM = 0

1/Feff = (1/FM) – (2/FL) = 0 -2[(μ-1)/R] = -2(μ-1)/R

Feff = -R/[2μ-1]

Hence silvered lens behaves like concave mirror & nature is diverging

(c)

= +

R 2R R 2R 2R

FM = R/L = 2R/L ⇒1/FM = 1/R

1/FL =(μ-1)[(1/R1) - (1/R2)]= (μ-1)[(1/+R) - (1/2R)] = (μ-1)/2R

1/Feff = (1/FM) – (2/FL) = (1/R)-2[(μ-1)/2R]= (1/R)[1-2(μ-1)/2] = (1/R)[1-μ+1]

= (2-μ)/R ⇒ Feff = [R/(2-μ ]

(d)

= +

R ∞ R ∞ R

FM = R/L = +R/L, 1/FM = 2/R

Page 201: Geometrical Optics Book With Special Lecture

ANUJ PHYSICAL SCIENCES PHYSICS FOR IIT-JEE /AIEEE/AIPMT

Office: 128/652 “Y” Block Kidwai Nagar Kanpur -11 UP, Website: www.apskanpur.in Mob: 09795647531, 09889787531, 0848664308 E-mail:[email protected]

201

1/FL =(μ-1)[(1/R1) - (1/R2)]= (μ-1)[(1/-R) - (1/R)] = -2(μ-1)/R

1/Feff = (1/FM) – (2/FL) = 2/R-2[-2(μ-1)/R] = 2/R[1+2(μ-1)]=(2/R)[1+2μ-2]

=(2/R)[2μ-1]

Feff = [R/2(2μ-1)]

focal length of silvered lens is +ive hence its behaves like a convex mirror or its

nature will be diverging.

(e)

= +

R ∞ R ∞ ∞

FM = R/L = ∞/2 = ∞, 1/FM = 0

1/FL =(μ-1)[(1/R1) - (1/R2)]= (μ-1)[(1/-R) - (1/∞)] = -(μ-1)/R

1/Feff = (1/FM) – (2/FL) = 0 -2[-(μ-1)/R] = 2(μ-1)/R

Feff = [R/2(μ-1)]

focal = +ive, hence silvered lens behaves like convex mirror or its nature will be

diverging.

(f)

= +

2R R 2R R 2R

FM = R/2 = -R/2 ⇒ 1/FM = -2/R

1/FL =(μ-1)[(1/R1) - (1/R2)]= (μ-1)[(1/-2R) - (1/-R)] = [(μ-1)/R][(-1/2)+1]

= (μ-1)/2R

1/Feff = (1/FM) – (2/FL) = -2/R -2[(μ-1)/2R] =-2/R[1+(μ-1)/2]

Feff = [-R/(μ-1)]

hence the focal length of the silvered lens is –ive it behaves like concave mirror or

converging

Feq = -R/ μ+

Page 202: Geometrical Optics Book With Special Lecture

ANUJ PHYSICAL SCIENCES PHYSICS FOR IIT-JEE /AIEEE/AIPMT

Office: 128/652 “Y” Block Kidwai Nagar Kanpur -11 UP, Website: www.apskanpur.in Mob: 09795647531, 09889787531, 0848664308 E-mail:[email protected]

202

Problem 10: Find focal length and nature of a silvered equviconvex lens of radius of

curvature 20 cm & refractive index 2 placed in a medium of refractive index 3.

(a) +30 cm, silvered lens will be like a concave mirror or converging

(b) +30 cm, silvered lens will be like a convex mirror or diverging

(c) +20 cm, silvered lens will be like a concave mirror or converging

(d) +20 cm, silvered lens will be like a convex mirror or diverging

Solution:

3 3

= +

20 cm 10 cm 20 cm 20 cm 20 cm

concave

2 2

FM = R/2 = -20/2 =-10cm

1/FL =(μ-1)[(1/R1) - (1/R2)]

1/FL = [(μL/μM)-1][(1/R1) - (1/R2)]

1/FL = [(2/3)-1][(1/20)- (1/-20)]

1/FL= (-1/3)(1/10) = -1/30

1/Feff = (1/FM) – (2/FL)

1/Feff = (-1/10)-2(-1/30)

Feff = -30cm

i.e. silvered lens will be like a concave mirror of focal length 30 cm i.e. nature will be

converging.

Page 203: Geometrical Optics Book With Special Lecture

ANUJ PHYSICAL SCIENCES PHYSICS FOR IIT-JEE /AIEEE/AIPMT

Office: 128/652 “Y” Block Kidwai Nagar Kanpur -11 UP, Website: www.apskanpur.in Mob: 09795647531, 09889787531, 0848664308 E-mail:[email protected]

203

Problem 11: Half part of a equviconvex glass lens L (μ =3/2) of radius of curvature

40 cm is silvered at right side. A plane mirror M and an object O is placed ⏊ to the

principle axis, as shown at the indicated positions. Image formed by silvered lens

and plane mirror has no parallel x but size of image formed by silvered is one third

of that formed by plane mirror. Find the distance of plane mirror a & silvered lens b

from the object.

(a) a = 40/3 cm & b = 20 cm (b) a = 20 cm & b = 40/3 cm

(c) a = 20/3 cm & b = 20 cm (d) a = 40/3 cm & b = 10 cm

b

O

a M L

Solution:

b

O I

a a

From the problem 9(d) FL = R/2(2μ-1) = 40/2[2(3/2)-1] = 10 cm (+convex)

It is given in problem image formed by silvered lens & plane mirror has no parallel but

size of image formed by silvered is (1/3) of that formed by plane mirror. So image

formed by both mirrors coincide.

So for convex mirror u =-b & v = (2a-b)

MS = -(v/u) = -(2a-b)/-b = (2a-b)/b

so for plane mirror MP = 1

MS/MP = [Is/O]/[ Ip/O]= IS/Ip = (Ip/3)/Ip = 1/3

MS = (1/3) MP= (1/3)1

MS = (1/3) = (2a-b)/b ⇒ (1/3) = (2a-b)/b ⇒ b = 6a-3b ⇒ 6a = 4b

a = (2/3)b

(1/f) = (1/v) +(1/u) = 1/(2a-b) + 1/(-b) = 1/10 ⇒ 1/[2(2/3)b-b]-(1/b)]= 1/10

⇒ b = 20 cm, a = (2/3)b = 40/3 cm

Page 204: Geometrical Optics Book With Special Lecture

ANUJ PHYSICAL SCIENCES PHYSICS FOR IIT-JEE /AIEEE/AIPMT

Office: 128/652 “Y” Block Kidwai Nagar Kanpur -11 UP, Website: www.apskanpur.in Mob: 09795647531, 09889787531, 0848664308 E-mail:[email protected]

204

Problem 12: A thin equviconvex lens (μ=3/2) of radius of curvature 10 cm is placed

in contact with a concave mirror of radius of curvature 15 cm and the space between

them is filled with water. Find the focal length and overall combination of nature.

(a)Concave mirror of focal length 22.5 cm

(b) Convex mirror of focal length 22.5 cm

(c) Concave mirror of focal length 18 cm

(d) Concave mirror of focal length 7.5 cm

R=10cm equviconcave lens

4 μ = 3/2

LL μ= 4/3 water R =15 cm concave mirror

Solution:

1/FL =(μ-1)[(1/R1) - (1/R2)]

1/FL =[(3/2)-1][(1/-10)-(1/10)]

1/FL = (1/2)(-2/10)= -1/10 cm

FL = -10cm

1/FL =[(4/3)-1][(1/10)-(1/-15)]

1/FL = (1/3)[(3+2)/30]=(1/3)(5/30)=1/18

FL = 18 cm

FM= -R/2 concave mirror

FM= -15/2

1/Feff = (1/FM) – (2/FL1)-( 2/FL2)

1/Feff = -(2/15)-2(-1/10)-2(1/18)

1/Feff = -2[(1/15)-(1/10)+(1/18)]

1/Feff = -2[(6-9+5)/90]= (-2*2)/90 = -4/90

Feff = -90/4 = -22.5 cm

Feff = -22.5 cm,

i.e. overall combination behaves as a concave mirror or convergent nature.

Page 205: Geometrical Optics Book With Special Lecture

ANUJ PHYSICAL SCIENCES PHYSICS FOR IIT-JEE /AIEEE/AIPMT

Office: 128/652 “Y” Block Kidwai Nagar Kanpur -11 UP, Website: www.apskanpur.in Mob: 09795647531, 09889787531, 0848664308 E-mail:[email protected]

205

Problem 13: A thin equviconvex glass lens (μ = 1.5) is placed on a plane mirror. When

the space between lens and mirror filled with water μ = 4/3, then image is found to

coincide with object at 15 cm above the lens on its principle axis. When water is

replaced by another liquid, It is found that image coincides with object at 25 cm above

the lens. Find the refractive index of the other liquid.

(a) μ = 1.6 (b) μ = 1.5 (c) μ = 1.3 (d) μ = 9/8

Solution:

L1 μ = 1.5 water μ = 4/3

For equviconvex glass lens (1/fL1) =(μ-1)[(1/R1) - (1/R2)]

(1/fL1) =(1.5-1)[(1/R) - (1/-R)] = (1/2)(2/R) = 1/R

fL1 =R

For Plano concave water lens:

(1/fL2) =(μ-1)[(1/R1) - (1/R2)] = [(4/3)-1][(1/-R)- (1/∞)] = -1/3R

fL2 = -3R

FM = R/2 = ∞, FM = R/2 = ∞

1/Feff = (1/FM) – (2/FL1)-( 2/FL2) = (1/∞)- (2/R) – (2/-3R) = (2/3R) –(2/R)

= (2/R)[(1/3)-1] = -4/3R

Feff = -3R/4 i.e. a concave mirror

In case of concave mirror, object & its image coincide at the centre of curvature.

-15 cm = radius of curvature

fe =+R/2 ⇒ Radius of curvature = -2 focal

-15 = -2 feff = +2[-3R/a]

-15 = -(3/2)R ⇒ R = 10 cm for combination

Now for Plano concave liquid lens

1/F’L2 = (μ-1)[(1/R1) - (1/R2)]

1/F’L2 = (μ-1)[(-1/R) - (1/∞)] = -( μ-1)/R

Now 1/F’eff = (1/FM) – (2/FL1)-( 2/F’L2) = (1/∞)-(2/R) – 2[-(μ-1)/R]

= (-2/R) +2[(μ-1)/R] = 2/R[-1+μ-1] = 2(μ-2)/R= (2μ-4)/R

F’eff = R/(2μ-4) = -R/(4-2μ)

Focal length = Radius of curvature /2

+R/(2μ-4) = -25/2 (now image coincides object at a distance 25 cm)

10/(2μ-4) = -25/2

-20 = 50μ -100, -20 +100 = 50μ

80 /50 = μ

μ = 6

Page 206: Geometrical Optics Book With Special Lecture

ANUJ PHYSICAL SCIENCES PHYSICS FOR IIT-JEE /AIEEE/AIPMT

Office: 128/652 “Y” Block Kidwai Nagar Kanpur -11 UP, Website: www.apskanpur.in Mob: 09795647531, 09889787531, 0848664308 E-mail:[email protected]

206

Problem 14 (IIT – JEE 2006):

20 cm

A point object is placed at distance of 20 cm from a thin Plano convex lens of focal

length 15 cm. The plane surface of the lens is now silvered.

The image created by the system is at

(a) 60 cm to the left of the system

(b) 60 cm to the right of the system

(c) 12 cm to the left of the system

(d) 12 cm to the right of the system

Solution:

Refraction from lens :

(1/v)-(1/u) = 1/f

(1/v1)-(1/-20) = 1/15

v1 = 60 cm +ive direction

i.e. first image is formed at 60 cm to the right of lens system.

Reflection from mirror:

After reflection from the mirror, the second image will be formed at a distance of

v2 =60 cm to the left of lens system.

Refraction from lens:

(1/v)-(1/u) = 1/f

(1/v3) – (1/60) = (1/15) +ive direction

v3 = 12 cm

Therefore, the final image is formed at 12 cm to the left of the lens system.

Page 207: Geometrical Optics Book With Special Lecture

ANUJ PHYSICAL SCIENCES PHYSICS FOR IIT-JEE /AIEEE/AIPMT

Office: 128/652 “Y” Block Kidwai Nagar Kanpur -11 UP, Website: www.apskanpur.in Mob: 09795647531, 09889787531, 0848664308 E-mail:[email protected]

207

Problem 15 :( Based on silvered lens): A thin Plano convex lens (μ = 5/4) fits exactly into a Plano concave lens μ = 3/2. The radius of curvature of the curved interface

is 30 cm.

5/4

L2

L1

3/2

(1) If plane surface of Plano convex lens is silvered, then the

(a) System behaves like a concave mirror

(b) System behaves like a convex mirror

(c) Focal length of the system is 45 cm

(d) Focal length of the system is 60 cm

(2) An object of height 5 cm is placed at a distance of 15 cm from equivalent mirror of

the previous problem. Then transverse magnification produced by the system is.

(a) -4/5 (b) + 4/5 (c) -5/4 (d) +5/4

Solution:

1/f1 = [(3/2)-1][(1/∞)- (1/30)] = -1/60

1/f2 = [(5/4)-1][(1/30)- (1/∞)] = 1/120

fM = ∞

1/feq = (1/FM) – (2/F1)-( 2/F2)= (1/∞) –(2/-60)-(2/120) = (1/30)- (1/60) = 1/60

feq = +60cm

So it behaves like a convex mirror

(1/v) +(1/u) = (1/f)

(u/v) +(u/u) = (u/f)

(-1/m) +1 = (u/f)

(1/m) = 1-(u/f)

(1/m) = (f-u)/f

m = f/(f-u) = -f/(u-f) m = -(v/u) =-f/(u-f)

m =-60/(-15-60) =

m =--60/-75 = +4/5

Page 208: Geometrical Optics Book With Special Lecture

ANUJ PHYSICAL SCIENCES PHYSICS FOR IIT-JEE /AIEEE/AIPMT

Office: 128/652 “Y” Block Kidwai Nagar Kanpur -11 UP, Website: www.apskanpur.in Mob: 09795647531, 09889787531, 0848664308 E-mail:[email protected]

208

Problem 16: In following four situations, silvered lenses of refractive index 3/2 are

placed in medium of refractive index 1 or 4. Then match the column.

(a) 1 3/2 p) Power is +ive

R R

(b) q) Power is -ive

4 3/2

R R

(c) 1 r) focal length is +ive

R R

(d) 4 s) focal length is –ive

R R

Solution:

For (a) Feq = -R/2(2μ-1) = -R/[2(2(3/2)/1)-1]= -R/4

a→ p, s) F →-ive, p → +ive

For (b) ) Feq = -R/2(2μ-1) = -R/[2(2(3/2)/4)-1]= 2R

b→ q, r) F →+ive, p → -ive

For (c) ) Feq = R/2(2μ-1) = R/[2(2(3/2)-1)]= R/4

c→ q, r) F →+ive, p → -ive

For (d) ) Feq = R/2(2μ-1) = R/[2(2(3/2)/4)-1]= -2R

d→ p, s) F →-ive, p → +ive

Page 209: Geometrical Optics Book With Special Lecture

ANUJ PHYSICAL SCIENCES PHYSICS FOR IIT-JEE /AIEEE/AIPMT

Office: 128/652 “Y” Block Kidwai Nagar Kanpur -11 UP, Website: www.apskanpur.in Mob: 09795647531, 09889787531, 0848664308 E-mail:[email protected]

209

Lecture-12

Geometrical Optics

Physics for IIT - JEE

Optical Instruments:

1. Human eye

2. Defects of vision

3. Simple microscope or Magnifier

4. Compound Microscope

5. Astronomical Telescope

6. Terrestrial Telescope

7. Galilean Telescope

8. Reflecting Telescope:

(a) Newtonian reflecting telescope (b) Cassegranian reflecting telescope

9. Camera

10. Prism Binocular

Page 210: Geometrical Optics Book With Special Lecture

ANUJ PHYSICAL SCIENCES PHYSICS FOR IIT-JEE /AIEEE/AIPMT

Office: 128/652 “Y” Block Kidwai Nagar Kanpur -11 UP, Website: www.apskanpur.in Mob: 09795647531, 09889787531, 0848664308 E-mail:[email protected]

210

1. Human Eye:

Optically eye is like an exceptionally fine camera with an elaborate lens system on one

side and a sensitive screen on the other.

Human eyes are sensitive to a small range of electromagnetic spectrum called visible

region. Visible region consists of wavelength ranging from 4000 Å to 7800 Å.

The colour corresponding to wavelength 400 nm is violet while corresponding to

wavelength780 nm is red, above 780 nm, we get infra-red radiations and below 400 nm,

electromagnetic radiations are called ultraviolet.

Human eyes are not equally sensitive for all wavelengths. The sensitivity varies and it is

asymptotic at both ends. Sensitivity of human eye is defined as the sensation of sight

produced for a given power per unit area of light given wavelength falling on eye.

The variation of relative sensitivity of human eye with the wavelength is shown in fig

Relative

sensitivity

Infra-red

UV

400nm 500nm 700nm

It is clear from the graph that sensitivity of human eye is Maximum for 555nm. This

wavelength corresponds to yellow-green region.

1(a) Construction: It consists of a nearly spherical eye ball having sclerotic; an

opaque, white, tough and fiberous material as the outermost layer. Being tough it

maintains the shape of eye. Inside sclerotic there is a layer charged with black pigment

and is known as choroid, this ensures the absence of to and from reflections inside the

eye ball.

Front portion of the eye is of a transparent material and bulges outward. This is called

cornea. Behind cornea a crystalline lens made of a gelatinous material is held in

position by zonule fibers attached to the ciliary muscles. The lens is more curved on

back side and is made up of such a matter which becomes more and denser towards the

centre. The front side of the lens is partially covered by a diaphragm called iris.

Page 211: Geometrical Optics Book With Special Lecture

ANUJ PHYSICAL SCIENCES PHYSICS FOR IIT-JEE /AIEEE/AIPMT

Office: 128/652 “Y” Block Kidwai Nagar Kanpur -11 UP, Website: www.apskanpur.in Mob: 09795647531, 09889787531, 0848664308 E-mail:[email protected]

211

The aperture of iris is called pupil. The colour of this aperture varies from black to blue

to brown from person-to-person, a layer called retina covers the choroid and is

sensitive to light. Just opposite to the lens, on retina there is a depressed portion called

yellow spot. This is the most sensitive part of retina. The most depressed spot of this

portion is called fovea centrallis. Retina is ultimately connected to optic nerve which is

embedded in nerve canal and goes to brain. Portion of eye in between cornea and lens is

filled with a liquid called aqueous humour while that is between lens and retina is filled

with a liquid called vitreous humor.

1(b) Action of eye: A beam of light coming from the object gets refracted through the

crystalline lens and is focussed on the retina. In retina, the light pulses are received by

tiny cones and rods whose function, it seems to be, to change the light into electricity.

Each cone and rod is connected with an individual nerve which conducts the electricity

through the nerve canal to the brain. How these electric impulses are produced by cell

like structures, the cones and rods and how are they interpreted by brain as vision, is

still only vaguely understood by the scientists. Experiments seems to indicate that cones

respond only to bright light and are particularly responsible for detection and

distinction of colours, whereas rods are sensitive to feeble light, motion and slight

variation in intensity.

Note: A common misconception is that the lens of the eye is what does the focussing.

All the transparent parts of the eye are made of fairly similar stuff. So the dramatic

change in medium is when a ray crosses from the air into the eye (at the outside surface

of the cornea). This is where nearly all the refraction takes place. The lens medium

differs only slightly in its optical properties from the rest of the eye, so very little

refraction occurs as light enters and exits the lens, whose shape is adjusted by muscles

attached to it, is only meant for fine-tuning the focus to form images of near or far

objects.

1(c) Accommodation of eye: In case of eye, images of all the objects situated at

different distances from eye are formed on the retina. Thus, image distance is fixed for

all of them which have different object distance. This is possible if the focal length of the

lens undergoes a variation along with a variation in the distance of object. When eye

does so, it is said to be exerting accommodation.

The ability of eye to bring objects lying at various distances to focus on the retina is

called accommodation.

The ability of eye to change focal length of its lens is called power of accommodation.

This is achieved by changing the radius of curvature of the lens. The ciliary muscle, by

contracting, causes the lens to bulge out. This reduces the focal length of the eye lens

bringing nearly objects to focus on the retina. When ciliary muscles get relaxed, the

Page 212: Geometrical Optics Book With Special Lecture

ANUJ PHYSICAL SCIENCES PHYSICS FOR IIT-JEE /AIEEE/AIPMT

Office: 128/652 “Y” Block Kidwai Nagar Kanpur -11 UP, Website: www.apskanpur.in Mob: 09795647531, 09889787531, 0848664308 E-mail:[email protected]

212

edges of the lens are pulled out, thus tending to flatten it. Under these conditions the

focal length increases, bringing distant objects to focus on retina.

1(d) Distance of distinct vision: The normal eye can exert accommodation power

between 20 cm and infinity. In case of defective eye we define two points, a far point

and a near point. Far point is the farthest point up to which the eye can see clearly. Near

point is the nearest point up to which the eye can see clearly.

In between these two points the lens will be free from all stresses and strains. At this

stage objects lying at particular distance will be focussed on retina. This distance is

called distance of distinct vision. Distance of distinct vision is the distance of object from

the eye such that the eye can see that object clearly without any deformation being

produced in the lens. Distance of distinct vision for a normal eye is 25 cm. That is why,

while reading a book we keep it nearly at a distance of 25 cm from our eye.

The far point of a normal eye is at infinity. The position of the near point changes with

age, on average it is 25 cm.

Age Near point ( cm) 10 7 20 10 30 14 40 22 50 40 60 200

Note: 1.The retina is nearly2.5 cm behind the eye lens, and so maximum focal length of the

eye lens that can be 2.5 cm for relaxed eye.

2.The minimum focal length corresponds to, when object is at near point.

Thus u=-25 cm, v=+2.5 cm

1/f =1/v -1/u = 1/ (2.5) -1/ (-25)

f ≃2.2 cm

1(e) Persistence of vision: Eye possesses a peculiar property called persistence of

vision by virtue of which the impression of the image persists upon retina up to (1/10th)

of a second after the object has been removed. This is the basic property due to which

we can see the continuity of a movie.

1(f) Resolving limits: The minimum angular separation between two objects so that

they are just resolved i.e. eyes are able to see separately is called resolving limit. For

human eye it is 1 minute of arc or (1/60)0.

Page 213: Geometrical Optics Book With Special Lecture

ANUJ PHYSICAL SCIENCES PHYSICS FOR IIT-JEE /AIEEE/AIPMT

Office: 128/652 “Y” Block Kidwai Nagar Kanpur -11 UP, Website: www.apskanpur.in Mob: 09795647531, 09889787531, 0848664308 E-mail:[email protected]

213

2. Defects of Vision:

2(a) Short sightedness or “Myopia”: It is a defect of vision by which a person can see objects situated nearby and cannot see

objects situated far away.

The rays from the far object are focused in front of retina, thus the far point of myopic

eye becomes less than infinity.

Thus the eye suffering from this defect: For myopic eye:

(i) Near point distance is 25 cm.

(ii) Far point distance is less than infinity; may be few meters.

(iii) This defect can be removed by putting a concave lens of suitable focal length before

the eye.

Myopic eye

x

Far point of defective eye

If x is the far point distance of myopic eye (defective eye) and f be the focal length of the

lens required, then by lens formula 1/f=1/v-1/u

1/f =1/(-x) -1/(-∞)

f=-x

Page 214: Geometrical Optics Book With Special Lecture

ANUJ PHYSICAL SCIENCES PHYSICS FOR IIT-JEE /AIEEE/AIPMT

Office: 128/652 “Y” Block Kidwai Nagar Kanpur -11 UP, Website: www.apskanpur.in Mob: 09795647531, 09889787531, 0848664308 E-mail:[email protected]

214

There can be following two reasons for this defect:

(i) Elongation of eye ball: If the eye ball gets elongated, retina gets drifted away.

Principal focus of the lens falls short of retina. This is likely to be produced due to a

blow on head downwards.

(ii) Shortening of focal length of eye lens: This may be due to a change in density of

eye lens or due to deformation in ciliary muscles.

2 (b) Long sightedness or “Hyper-metropia”: It is defect of vision by virtue of which a man can see objects situated at greater

distances from eye and cannot see objects situated nearby.

The image of near object will be formed behind the retina.

Thus the eye suffering from this defect:

(i) Near point distance is greater than 25 cm.

(ii) Far point distance remains as such i.e. infinity.

(iii) This defect can be removed by putting a convex lens of suitable focal length before

the eye.

25 cm

Hypermetropic eye

Page 215: Geometrical Optics Book With Special Lecture

ANUJ PHYSICAL SCIENCES PHYSICS FOR IIT-JEE /AIEEE/AIPMT

Office: 128/652 “Y” Block Kidwai Nagar Kanpur -11 UP, Website: www.apskanpur.in Mob: 09795647531, 09889787531, 0848664308 E-mail:[email protected]

215

Near point of defective eye

25 cm

Y

If y is the near point distance of defective eye and f be the focal length of the lens

required, then by lens formula 1/f=1/v-1/u

1/f =1/(-y) -1/(-25)

⇰ f=25 y/(y-25)

There can be following two reasons for this defect:

(i) Shortening of eye ball: It may happen due to some accident in which face gets a

blow from front or back.

(ii) Increase of focal length of eye lens: This may, again, be due to a change in density

of eye lens or a deformation in ciliary muscles.

2(c) Astigmatism:

It is a defect of vision by virtue of which eye is able to see objects situated in one plane

while it cannot see the objects situated in a perpendicular plane.

If a man, suffering from this defect looks at wire gauze, he will be able to see only one

set of wires. If vertical wires are visible to him, he will not be able to see the horizontal

ones and vice-versa.

A normal eye will see all the lines in fig. Equally black. An astigmatic eye will see

variation in the intensity of the lines.

A diagram for detection of Astigmatism

Page 216: Geometrical Optics Book With Special Lecture

ANUJ PHYSICAL SCIENCES PHYSICS FOR IIT-JEE /AIEEE/AIPMT

Office: 128/652 “Y” Block Kidwai Nagar Kanpur -11 UP, Website: www.apskanpur.in Mob: 09795647531, 09889787531, 0848664308 E-mail:[email protected]

216

This defect is due to the irregular curvature of cornea.

This defect can be removed by putting plane cylindrical lenses (Torric lenses) of

suitable focal length before the eye.

2 (d) Presbyopia: As people get older, the ciliary muscles weaken and the lens loses some of its elasticity.

So, the power of accommodation diminishes with age. In order to compensate,

converging spectacle lenses are employed as in the case of hyper metropia. But the

patient only requires these for reading or similar close work, since his vision for distant

objects is unimpaired. A short-sighted person on aging will normally be prescribed

bifocal spectacles. The upper half of each lens is diverging and corrects the Presbyopia

with the suitable converging lens, and the wearer looks through this part when reading.

Problem1: A person wears glasses of power-2.5 D. Is the person far sighted or near

sighted? What is the far point of the person without the glass?

Solution: Near sighted, the focal length of the lens f=-100/2.5 cm=-40 cm thus far point

of the person is at 40cm.

Problem 2: For a normal eye, the far point is at infinity and the near point of distinct

vision is 25 cm in front of eye. The cornea of the eye provides a converging power of

about 40 D and the least converging power of the eye-lens behind the cornea is about 20

D. From this rough data, estimate the range of accommodation of a normal eye.

Solution: To see objects at infinity, the eye uses its least converging power i.e.

(40+20)D or 60D

Using lens equation, we get

(1/v) – (1/-∞)= (60/100) ⇒ v = (5/3) cm

To focus an object at the near point, if the focal length is f, then

(1/f) = (1/(5/3))-(1/-25) =(25/16)cm

∴ P = (100*16)/25 = 64D

Hence, Power of eye-lens = (64-40)D = 24D

Therefore, the range of accommodation of the eye-lens is approximately 20D to 24D.

Page 217: Geometrical Optics Book With Special Lecture

ANUJ PHYSICAL SCIENCES PHYSICS FOR IIT-JEE /AIEEE/AIPMT

Office: 128/652 “Y” Block Kidwai Nagar Kanpur -11 UP, Website: www.apskanpur.in Mob: 09795647531, 09889787531, 0848664308 E-mail:[email protected]

217

3. Simple Microscope or Magnifier:

3 (a) Principle: When an object is placed between a convex lens and its principal

focus, an erect, virtual and magnified image is formed on the same side as the object.

A single convex lens, of short focal length, thus used constitutes a simple microscope

which is commonly known as a magnifying glass or a reading glass.

3 (b) Formation of images: A’

C A D

θo θi

B’ F B O

D

u eye

Simple Microscope

A small extended object AB which when viewed by an unaided eye cannot be seen

distinctly. A convex lens is then interposed between the eye and the object so that the

distance ’u’ of the object from lens is less than the focal length ‘f’ of the lens. A virtual,

erect and magnified image A’B’ will be produced. By adjusting ‘u’, the image is set at

least distance of distinct vision (D = 25 cm) from the eye so that the image becomes

most distinct. The rays of light forming the virtual and magnified image A’B’ of the small

object AB as seen through the lens by placing the eye very close to it are shown in fig.

3(c) Magnifying power or magnification: The magnifying power of a

simple of a simple microscope is defined as the ratio between the angle subtended by

the image at the eye to the angle subtended by the object at the eye when both the

image and the object are situated at the distance of distinct vision. Thus, if θi is the angle

which the image A’B’ subtends at the eye and θo is the angle which the object AB (= CB’)

subtends at the eye when both are placed at the least distance of distinct vision D, then

Magnifying power, M = θi/ θo = tan θi/ tan θo (For small angle, tan θ ≈ θ)

M= (AB/OB)/(CB’/OB’)= (AB/OB)(OB’/CB’) = OB’/OB (∵ AB = CB’)

M= -D/-u = D/u

Since the virtual image A’B’ is formed at the leased distance of distinct vision from the

lens, therefore, v = -D

Using lens formula, we get –(1/D) +(1/u) =(1/f) (1)

multiplying both sides by D, we get –(D/D) + (D/u) =(D/f)

or -1 + (D/u) = (D/f)

Page 218: Geometrical Optics Book With Special Lecture

ANUJ PHYSICAL SCIENCES PHYSICS FOR IIT-JEE /AIEEE/AIPMT

Office: 128/652 “Y” Block Kidwai Nagar Kanpur -11 UP, Website: www.apskanpur.in Mob: 09795647531, 09889787531, 0848664308 E-mail:[email protected]

218

(D/u) = 1 + (D/f) (2)

from eq.(1) and (2) we get

M = 1+(D/f) (3)

Thus, a convex lens of shorter focal length yields higher magnification.

Note:: (i) When final image is formed at infinity then magnification is given by

M∞ = D/f

(ii) If the lens is kept at a distance x from the eye then

MD = magnification for image at least distance of distinct vision

MD = 1+ [(D-x)/f]

M∞ = magnification for image formed at infinity

M∞ = (D-x)/f

3 (d) Uses: A simple microscope is used by:

(i) astrologers to read fate-lines of the hand

(ii) biology students to see the slides

(iii) watch repairers to locate defects, and

(iv) detective department to match finger prints.

Problem 3: A convex lens of focal length f is used as a simple microscope. Find its

magnifying power if the final image is at the distance of distinct vision. Find also the

magnification and the distance of the object f = 5 cm, D = 25 cm.

Solution: The object is within the focal distance θi = A’B’/v1 = AB/u1 and θo = AB/D

M = θi/ θ0 = [1+ (D/f)]

1+ (25/5) = 6

magnification = A’B’/AB = v1/u1 =25/u1

∴ u1= (v1f)/( v1- f) = (-25*5)/(-25-5) = 125/30 = 25/6 cm

∴ magnification = (25*6)/25 =6

Problem 4: A Professor reads a greeting card received on his 50 th birthday

with+2.5D glasses keeping the card 25 cm away. Ten years later, he reads his farewell

letter with the same glasses but he has to keep the letter 50 cm away. What power of

lens should he now use?

Solution: The image of the letters 25 cm away, now form at 50 cm away. Thus u=-25

cm, v=-50 cm

by lens formula 1/v-1/u=1/f

1/(-50)-1/(-25)=1/f

f=50 cm=0.5 m

Additional power of the lens P=1/0.5=2D

The total power of the lens required=2.5+2=4.5 D

Page 219: Geometrical Optics Book With Special Lecture

ANUJ PHYSICAL SCIENCES PHYSICS FOR IIT-JEE /AIEEE/AIPMT

Office: 128/652 “Y” Block Kidwai Nagar Kanpur -11 UP, Website: www.apskanpur.in Mob: 09795647531, 09889787531, 0848664308 E-mail:[email protected]

219

Problem 5: A simple microscope is rated 5X for a normal relaxed eye. When will be

its magnifying power for a relaxed farsighted eye whose near point is 40 cm?

Solution: If fe is the focal length of the lens of microscope,

then M=D/fe

5=25/fe , fe = 5 cm

For farsighted person whose near point D’= 40 cm

M’= D’/Fe = 40/5 = 8 X

Problem 6: A person uses +1.5 D glasses to have normal vision from 25 cm onwards.

He uses a 20 D lens as a simple microscope to see an object. Find the maximum

magnification power if he uses the microscope (a) together with his glass (b) without

the glass.

Solution: The focal length of the glasses (lens) used

f = 100/1.5 cm

If y is the distance of near point, then (1/y) – (1/-25) = (1.5/100)

y = -40cm

(a) The focal length of the lens of the microscope fe = (1/P) = 100/20 = 5cm

The magnifying power of the microscope together with the glass

M = 1+(D/fe) = 1+(25/5) = 6X Ans.

(b) without the glass, D’ = y= 40cm

∴ M’= 1+ (D’/fe) = 1+ (40/5) = 9X

Problem 7: The image of the moon is focussed by a converging lens of focal length 50

cm on a plane screen. The image is seen by an unaided eye from a distance of 25cm.

Find the angular magnification achieved due to the converging lens.

α α d β

Moon

f D

Solution: The ray diagram of the image of moon formed by a lens is shown in fig.

Suppose d is the diameter of the image of the moon. If α and β are the angle made by

moon and its image respectively, then

α = d/f and β = d/D

Angular magnification M = - (β/α) = -∣f/D∣= -∣50/25∣ =-2

Page 220: Geometrical Optics Book With Special Lecture

ANUJ PHYSICAL SCIENCES PHYSICS FOR IIT-JEE /AIEEE/AIPMT

Office: 128/652 “Y” Block Kidwai Nagar Kanpur -11 UP, Website: www.apskanpur.in Mob: 09795647531, 09889787531, 0848664308 E-mail:[email protected]

220

4. Compound Microscope: The magnification produced by a simple microscope is generally not sufficient to give a

detailed view of two very closely situated objects. A combination of two convex lenses is

employed to have a larger magnification. The instrument so designed is known as a

compound microscope. The convex lens O of short focal length and small aperture,

which faces the object to be viewed, is known as the objective. The convex lens E of

short focal length and comparatively large aperture, near the eye, is known as eyepiece.

The two lenses are placed in two metal tubes so as to have a common principle axis. The

eyepiece is fitted in a draw-tube and can be slided within the main tube by means of

rack and pinion arrangement to focus the microscope upon the object.

4 (a) Image Formation:

E

O C

A B1 θo

B F O1 B2 A1 θ1 O2

u A2 Eye

v

D

Compound Microscope

Let AB be an extended object situated on the principal axis at a distance slightly greater

than the focal length of the objective. As refraction takes place through the objective O, a

real, inverted and magnified image A1B1 is formed.

The lens E is so adjusted that A1B1 falls within its focal length and so the final virtual

image A2B2 is thus, highly magnified but is inverted with respect to the object AB. The

course of rays forming the final image is shown in fig.

4 (b) Magnifying power or magnification: The magnifying power of a

compound microscope is defined as the angle subtended by the final image at the eye to

the angle subtended by the object at the eye when both the object and the image are

situated at the distance of distinct vision from the eye.

If the object was situated at B2, it would have occupied length B2C such that B2C = AB

Let the angles subtended by the object B2C and final image A2B2 at the eye be θo and θi

respectively. The magnification M of the microscope is, then given by,

M = θi /θo = tan θi / tan θo (∵θi and θo are very small)

In ∆ O2B2C, tan θi = AB/O2B2 , M = A2B2/O2B2

In ∆ O2B2A2, tan θi = A2B2/O2B2 , M = A2B2/ AB = (A2B2/ A1B1)( A1B1/AB)

Page 221: Geometrical Optics Book With Special Lecture

ANUJ PHYSICAL SCIENCES PHYSICS FOR IIT-JEE /AIEEE/AIPMT

Office: 128/652 “Y” Block Kidwai Nagar Kanpur -11 UP, Website: www.apskanpur.in Mob: 09795647531, 09889787531, 0848664308 E-mail:[email protected]

221

but (A1B1/AB) = M1 and A2B2/ A1B1= M2

where M1 and M2 are magnifications produced by the two lenses O and E respectively.

M = M1 * M2 (1)

For the lens O, M1 = Distance of image/Distance of object = (v/u)

Again, since the lens E acts like a simple microscope, its magnification M2 is given by

M2 = 1+(D/fe)

where fe is the focal length of the eyepiece.

Substituting for M1 and M2 in equation (1), we get

M = (v/u) [1+ (D/fe)] (2)

If the object AB is situated very near the principal focus of the objective, the image A1B1

will be far removed from the lens O.

In that case, u = f0= Focal length of the objective

v = L = length of the microscope tube

M = (L/f0)[1+ (D/fe)] (3)

It is clear from equation (3) that in order to have a microscope of large magnification,

the two lenses used should have small focal lengths.

A good microscope has magnification as high as 2000 or even more.

Note: If final image is formed at infinity then M∞ = (v/u) (D/fe)

Length of tube = v + fe

Where u and v are distance of object and image from objective lens respectively.

Problem 8: The focal lengths of the objective and the eyepieces of a compound

microscope for 4cm and 6cm respectively. If an object is placed at a distance of 6cm in

front of the objective, what is the magnification produced by the microscope? Distance

of distinct vision is 25cm.

Solution:

A’ v1

v1 =D

A θi

B’ B Θo

u1

Magnification of microscope = (magnification produced by the objective) * (Angular

magnification produced by the eyepiece)

∴ m = (v/u)[ 1+ (D/fe)]

u = 6 cm, f0 =4 cm, fe = 6 cm, D = 25 cm

∴ v = ufo/(u – f0) = (6*4)/2 = 12 cm

∴ m = (12/6)[1+ (25/6)] = 2(31/6) = 10.33

Page 222: Geometrical Optics Book With Special Lecture

ANUJ PHYSICAL SCIENCES PHYSICS FOR IIT-JEE /AIEEE/AIPMT

Office: 128/652 “Y” Block Kidwai Nagar Kanpur -11 UP, Website: www.apskanpur.in Mob: 09795647531, 09889787531, 0848664308 E-mail:[email protected]

222

Problem 9: The focal length of the objective and the eyepiece of a microscope are 2cm

and 5cm respectively and the distance between them is 20cm. Find the distance of the

object from the objective when the final image seen by the eye is 25cm from the eye-

piece. Also find the magnifying power.

Solution: For eye piece: fe = 5cm, ve = -25cm

By lens formula, (1/v) – (1/u) = (1/f) we have

(1/-25) – (1/ue) = (1/5)

∴ ue = -25/6 cm

Given the length of the microscope vo +∣ue∣ = 20

∴ vo =20- ∣ue∣=20-(25/6) =95/6cm

Now for objective lens, (1/ vo)-(1/ uo) = (1/ fo)

[1/(95/6)]- (1/ uo)=(1/2)

∴ uo = -190/83 cm

Magnifying power M = -(vo/ uo)[1+(D/fe)] = -[(95/6)/(190/86)][1+(25/5)] = -41.5

Problem 10: A compound microscope is used to enlarge an object kept at a distance

of 0.03m from its objective which consists of several convex lenses in contact and focal

length 0.02m, if the lens of focal length 0.1mis removed from the objective, find out the

distance by which the eye-piece of the microscope must be removed to refocus the

image.

Solution: Initially, uo =-0.03m, f0 =0.02m

By lens formula, (1/v) – (1/u) = (1/f) we have,

(1/v0) – (1/-0.03) = (1/0.02)

∴ (1/v0) = (1/0.02) – (1/0.03)

or v0 =0.06m

When a lens of focal length 0.1 is removed, the focal length f0‘ of the remaining is:

(1/0.02) = (1/0.1)+(1/fo’)

∴ (1/fo’) =(1/0.02) – (1/0.1)

or f0’ = (0.02*0.1)/(0.1-0.02)=0.025 m

If v0’ now is the image position from the objective, then

(1/v0’) – (1/-0.03)= (1/0.025)

∴ (1/v0’) = (1/0.025) –(1/0.03)

or v0’ = 0.15 m

Thus displacement of eye = 0.15-0.06

Thus displacement of eye = 0.09 m

Page 223: Geometrical Optics Book With Special Lecture

ANUJ PHYSICAL SCIENCES PHYSICS FOR IIT-JEE /AIEEE/AIPMT

Office: 128/652 “Y” Block Kidwai Nagar Kanpur -11 UP, Website: www.apskanpur.in Mob: 09795647531, 09889787531, 0848664308 E-mail:[email protected]

223

Problem 11: The eye-piece and objective of a microscope, having focal lengths of

0.3m and 0.4m, respectively are separated by a distance of 0.2m. Now the eye-piece and

the objective are to be interchanged such that the angular magnification of the

instrument remains same. What is the new separation between the lenses?

Solution: Suppose the microscope is adjusted for relaxed eye. The magnification is

given by M = - (v0/u0)(D/fe)

In compound microscope, uo =f0 and v0 +fe = L or v0 = L-fe

∴ M = -[( L-fe)/f0]/[D/fe] (1)

Fe

f0

u0 = f0 v0

fe

When lenses are interchanged, let the new separation between the lenses be L’, then

M’ = -[( L’-fe)/fe]/[D/f0] (2)

given M =M’

∴ [( L-fe)/f0]/[D/fe]= [( L’-fe)/fe]/[D/f0]

or L-fe = L’-f0

∴ L’ = (f0-fe)+L = (0.4-0.3)+0.2 =0.3m

Problem 12: A compound microscope is used to enlarge an object kept at a distance

0.03m from its objective which consists of several convex lenses in contact and has focal

length 0.02m. if a lens of focal length 0.1m is removed from the objective, find out the

distance by which the eyepiece of the microscope must be moved to refocus the image.

Solution: If initially the objective forms the image at distance v1,

(1/v1) – (1/-3) = (1/2) i.e., v1 =6m

Now as in case of lenses in contact, (1/F) = (1/f1) + (1/f2) +. . . .

or (1/F) = (1/f1) +(1/F’) with (1/F’) =(1/f2) + (1/f3) +. . . .

so if one of the lenses is removed, the focal length of the remaining lens system.

(1/F’) = (1/F)-(1/f1) = (1/2)-(1/10) i.e., F’ = 2.5cm

This lens will form the image of same object at a distance v2 such that

(1/v2) – (1/-3) = (1/2.5) i.e., v2 =15cm

So to refocus the image, the eyepiece must be moved by the same distance through

which the image formed by the objective has shifted, i.e., 15-6 =9 cm away from the

objective.

Page 224: Geometrical Optics Book With Special Lecture

ANUJ PHYSICAL SCIENCES PHYSICS FOR IIT-JEE /AIEEE/AIPMT

Office: 128/652 “Y” Block Kidwai Nagar Kanpur -11 UP, Website: www.apskanpur.in Mob: 09795647531, 09889787531, 0848664308 E-mail:[email protected]

224

Problem 13: The focal lengths of the objective and the eyepiece of a compound

microscope are 2.0cm and 3.0 cm respectively. The distance between the objective and

the eyepiece is 15.0cm. The final image formed by the eyepiece is at infinity. Find the

distance of object and image produced by the objective, from the objective lens.

Solution: As the final image is at infinity, the distance of intermediate image from eye

lens ue will be given by (1/-∞) –(1/ue) = (1/fe) i.e., ue = -fe = -3cm

and as the distance between the lenses is 15.0 cm, the distance of intermediate image

(formed by objective) from the objective will be

v = L-ue = L-fe =15-3 = 12 cm

and if u is the distance of object from objective,

(1/12) – (1/u) = (1/2) i.e., u = -2.4cm

so the object is at a distance of 2.4 cm in front of the objective

Problem 14: In a compound microscope the objective and the eyepiece have focal

lengths of 0.95 cm and 5 cm respectively, and are kept at a distance of 20cm from the

eyepiece. Calculate the position of the object and the total magnification.

Solution: As the final image is at 25cm in front of the eyepiece,

(1/-25) – (1/ue) = (1/5) i.e., ue = -25/6

and so, me = ve/ue = -25/(-25/6) = 6 (1)

Now for the objective, v = L-ue = 20-(25/6) = (95/6)

so if the object is at a distance u from the objective,

(6/95) – (1/u) = (1/0.95)

i.e., u = -(95/94) cm

i.e., the object is at a distance (95/94) cm in front of the field lens.

Also, m = v/u = (95/6)/(-95/94)=-[94/6] (2)

So total magnification, M = m*me = -[94/6][6] = -94

i.e., the final image is inverted, virtual, and 94 times that of object.

Remark: In this example as final image is at least distance of distinct vision D (=25cm)

mθ = [1+D/fe] = 1+(25/5) =6 =me

so MP = m* mθ = m* me = Linear magnification

Page 225: Geometrical Optics Book With Special Lecture

ANUJ PHYSICAL SCIENCES PHYSICS FOR IIT-JEE /AIEEE/AIPMT

Office: 128/652 “Y” Block Kidwai Nagar Kanpur -11 UP, Website: www.apskanpur.in Mob: 09795647531, 09889787531, 0848664308 E-mail:[email protected]

225

5. Astronomical Telescope:

O

fo fe

θo

O1 B1 θi O2

A1

Astronomical Telescope

It is an optical instrument used to have clear and detailed view of heavenly objects. In

simplest form if consists of two convex lenses separated some distance apart and

mounted in two metal tubes so as to have a common principal axis. The lens O which

faces the object is known as objective and has a large aperture and a large focal length

’f0’. The lens E which faces the eye is known as eyepiece and has a small aperture and a

smaller focal length ‘fe’. The tube carrying eyepiece can slide into the tube carrying

objective by means of rack and pinion arrangement. A parallel beam of light coming

from a distant object suffers refraction through objective and produces a real, inverted

and diminished image at a distance ’f0’ from O1. This image then acts as an object for the

eyepiece E and so the final magnified image is obtained after refraction through the

eyepiece.

As astronomical telescope can be used in the following two adjustments:

(i) Normal adjustment: An astronomical telescope is said to be in normal

adjustment if the final image formed after refraction through both the lenses is situated

at infinity. The course of rays, in such an adjustment, is shown in fig.

A1B1 is the image of distant object situated at infinity. If A1B1 happens to be situated at

the principle focus of the lens E, the rays will emerge in parallel direction and a final

virtual but magnified image is formed at infinity.

Magnification ‘M’, in normal adjustment, is defined as the ratio of the angle θi subtended

by the final image at the ratio of the angle θi subtended by the final image at the eye as

seen through the telescope to the angle θo subtended by the object at the unaided eye

when both the image and the object are situated at infinity.

Thus, M = θi/θo = = tan θi /tan θo (∵θi and θo are small)

Now, In ∆ A1O1B1, tan θo = A1B1/O1B1

and In ∆ A1O2B1, tan θi = A1B1/O2B2

Page 226: Geometrical Optics Book With Special Lecture

ANUJ PHYSICAL SCIENCES PHYSICS FOR IIT-JEE /AIEEE/AIPMT

Office: 128/652 “Y” Block Kidwai Nagar Kanpur -11 UP, Website: www.apskanpur.in Mob: 09795647531, 09889787531, 0848664308 E-mail:[email protected]

226

M = (A1B1/ O2B1)( O1B1/ A1B1) = O1B1/O1B1

But O1B1 = f0 = focal length of the objective

and O2B2 = fe = focal length of eyepiece

M = f0/fe (1)

Thus, the magnification of astronomical telescope in normal adjustment is equal to ratio

of the focal length of objective to the focal length of the eyepiece. The distance between

the two lenses in this case will be (f0 + fe). The strain on the eye will be least if the object

is viewed for a long period.

(ii) When the final image is formed at the distance of distinct vision:

O

f0 u

θo

O1 θo B2 θi O2

B1

A2 D

The course of rays in such an adjustment is shown in fig.

A parallel beam of light coming from an object situated at infinity gets refracted through

the objective O and produces a real, inverted and diminished image A1B1 at a distance f0

from O1. If A1B1 happens to be within the focal length ‘fe’ of the eyepiece, a final virtual

but magnified image A2B2 is observed. The position of eyepiece E is so adjusted that

final image is obtained at a distance of distinct vision D from the eye. Magnification in

this case is defined as the ratio of the angle ‘θi’ subtended at the eye by the final image

formed at the distance of distinct vision to the angle ‘θ0’ subtended at the unaided eye

by the object situated at infinity.

Thus, M = θi/θo = = tan θi / tan θo (∵θi and θo are very small)

In ∆ A1B1O1, tan θo = A1B1/O1B1

and In ∆ A1B1O2, tan θi = A1B1/O2B1

M = (A1B1/ O2B1)( O1B1/ A1B1) = O1B1/O2B1 = f0/-u (2)

where u is the distance of A1B1 from the lens E. Considering refraction through lens E,

(1/fe) = (1/v) – (1/u)

Here, v = -D

–(1/u) = (1/fe) +(1/D) = (fe +D)/( feD)

We get u = (fe D)/( fe +D) (3)

substituting for u in equation (1) we get

M = f0[(fe +D)/( fe D)] (4)

Out of the two adjustments discussed above the second adjustment gives a higher

Page 227: Geometrical Optics Book With Special Lecture

ANUJ PHYSICAL SCIENCES PHYSICS FOR IIT-JEE /AIEEE/AIPMT

Office: 128/652 “Y” Block Kidwai Nagar Kanpur -11 UP, Website: www.apskanpur.in Mob: 09795647531, 09889787531, 0848664308 E-mail:[email protected]

227

magnification since the factor (f0/fe)[(fe +D)/D]>1.

Equation (1) and (4) also show that for greater magnification.

(i) the focal length ‘f0’ of the objective should be large, and

(ii) the focal length ‘fe’ of the eyepiece should be small

The objective of an astronomical telescope should have a large aperture so as to allow a

large number of rays to fall on it. In that case, the intensity of the image is large. This

becomes even more essential in the case of heavenly bodies which are situated at large

distances. A powerful astronomical telescope placed at Yerke’s observatory at Lake

Geneva has an objective aperture about one metre and of focal length about 18 m.

Problem 15: A telescope has an objective of focal length 50 cm and an eye piece of

focal length 5 cm. The least distance of distinct vision on a scale 200cm away from the

objective. Calculate:

(i) The separation between the objective and the eyepiece

(ii) The magnification produced

Solution: Let AB be the position of the object

u = 200 cm, f = 50 cm

∴ The image A’B’ formed by the objective is a distance v from it

∴ v = uf/(u-f) = (200*50)/(200-50) = 1000/15 = (200/3) cm

A l

E

A’’ A’ fe

B O f0

B’

B’’

This serves as an object for the eyepiece

the distance between A’B’ and the eyepiece is u’ = l-(200/3) where l is the separation

between the lenses. The image (final) distance is v’ = 25 cm from the eyepiece and

f’ = 5 cm

∴ u’= v’f’/(v’-f’) = (-25*5)/(-25-5) = (25/6) cm

∴ l = (25/6) + (200/3)=425/6 = 70.83 cm

Total magnification mo*me = (v/u)(v’/u’) = (200/3)(1/200)(25)(6/25) = 2

Page 228: Geometrical Optics Book With Special Lecture

ANUJ PHYSICAL SCIENCES PHYSICS FOR IIT-JEE /AIEEE/AIPMT

Office: 128/652 “Y” Block Kidwai Nagar Kanpur -11 UP, Website: www.apskanpur.in Mob: 09795647531, 09889787531, 0848664308 E-mail:[email protected]

228

Problem 16: A telescope has an objective of focal length 50 cm and an eye-piece of

focal length 5cm. The least distance of distinct vision 25cm. The telescope is focussed

for distinct vision on a scale 200cm away from the objective. Calculate (a) the

separation between objective and eye-piece and (b) the magnification produced

Solution:

fo vo ue fe

ve =D

(a) For objective lens u = -200cm, fo = +50cm

By lens formula, (1/v) –(1/u) = (1/f) we have

(1/v0) – (1/-200) = (1/50)

which gives v0 = 200/3 cm

For eyepiece, ve = -25 cm fe =+5cm

Now by lens formula, (1/v) – (1/u) = (1/f)

we have (1/-25) –(1/ue) = (1/+5)

which gives ue = -(25/6) cm

Length of the telescope L = ∣vo∣+∣ue∣ = (200/3) +(25/6) =70.80 cm Ans.

(b) magnification M = -M0*Me = -(v0/u0)( ve/ue) =-[(200/3)/200][-25/(-25/6)]=-2 Ans.

Problem 17: An astronomical telescope has an angular magnification of magnitude 5

for distant objects. The separation between the objective and eyepiece is 36 cm and the

final image is formed at infinity. Determine the focal length of objective and eyepiece

Solution: In case of astronomical telescope if object and final image both are at

infinity.

MP = -(fo/fe) and L = fo+fe

so here –( fo/fe) = -5 and (fo+fe) =36

solving these for fo and fe we get

fo = 30 cm and fe = 6 cm

Page 229: Geometrical Optics Book With Special Lecture

ANUJ PHYSICAL SCIENCES PHYSICS FOR IIT-JEE /AIEEE/AIPMT

Office: 128/652 “Y” Block Kidwai Nagar Kanpur -11 UP, Website: www.apskanpur.in Mob: 09795647531, 09889787531, 0848664308 E-mail:[email protected]

229

Problem 18: The diameter of the moon is 3.5*103 km and its distance from the earth

is 3.8*105 km. It is seen through a telescope having focal lengths of objective and

eyepiece as 4m and 10 cm respectively. Calculate (a) magnifying power of telescope (b)

length of telescope tube and (c) angular size of image of moon.

Solution: For normal adjustments,

(a) ∣MP∣ = (fo/fe) = (4*100)/10 = 40

(b) L = fo+fe = 400 +10 = 410 cm = 4.10 m

(c) As the angle subtended by moon on the objective of the telescope.

Θ0 = (3.5*103)/(3.8*105) = (3.5/3.8)*10-2 rad.

And as ∣MP∣= ∣( Θ/ Θ0)∣, the angular size of final image.

∣θ∣= ∣MP∣ Θ0 = 40(3.5/3.8)10-2 = 0.3684 rad.

i.e., ∣θ∣= 0.368(180/π) ≈ 210

Problem 19: An astronomical telescope consisting of an objective of focal length 60

cm and eyepiece of focal length 3cm is focussed on the moon so that the final image is

formed at least distance of distinct vision, i.e., 25 cm from the eyepiece. Assuming the

angular diameter of moon as (1/2)o at the objective, calculate (a) angular size and (b)

linear size of image seen through the telescope.

Solution: As final image is at least distance of distinct vision

∣MP∣ = (fo/fe)[1+(fe/D)]= (60/3)[1+(3/25)]=22.4

Now as by definition MP = (θ/θo), so the angular size of image.

Θ = MP*θo = 22.4(1/2)0 =11.20 =(π/180)(11.2)≈0.2 rad.

And if I is the size of final image which is at least distance of distinct vision,

θ = (1/25)

i.e., I = (25*θ) = 25*0.2 = 5cm

Problem 20: The aperture of the largest telescope in the world is 5 m. If the

separation between the moon and the earth is 4*105km and the wavelength of visible

light is 5000Å. Calculate (a) resolving power of telescope and (b) minimum separation

between objects on the surface of moon so that they are just resolved.

Solution: (a) definition, RP = D/1.22λ = [5/(1.22*5*10-7)]= (1/1.22)*107

(b) As limit of resolution ∆θ =(1/RP) and if d is the distance between objects on the

surface of moon which is at a distance r from the telescope, ∆θ = (d/r)

so, ∆θ = 1/RP =d/r, i.e., d = r/RP

d = [(4*105)*103*1.22]/107 = 48.8m

Page 230: Geometrical Optics Book With Special Lecture

ANUJ PHYSICAL SCIENCES PHYSICS FOR IIT-JEE /AIEEE/AIPMT

Office: 128/652 “Y” Block Kidwai Nagar Kanpur -11 UP, Website: www.apskanpur.in Mob: 09795647531, 09889787531, 0848664308 E-mail:[email protected]

230

Problem 21: (a) An astronomical telescope consists of an eyepiece having two lenses

of focal lengths 5cm and 4cm. The gap t between the eyepiece lenses is adjusted such

that the longitudinal spherical aberration caused by it is minimum. The telescope is

adjusted for final image at infinity. Determine the magnification of the telescope.

100cm 5cm 4cm

Objective eyepiece

(b) An astronomical telescope is being used in normal adjustment (i.e. the final image is

formed at infinity). If the length of the telescope is reduced by (1/11)th of the focal

length of the eyepiece, then the final image is found to be formed at the near point of the

eye, the least distance of distinct vision is 25cm and the angular magnification of the

telescope in the normal use is 20. Determine the angular magnification after changing

the tube length. Also draw the ray diagrams in both cases.

Solution:

(a) The least spherical aberration is due to separated doublet. The separation of t must

be equal to the difference of focal length of lenses.

t = f1-f2 = 5-4 =1cm

Now the focal length of the eyepiece is given by (1/fe) = (1/f1)+(1/f2) –(t/f1f2)

=(1/5)+(1/4)-(1/5*4) = (8/20)

fe = (20/8) cm

For final image at infinity M = fo/fe =[100/(20/8)] = 40

(b) When the final image is formed at infinity, the first image is formed at the foci of

both the lenses and tube length = f0 +fe

Page 231: Geometrical Optics Book With Special Lecture

ANUJ PHYSICAL SCIENCES PHYSICS FOR IIT-JEE /AIEEE/AIPMT

Office: 128/652 “Y” Block Kidwai Nagar Kanpur -11 UP, Website: www.apskanpur.in Mob: 09795647531, 09889787531, 0848664308 E-mail:[email protected]

231

f0 fe

Fθ, Fo

O E

When the final image is at least distance of distinct vision, the first image will be lying

between focus Fo and optical centre E of the eye lens.

f0 fe

Fθ, Fo

O E

D

∴ In the second case ue = - [fe-(fe/11)] = - [+(10/11)fe]

ve = -D = -25

Using (1/ve) - (1/ue) = (1/fe)

We get fe = 2.5cm

In normal adjustment, ∣M1∣ = (f0/fe)

where M1=+20, fe = 2.5 ⇒ f0 = 2.5*20 = 50cm

when the final image is formed at near point, ∣M2∣ = (f0/fe)[1+( fe/D)]

∣M2∣ = 20[1+(2.5/25)]= 20(1.1)

∣M2∣ = 2.2

Page 232: Geometrical Optics Book With Special Lecture

ANUJ PHYSICAL SCIENCES PHYSICS FOR IIT-JEE /AIEEE/AIPMT

Office: 128/652 “Y” Block Kidwai Nagar Kanpur -11 UP, Website: www.apskanpur.in Mob: 09795647531, 09889787531, 0848664308 E-mail:[email protected]

232

6. Terrestrial Telescope: As astronomical telescope is used to view heavenly objects

since the inversion of their images does not produce any complication. While viewing objects on

earth we would prefer to have their images erect. Therefore, astronomical telescope is not

suitable in such cases. By using an additional convex lens O in between O1 and O2 of an

astronomical telescope we can have the final erect image. The lens O is called erecting lens,

while the improved version of the telescope is called terrestrial telescope.

Rays from the distant object get refracted through the objective O1, giving a real inverted image

A1B1 fig. (a). The erecting lens O is so adjusted that its distance from A1B1 is equal to twice its

(erecting lens) focal length. An image A2B2 having same size as that of A1B1, inverted w.r.t. A1B1

and hence erect w.r.t. the object is obtained at a distance 2f on other side of O. A2B2 acts as an

object for lens at O and final erect and magnified image is obtained after refraction through O2. If

the distance O2B2 is equal to focal length ‘fe’ of the eye lens O2, final image is formed at infinity

figure and the telescope is said to be in normal adjustment. If the distance O2B2 is less than ‘fe’

then corresponding to a certain value of this distance, a virtual and magnified image is obtained

at the distance of distinct vision as shown in fig. (b)

2f E

A2

ө0 O1 B1 O B2 O2

A1

2f fe

(a) Terrestrial telescope: Normal adjustment

2f E

A2

Ө0 O1 B1 O B2 O2

A1

2f u<fe

D

(b) Terrestrial telescope: Adjustment for distance of distinct vision

Since the sizes of A2B2 and A1B1 are same, introduction of the erecting lens O has not produced

any charge in its magnifying power but has helped in getting the final image erect only. It may

also be noted that the use of erecting lens O results in a slight increase (equal to four times the

focal length of erecting lens) in the length of tube of telescope.

Note: (i) MD = (fo/fe) [1+ (fe/D)] & M∞ = (f0/fo)

(ii) LD = fo +4f +ue & L∞ = fo +4f +fe

Page 233: Geometrical Optics Book With Special Lecture

ANUJ PHYSICAL SCIENCES PHYSICS FOR IIT-JEE /AIEEE/AIPMT

Office: 128/652 “Y” Block Kidwai Nagar Kanpur -11 UP, Website: www.apskanpur.in Mob: 09795647531, 09889787531, 0848664308 E-mail:[email protected]

233

7. Galilean Telescope:

A2

O2 B1

Θ0 O1 B2 θo θi

A1

D

Galileo’s telescope

Instead of using a combination of two lenses O1 and O2 for getting an erect image,

Galileo used only one concave lens to get the final erect image. Parallel beam of incident

rays from infinity are focussed by the objective O. An inverted image A1B1 would have

been formed after refraction through O1. Before the rays meet at A1 a concave lens (at

O2) intercepts them figure. The beam diverges and the final erect image A2B2 is

obtained. The distance O2B1 is adjusted that final image is formed at the distance of

distinct vision. If O2B1 is equal to the focal length ‘fe’ of eye lens at O2, final image is

formed at infinity and the telescope is said to be set in normal adjustment. In such a case

the length of the tube is equal to the difference between the focal lengths of two lenses.

The field of view of this telescope is small because of the use of concave lens.

When set in normal adjustment. Its magnifying power ‘M’ is given by

M = θi/ θo = tan θi/tan θo =

M=(A1B1/B1O2)/( A1B1/B1O1) = (B1O1/ B1O2)

or M = f0/fe = focal length of objective/focal length of eye lens

Note:

(i) magnification when final image is formed at least distance of distinct vision is given

by MD = (fo/fo) [1+ (fe/D)]

(ii) Length of tube is given by

LD = fo - ue (for least distance)

L∞ =fo – fe (for normal adjustment)

M∞ =( f0/fo)

Page 234: Geometrical Optics Book With Special Lecture

ANUJ PHYSICAL SCIENCES PHYSICS FOR IIT-JEE /AIEEE/AIPMT

Office: 128/652 “Y” Block Kidwai Nagar Kanpur -11 UP, Website: www.apskanpur.in Mob: 09795647531, 09889787531, 0848664308 E-mail:[email protected]

234

Problem 22: A Galilean telescope consists of an objective of focal length 12cm and an

eyepiece of focal length 4 cm. What should be the separation of the two lenses when the

virtual image of a distant object is formed at a distance of 24cm from the eyepiece?

What is the magnifying power of the telescope under this condition?

Solution: As the object is distant, i.e. u = -∞, so

(1/v) –(1/-∞) = (1/f0)

i.e., v = f0 =12 cm

i.e., The objective will form the image IM at its focus which is at a distance of 12cm from

O. Now as the eyepiece of focal length -4cm forms the image I at a distance 24 cm from

it.

(1/-24) – (1/ue)=(1/-4), i.e., ue = 24/5 = 4.8 cm

i.e., the distance of IM from the eye lens EA is 4.8cm. so the length of tube,

I fo =12 cm

f0 f0 4.8cm

θ ue

θ0 O θ0 θ A IM

L =7.25cm B

24 cm

L = OA –EA = 12-4.8 = 7.2 cm

Now by definition, MP = (θ/θ0) ≈ tan θ/tan θ0 = [(AB/EA)/ (AB/OA)] = OA/EA

so, MP = fo/ue = 12/4.8 =10/4 = 2.5

i.e., the image is erect, virtual and is at a distance of 24-7.2 = 16.8cm in front of the

objective.

Page 235: Geometrical Optics Book With Special Lecture

ANUJ PHYSICAL SCIENCES PHYSICS FOR IIT-JEE /AIEEE/AIPMT

Office: 128/652 “Y” Block Kidwai Nagar Kanpur -11 UP, Website: www.apskanpur.in Mob: 09795647531, 09889787531, 0848664308 E-mail:[email protected]

235

8. Reflecting Telescope:

Reflecting telescope is based upon the same principle except that the formation of

images takes place by reflection instead of refraction.

There are two types of reflecting telescopes which are described below:

A) Newtonian reflecting telescope B) Cassegranian reflecting telescope

A) Newtonian reflecting telescope:

C B2 A2

M

B1 A1 I

E

Newtonian reflecting telescope

It consists of a concave mirror C whose reflecting surface is exposed to the incident

beam coming from a distant source. As the beam of light strikes, it gets reflected and

would have produced an image at I if a plane mirror M were not placed in its path. The

plane mirror which is inclined with the vertical deflects the position of image from I to

A1B1 acts as an object for an eye lens (convex) E, thereby producing a virtual and

magnified image A2B2. By adjusting the position of lens E, image A2B2 can be managed to

be at the distance of distinct vision. If A1B1 is situated on the focal plane of E, final image

A2B2 would be formed at infinity and the telescope is said to be in normal adjustment.

Use of plane mirror can be avoided by making the concave mirror ’C’ inclined so that it

throws the image to one side where the eyepiece can be placed to view it directly.

Page 236: Geometrical Optics Book With Special Lecture

ANUJ PHYSICAL SCIENCES PHYSICS FOR IIT-JEE /AIEEE/AIPMT

Office: 128/652 “Y” Block Kidwai Nagar Kanpur -11 UP, Website: www.apskanpur.in Mob: 09795647531, 09889787531, 0848664308 E-mail:[email protected]

236

B) Cassegranian reflecting telescope:

A Cassegranian type reflecting telescope is shown in the fig. Parallel beam of light from a

distant object strikes a concave mirror C and is reflected towards a convex mirror M. If

the convex mirror M were not there the beam would have met at F. Mirror M reflects the

beam forming a real image of the object at I which happens to be on the focal plane of an

eyepiece E.

After refraction through the eyepiece which is placed behind a central hole in the

concave mirror, the image can be seen by the eye. A reflecting telescope of this type is

fixed at Mount Palomar in U.S.A. The telescope has a concave mirror of 200 inches in diameter.

C

E

C

M

Cassegranian reflecting telescope

Advantages of reflecting telescope over a refracting telescope are

following:

1. Since there is no loss of light in case of reflection, image produced by a reflecting

telescope is much brighter than that produced by a refracting telescope.

2. Errors like chromatic aberration, etc., associated with refraction through lenses are

not present in a reflecting telescope.

3. Presence of spherical aberration can be avoided by using parabolic reflecting mirrors.

4. It is more convenient to obtain a concave mirror than to obtain, a convex lens of same

size, since, a number of aberrations creep in as the size of lens is increased.

Page 237: Geometrical Optics Book With Special Lecture

ANUJ PHYSICAL SCIENCES PHYSICS FOR IIT-JEE /AIEEE/AIPMT

Office: 128/652 “Y” Block Kidwai Nagar Kanpur -11 UP, Website: www.apskanpur.in Mob: 09795647531, 09889787531, 0848664308 E-mail:[email protected]

237

9. Camera:

The principle of a photographic camera is that of a convex lens forming a real image on

a sensitive plate or film held at the back of a light-tight chamber formed by canvas

bellows with blackened interior.

Lens Filter

Sliding mechanism

Camera

The lens is mounted on a slide so that its distance from the sensitive plate can be altered

for bringing to sharp focus the images of objects situated at varying distances.

An adjustable stop is placed in front of the lens so that the amount of light passing

through it may be varied. The lens (combination) is corrected for chromatic aberration

and curvature of field.

It can be easily shown that the intensity of illumination E of the image is directly

proportional to the area of the aperture and inversely proportional to the square of the

focal length of the lens,

E α [Area of the aperture/(Focal length)2]

If d be the diameter of the aperture and focal length of the lens, the

E α [π(d/2)2/f2]= constant(d2/f2) (2)

The quantity (d/f) is called the aperture ratio of the lens. It is clear from eq. (2) that

different lenses with the same value of (d/f) will give the same brightness of a given

object.

The reciprocal (f/d) of the aperture ratio is called the focal ratio or f-number of the lens.

Thus, a lens of 20 cm focal length and 4 cm diameter is said to have a f-number of 5

which means that the diameter of the lens aperture is (f/5).

Since exposures needed under given conditions may be assumed to be proportional to

the square of this number, the apertures are usually indicated in a series such that the

exposures required are doubled in passing from one to the next, thus we have the series

(f/2), (f/2.8), (f/4), (f/5.6), (f/8), (f/11.3), (f/16) etc.

Page 238: Geometrical Optics Book With Special Lecture

ANUJ PHYSICAL SCIENCES PHYSICS FOR IIT-JEE /AIEEE/AIPMT

Office: 128/652 “Y” Block Kidwai Nagar Kanpur -11 UP, Website: www.apskanpur.in Mob: 09795647531, 09889787531, 0848664308 E-mail:[email protected]

238

Problem 23: Camera A, having an (f/8) lens, 2.5cm in diameter, photographs an

object using the correct exposure of (1/100) sec. What exposure should a camera B use

in photographing the same object if it has an (f/4) lens 5 cm in diameter?

Solution: focal length of the lens A is fA = 8*2.5 = 20 cm

focal length of the lens B is fB = 4*5 = 20 cm

Now time of exposure α [1/(f/4)]2

∴ t(f/n)2 = constant

∴ tA[(f/n)A]2 = tB[(f/n)B]2 ; (1/100)(20/8)2= tB(20/4)2

∴ tB =(1/100)[(20/8)(4/20)]2 = (1/400)s

Problem 24: To print a photograph from a negative, the time of exposure to light from

a lamp placed 0.5m away is 2.5 second. How much exposure time is required if the lamp

is placed 1m away?

Solution: Let E = Amount of light falling on the photographic film

x = Distance of the lamp

and t = Time of exposure

then, E α (t/x2)

∴ E1/E2 =(t1x22)/(t2x12)

According to problem, E1 = E2

x12 t2 = x22 t1

or t2 = [(x22)/(x12)]t1 = 10 sec

Page 239: Geometrical Optics Book With Special Lecture

ANUJ PHYSICAL SCIENCES PHYSICS FOR IIT-JEE /AIEEE/AIPMT

Office: 128/652 “Y” Block Kidwai Nagar Kanpur -11 UP, Website: www.apskanpur.in Mob: 09795647531, 09889787531, 0848664308 E-mail:[email protected]

239

10. Prism Binocular:

O

P1

P2

E

Prism Binocular

Prism binocular consists of a pair of astronomical telescope, one for each eye. The two

telescopes are mounted side by side. A pair of right-angled totally reflecting prisms P1

and P2 are placed between the objective O and eyepiece E of an astronomical telescope.

The reflecting edges of these prisms are at right angles to each other.

The objective forms an image which is both inverted and perverted. The two internal

reflections in the prism P1 invert vertically the image formed by objective. The prism P2

inverts the image laterally. Thus, the final image as seen through the eyepiece is natural

and erect. The image is also free from lateral inversion.

In prism binocular, we get two images of the same object from different angles

simultaneously. The superposition of these two images gives the perception of depth

along with length and breadth. Hence, binocular vision gives proper three dimensional

images.

The main advantage of prism binocular is that an objective of large focal length can be

used. This would give greater magnification. Moreover, the length of the instrument is

small. A wide field of view with good perception of depth is an added advantage.

Page 240: Geometrical Optics Book With Special Lecture

ANUJ PHYSICAL SCIENCES PHYSICS FOR IIT-JEE /AIEEE/AIPMT

Office: 128/652 “Y” Block Kidwai Nagar Kanpur -11 UP, Website: www.apskanpur.in Mob: 09795647531, 09889787531, 0848664308 E-mail:[email protected]

240

Optical instrument: Problems from competitive exams

Problem 25 (AIEEE 2003): The image formed by an objective of a compound

microscope is

(a) virtual and diminished

(b) real and diminished

(c) real and enlarged

(d) virtual and enlarged

Solution: objective of compound microscope is a convex lens. Convex lens forms real

and enlarged image when an object is placed between its focus and lens.

Problem 26 (AIEEE 2008): An experiment is performed to find the refractive index

of glass using a travelling microscope. In this experiment distance are measured by

(a) a vernier scale provided on the microscope

(b) a standard laboratory scale

(c) a meter scale provided on the microscope

(d) a screw gauge provided on the microscope

Solution: answer (a) explanation not given

Problem 27 (AIEEE 2005): Two point white dots are 1 mm apart on a black paper.

They are viewed by eye of pupil diameter 3 mm approximately, what is the maximum

distance at which these dots can be resolved by the eye?

[Take wavelength of light = 500nm]

(a) 5m (b) 1m (c) 6m (d) 3m

Solution: We know (y/D) ≥ 1.22 (λ/d)

D ≤ (yd)/1.22λ = (10-3 *3*10-3)/ 1.22*5*10-7 = 30/6.1 = 5m

∴ Dmax = 5m

Page 241: Geometrical Optics Book With Special Lecture

ANUJ PHYSICAL SCIENCES PHYSICS FOR IIT-JEE /AIEEE/AIPMT

Office: 128/652 “Y” Block Kidwai Nagar Kanpur -11 UP, Website: www.apskanpur.in Mob: 09795647531, 09889787531, 0848664308 E-mail:[email protected]

241

Geometrical optics problems asked in IIT-JEE/AIEEE

Reflection of light at plane and spherical mirror:

Problem 1(IIT-JEE 2011): A light ray travelling in glass medium on glass-air

interface at an angle of incidence θ. The reflected (R) and transmitted (T) intensities,

both as function of θ, are plotted. The correct sketch is?

100% T 100% T

Intensity

R R

0 θ 900 0 θ 90

0

100% T 100% T

R R

0 θ 900 0 θ 90

0

Solution: After critical angle reflection will be 100% and transmission is 0%. Options

(b) and (c) satisfy this condition. But option (c) is the correct option. Because in option

(b) transmission is given 100% at θ = 00, which is not true. So correct answer is (c).

Problem 2(AIEEE 2011): A car is fitted with a convex side view mirror of focal

length 20cm. A second car 2.8m behind the car is overtaking the first car is a relative

speed of 15m/s. The speed of the image of the second car as seen in the mirror of the

first one is?

(a) 1/15 m/s (b) 10 m/s (c) 15 m/s (d) 1/10 m/s

Solution: 1/u + 1/v = 1/f GIVES - (1/u2)(du/dt) –(1/v2)(dv/dt) = 0

dv/dt = -(v2/u2)(du/dt) but v/u = f/(u-f)

dv/dt = -[f/(u-f)]2(du/dt)

= {0.2/(-2.8-0.2)}2* 15 = 1/15 ms-1 answer

Problem 3 (IIT- JEE 2010): Image of an object approaching a convex mirror of

radius of curvature 20m along its optical axis is observed to move from 25/3 m to 50/7

m in 30 sec. What is the speed of the object in kmh-1?

(a) 3 (b) 4 (c) 5 (d) 6

Page 242: Geometrical Optics Book With Special Lecture

ANUJ PHYSICAL SCIENCES PHYSICS FOR IIT-JEE /AIEEE/AIPMT

Office: 128/652 “Y” Block Kidwai Nagar Kanpur -11 UP, Website: www.apskanpur.in Mob: 09795647531, 09889787531, 0848664308 E-mail:[email protected]

242

Solution: using mirror formula, [1/(+25/3)] + [1/(-u1)] = 1/10

or 1/u1 = (3/25) – (1/10) or u1 = 50 m

and 1/(+50/7) +(1/-u2) = 1/10

1/u2 = (7/50) – (1/10) or u2= 25 m

speed of object = (u1 -u2) / time = 25/30 ms-1 = 3 kmh-1

Problem 4 (IIT – JEE 2007):

Statement 1: The formula connecting u, v and f for a spherical mirror is valid only for

mirrors whose sizes are very small compared to their radii of curvature.

Statement 2: Laws of reflection are strictly valid for plane surfaces, but not for large

spherical surfaces.

(a) Statement 1 is true, statement 2 is true; statement 2 is a correct explanation for

statement 1

(b) Statement 1 is true, statement 2 is true; statement 2 is not a correct explanation for

statement 1

(c) Statement 1 is true, statement 2 is false

(d) Statement 1 is false, statement 2 is true

Solution: Laws of reflection can be applied to any type of surface.

Problem 5 (IIT-JEE 2005): A container is filled with water (μ= 1.33) upto a height

of 33.25 cm. A concave mirror is placed 15 cm above the water level and the image of an

object placed at the bottom is formed 25 cm below the water level. The focal length of

the mirror is

(a) 10 cm (b) 15 cm (c) 20 cm (d) 25 cm

15 cm

33.25cm 25cm

μ= 1.33

Solution: Distance of object from mirror

= 15+(33.25/1.33)= 40 cm

distance of image from mirror

= 15+(25/1.33) = 33.8 cm

for the mirror, 1/v + 1/u = 1/f

[1/(-33.8)] +[1/(-40)] =1/f

f = -18.3 cm so most suitable answer is (c)

Page 243: Geometrical Optics Book With Special Lecture

ANUJ PHYSICAL SCIENCES PHYSICS FOR IIT-JEE /AIEEE/AIPMT

Office: 128/652 “Y” Block Kidwai Nagar Kanpur -11 UP, Website: www.apskanpur.in Mob: 09795647531, 09889787531, 0848664308 E-mail:[email protected]

243

Refraction of light at plane surfaces:

Problem 6 (AIEEE 2011): Let the x-z plane be the boundary between two

transparent media. Medium 1 in z ≥ 0 has a refractive index of √2 and medium 2 with z

< 0 has a refractive index of √3. A ray of light in medium 1 given by the vector

A = 6√3 +8√3 -10k is incident on the plane of separation. The angle of refraction in

medium 2 is?

(a) 450 (b) 600 (c) 750 (d) 300

Solution: As refractive index for z > 0 and z ≤ 0 is different x-y plane should be

boundary between two media. Angle of incidence cos i = |[AZ/√Ax2+Ay2+Az2]| = ½

i=600 from Snell’s law sin i / sin r = √3/2 r = 450

Problem 7 (AIEEE 2011): A beaker contains water up to a height h1 and kerosene

of height h2 above water so that the total height of (water + kerosene) is (h1+ h2).

Refractive index of water is μ1 and that of kerosene is μ2. The apparent shift in the

position of the bottom of the beaker when viewed from above is

(a) [1-(1/μ1)] h2 + [1-(1/μ2)] h1 (b) [1+(1/μ1)]h1+[1+(1/μ2)]h2 (c) [1-(1/μ1)]h1 +[1-(1/μ2)]h2 (c) [1+(1/μ1)]h2-[1+(1/μ2)]h1

Solution: Apparent shift Δh = [1-(1/μ)] h

so apparent shift produced by kerosene Δh1 = [1-(1/μ1)] h1

and apparent shift produced by kerosene Δh2 = [1-(1/μ2)] h2

Δh = Δh1+Δh2 = [1-(1/μ1)] h1 + [1-(1/μ2)] h2

Problem 8(IIT – JEE 2009): A ball is dropped from a height of 20 m above the

surface of water in a lake. The refractive index of water is 4/3. A fish inside the lake, in

the line of fall of the ball, is looking at the ball. At an instant, when the ball is 12.8 m

above the water surface, the fish sees the speed of ball as

(a) 9 ms-1 (b) 12 ms-1 (c) 16 ms-1 (d) 21.33 ms-1

Solution: v = √2gh = √2* 10* 7 = 12ms-1

7.2 m

12.8 m v x

θ

In this case when eye is inside water Xapp = μx

d Xapp / dt = μ(dx/dt) or vapp = μv = (4/3)* 12 = 16 ms-1

Page 244: Geometrical Optics Book With Special Lecture

ANUJ PHYSICAL SCIENCES PHYSICS FOR IIT-JEE /AIEEE/AIPMT

Office: 128/652 “Y” Block Kidwai Nagar Kanpur -11 UP, Website: www.apskanpur.in Mob: 09795647531, 09889787531, 0848664308 E-mail:[email protected]

244

Problem 9(IIT – JEE 2008): A light beam is travelling from region 1 to region 4

(refer figure). The refractive indexes in region 1, 2, 3, 4 are n0. n0 / 2, n0/6, n0/8

respectively. The angle of incidence θ for which the beam just misses entering region 4

is

Region 1 Region 2 Region 3 Region 4

θ n0 / 2 n0 / 6 n0 / 8

n0

0 0.2 m 0.6 m

(a) sin-1 (3/4) (b) sin-1 (1/8) (c) sin-1(1/4) (d) sin -1 (1/3)

Solution: Critical angle from region 3 to region 4 sin θc = (n0/8)/ (n0/6) = ¾

now applying Snell’s law in region 1 and region 3 n0 sin θ = (n0/6) sin θc

sin θ = 1/6 sin θc = (1/6)(3/4) = 1/8

so θ = sin-1 (1/8)

Problem 10 (IIT –JEE 2010): A large glass slab (μ = 5/3) of thickness 8cm is placed

over a point source of light on a plane surface. It is seen that light emerges out of the top

surface of the slab from a circular area of radius R cm. What is the value of R?

(a) 6 cm (b) 7 cm (c) 8 cm (d) 9 cm

Solution: (R/t) tan θc or R = t (tan θc)

R θc

r

θc

But sin θc = 1/μ = 3/5

tan θc = ¾ so R = ¾ t = ¾ (8cm) = 6 cm

Hence the answer is 6.

Page 245: Geometrical Optics Book With Special Lecture

ANUJ PHYSICAL SCIENCES PHYSICS FOR IIT-JEE /AIEEE/AIPMT

Office: 128/652 “Y” Block Kidwai Nagar Kanpur -11 UP, Website: www.apskanpur.in Mob: 09795647531, 09889787531, 0848664308 E-mail:[email protected]

245

Total internal reflection:

Problem 11(IIT – JEE 2010):

O B

600 135

0 C

P 900

A 750 D

A ray OP of monochromatic light is incident on the face AB of prism ABCD near vertex B

at an incident angle of 600. If the refractive index of the material of the prism is√3,

which of the following is (are) correct?

(a) The ray gets totally internally reflected at face CD

(b) The ray comes out through face AD

(c) The angle between the incident ray and the emergent ray is 900

(d) The angle between the incident ray and the emergent ray is 1200

Solution: √3 = sin 600/ sin r so r=300 θc = sin-1(1/√3) sin θc = 1/√3 = 0.577

At point Q, angle of incidence inside the prism is i = 450

Since sin i= 1/√2 is greater than sin θc = 1/√2 , Ray gets totally internally reflected at

face CD. Path of ray of light after point Q is shown in figure.

600 135

0

O p 450 Q

600 r=30

0 45

0

450

900 30

0 75

0

R 600

Page 246: Geometrical Optics Book With Special Lecture

ANUJ PHYSICAL SCIENCES PHYSICS FOR IIT-JEE /AIEEE/AIPMT

Office: 128/652 “Y” Block Kidwai Nagar Kanpur -11 UP, Website: www.apskanpur.in Mob: 09795647531, 09889787531, 0848664308 E-mail:[email protected]

246

From the figure, we can see that angle between incident ray OP and emergent ray RS is

900 therefore, correct options are (a), (b) and (c).

Problem 12 (AIEEE 2009):

θ

A transparent solid cylindrical rod has a refractive index of 2/√3. It is surrounded by

air. A light ray is incident at the mid-point of one end of the rod as shown in the figure.

The incident angle θ for which the light ray grazes along the wall of the rod is

(a) sin-1(1/2) (b) sin-1(√3/2) (c) sin-1(2/√3) (d) sin-1(1/√3)

Solution: sin c = √3/2 (1) sin r= sin (900 - c) = cos c = ½

sin θ/ sin r = μ2/μ1

sin θ =(2/√3)*(1/2)

θ = sin-1(1/√3) :

Page 247: Geometrical Optics Book With Special Lecture

ANUJ PHYSICAL SCIENCES PHYSICS FOR IIT-JEE /AIEEE/AIPMT

Office: 128/652 “Y” Block Kidwai Nagar Kanpur -11 UP, Website: www.apskanpur.in Mob: 09795647531, 09889787531, 0848664308 E-mail:[email protected]

247

Lenses and prism:

Problem 13 (IIT- JEE 2010): A biconvex lens of focal length 15 cm is in front of a

plane mirror. The distance between the lens and the mirror is 10cm. A small object is

kept at a distance of 30 cm from the lens. The final image is

(a) virtual and at a distance of 16 cm from the mirror

(b) real and at a distance of 16 cm from the mirror

(c)virtual and at a distance of 20 cm from the mirror

(d) none of the above

Solution:

10 cm

o l2 l3 l1

6cm

30 cm 10cm 20cm

Object is placed at distance 2f from the lens. So first image I will be formed at distance 2f

on other side. This image I1 will behave like a virtual object for mirror. The second

image I2 will be formed at distance 20 cm in front of the mirror

Problem 14(IIT-JEE 2010): The focal length of a thin biconvex lens is 20 cm. When

an object is moved from a distance of 25 cm in front of it to 50 cm, the magnification of

its image changes from m25 to m50 The ratio m25 / m50 is

(a) 6 (b) 7 (c)8 (d) 9

Solution: (1/v)-(1/u)= 1/f or

(u/v)-1=u/f or (u/v)=(u+f)/f or m=v/u=f/(u+f)

m25/m50 = [20/(-25+20)]/[20/(-50+20)] =6

Problem 15(AIEEE 2009): In an optics experiments, with the position of the object

fixed, a student varies the position of a convex lens and for each position, the screen is

adjusted to get a clear image of the object. A graph between the object distance u and

the image distance v, from the lens, is plotted using the same scale for the two axes. A

straight line passing through the origin and making an angle of 450 with the x-axis

meets the experimental curve at p. The coordinates of p will be

(a) (2f, 2f) (b) ((f/2),(f/2)) (c) (f, f) (d) (4f, 4f)

Page 248: Geometrical Optics Book With Special Lecture

ANUJ PHYSICAL SCIENCES PHYSICS FOR IIT-JEE /AIEEE/AIPMT

Office: 128/652 “Y” Block Kidwai Nagar Kanpur -11 UP, Website: www.apskanpur.in Mob: 09795647531, 09889787531, 0848664308 E-mail:[email protected]

248

Solution:

V

V

U u(u>f)

It is possible when object kept at centre of curvature u=v

u= 2f, v=2f

Problem 16(IIT – JEE 2008): Two beams of red and violet colours are made to pass

separately through a prism (angle of the prism is 600). In the position of minimum

deviation, the angle of refraction will be

(a) 300 for both the colours

(b) greater for the violet colour

(c) greater for the red colour

(d) equal but not 300 for both the colours

Solution: At minimum deviation (δ=δm)

r1=r2 =A/2=600/2 =300 (For both colours)

Problem 17 (AIEEE): A student measure the focal length of a convex lens by putting

an object pin at a distance u from the lens and measuring the distance v of the image

pin. The graph between u and v plotted by the student should look like

V cm v cm

(a) (b)

O u cm o u cm

V cm v cm

(c) (d)

O u cm o u cm

Solution: (1/v) – (1/u) = 1/f = constant, so (c) is correct graph.

Page 249: Geometrical Optics Book With Special Lecture

ANUJ PHYSICAL SCIENCES PHYSICS FOR IIT-JEE /AIEEE/AIPMT

Office: 128/652 “Y” Block Kidwai Nagar Kanpur -11 UP, Website: www.apskanpur.in Mob: 09795647531, 09889787531, 0848664308 E-mail:[email protected]

249

Problem 18(IIT- JEE 2007): In an experiment to determine the focal length (f) of a

concave mirror by the u-v method, a student places the object pin A on the principal axis

at a distance x from the pole p. The student looks at the pin and its inverted image from

a distance keeping his/her eye in line with PA. When the student shifts his/her eye

towards left, the image appears to the right of the object pin. Then

(a) x < f (b) f< x< 2f (c) x = 2f (d) x > 2f

Solution:

Image

Object Right

Since object and image move in opposite directions. The positioning should be as shown

in the figure. Object lies between focus and centre of curvature f < x < 2f

Problem 19 (AIEEE 2007): Two lenses of power - 15D and + 5D are in contact with

each other. The focal length of the combination is

(a) -20 cm (b) -10 cm (c) + 20 cm (d) + 10 cm

Solution: power of a lens is reciprocal of its focal length. Power of combined lens is

p = p1+p2 = - 15 + 5 = -10 D so f = 1/p = 100/(-10) cm

f = - 10 cm

Problem 20 (IIT – JEE 2006): A point object is placed at distance of 20 cm from a

thin Plano convex lens of focal length 15 cm. The plane surface of the lens is now

silvered. The image created by the system is at

(a) 60 cm to the left of the system

(b) 60 cm to the right of the system

(c) 12 cm to the left of the system

(d) 12 cm to the right of the system

Page 250: Geometrical Optics Book With Special Lecture

ANUJ PHYSICAL SCIENCES PHYSICS FOR IIT-JEE /AIEEE/AIPMT

Office: 128/652 “Y” Block Kidwai Nagar Kanpur -11 UP, Website: www.apskanpur.in Mob: 09795647531, 09889787531, 0848664308 E-mail:[email protected]

250

20 cm

Solution: Refraction from lens

(1/v1)-(1/-20) = 1/15

v = 60 cm +ive direction

i.e. first image is formed at 60 cm to the right of lens system.

Reflection from mirror

After reflection from the mirror, the second image will be formed at a distance of 60 cm

to the left of lens system.

Refraction from lens (1/v) – (1/60) = (1/15) +ive direction or v3 = 12 cm

therefore, the final image is formed at 12 cm to the left of the lens system.

Problem 21(IIT- JEE 2006): The graph between object distance u and image

V

+11

+10

+9 450

-9 - 10 - 11 u

Distance v for lens is given. The focal length of the lens is

(a) 5 ± 0.1 (b) 5 ± 0.05 (c) 0.5 ± 0.1 (d) 0.5 ± 0.05

Solution: from the lens formula: 1/f = (1/v) – (1/u) we have

1/f = (1/10) – (1/-10) or f = +5

further Δu = 0.1 and Δv = 0.1 (from the graph)

now differentiating the lens formula we have,

(Δf/f2) = (Δv/v2)+( Δu/u2) or Δf = [(Δv/v2)+( Δu/u2)]f2

substituting the value we have Δf = [(0.1/102)+(0.1/102)] (5)2 = 0.05

so f ± Δf = 5±0.05

Page 251: Geometrical Optics Book With Special Lecture

ANUJ PHYSICAL SCIENCES PHYSICS FOR IIT-JEE /AIEEE/AIPMT

Office: 128/652 “Y” Block Kidwai Nagar Kanpur -11 UP, Website: www.apskanpur.in Mob: 09795647531, 09889787531, 0848664308 E-mail:[email protected]

251

Problem 22 (IIT-JEE 2006): A biconvex lens of focal length f forms a circular image

of radius r of sun in focal plane. Then which option is correct?

(a) πr2 =f

(b) πr2 =f2

(c) if lower half part is covered by black sheet, then area of the image is equal to πr2 /2

(d) if f is doubled, intensity will increase

Solution: r = f tan θ or r=f

so πr2 =f2

Problem 23 (IIT – JEE 2005): A convex lens is in contact with concave lens. The

magnitude of the ratio of their focal length is 2/3. Their equivalent focal length is 30 cm.

What are their individual focal lengths?

(a) -75, 50 (b) -10, 15 (c) 75, 50 (d) -15, 10

Solution: Let focal length of convex lens is +f then focal length of concave lens would

be (-3/2) f. From the given condition,

1/30 = (1/f ) – (2/3f) = 1/3f

f = 10 cm Therefore, focal length of convex lens=+10 cm and that of concave lens

= -15 cm.

Problem 24 (AIEEE 2005): A thin glass (refractive index 1.5) lens has optical

power in a liquid medium with refractive index 1.6 will be

(a) 1D (b) -1D (c) 25D (d) -25D

Solution: (1/f0) = (μ-1)[(1/R1)-(1/R2)] =(1.5-1) [(1/R1)-(1/R2)] (1)

1/fm =[(μg – μm)/ μm] [(1/R1)-(1/R2)]

1/fm = [(1.5/1.6)-1] [(1/R1)-(1/R2)] (2)

fm/f0 =(1.5-1) / [(1.5/1.6)-1] = -8

fm = -8* f0 = -8*(-1/5) =1.6 m (∵ f0 = 1/p = (-1/5)m)

pm= μ/ fm = 1.6/1.6 = 1D

Page 252: Geometrical Optics Book With Special Lecture

ANUJ PHYSICAL SCIENCES PHYSICS FOR IIT-JEE /AIEEE/AIPMT

Office: 128/652 “Y” Block Kidwai Nagar Kanpur -11 UP, Website: www.apskanpur.in Mob: 09795647531, 09889787531, 0848664308 E-mail:[email protected]

252

Problem 25 (IIT – JEE 2004):

Q R

P S

A ray of light is incident on an equilateral glass prism placed on a horizontal table. For

minimum deviation which of the following is true?

(a) PQ is horizontal

(b) QR is horizontal

(c) RS is horizontal

(d) Either PQ or RS is horizontal

Solution: during minimum deviation the ray inside the prism is parallel to the base of

the prism in case of an equilateral prism.

Problem 26 (AIEEE 2004): A light ray is incident perpendicular to one face of a

900 prism and is totally internally reflected at the glass-air interface. If the angle of

reflection is 450, we conclude that the refractive index n

(a) n< 1/√2 (b) n > √2 (c) n> 1/√2 (d) n< √2

450

450

Solution: For total internal reflection from glass-air interface, critical angle c must be

less than angle of incidence.

C< i or C < 450 (∵∠i= 450)

but n= 1/ sin c ⇒ c = sin -1(1/n)

∴ sin-1 (1/n) < 450

1/n < sin 450 ⇒ n > 1/ sin 450 n > 1/(1/√2) ⇒ n > √2

Page 253: Geometrical Optics Book With Special Lecture

ANUJ PHYSICAL SCIENCES PHYSICS FOR IIT-JEE /AIEEE/AIPMT

Office: 128/652 “Y” Block Kidwai Nagar Kanpur -11 UP, Website: www.apskanpur.in Mob: 09795647531, 09889787531, 0848664308 E-mail:[email protected]

253

Problem 27(IIT – JEE 2003): The size of the image of an object, which is at infinity,

as formed by a convex lens of focal length 30 cm is 2 cm. If a concave lens of focal length

20 cm is placed between the convex lens and the image at a distance of 26 cm from the

convex lens, calculate the new size of the image

(a) 1.25 cm (b) 2.5 cm (c) 1.05 cm (d) 2cm

Solution:

5 cm

l1

28cm 4 cm l 2

(1/v) – (1/u) = 1/f or (1/v) – ¼ = 1/ -20 or v = 5 cm

magnification for concave lens m = v/u = 5/4 = 1.25

As size of the image at I1 is 2 cm. Therefore, size of image at I2 will be 2* 1.25 = 2.5 cm

Problem 28 (AIEEE 2011): When monochromatic red light is used instead of blue

light in a convex lens, its focal length will

(a) does not depend on colours of light

(b) increase

(c) decrease

(d) remain same

Solution: 1/f =(μ-1)[(1/R1)-(1/R2)]

also, by Cauchy’s formula μ=A+(B/λ2)+(c/λ4)+.......

λblue < λred so μblue > μred hence, fred > fblue

Page 254: Geometrical Optics Book With Special Lecture

ANUJ PHYSICAL SCIENCES PHYSICS FOR IIT-JEE /AIEEE/AIPMT

Office: 128/652 “Y” Block Kidwai Nagar Kanpur -11 UP, Website: www.apskanpur.in Mob: 09795647531, 09889787531, 0848664308 E-mail:[email protected]

254

Optical instrument

Problem 29 (AIEEE 2008): An experiment is performed to find the refractive index

of glass using a travelling microscope. In this experiment distance are measured by

(a) a vernier scale provided on the microscope

(b) a standard laboratory scale

(c) a meter scale provided on the microscope

(d) a screw gauge provided on the microscope

Solution: answer (a) explanation not given

Problem 30 (AIEEE 2005): Two point white does are 1 mm apart on a black paper.

They are viewed by eye of pupil diameter 3 mm. Approximately, what is the maximum

distance at which these dots can be resolved by the eye?

[Take wavelength of light = 500nm]

(a) 5m (b) 1m (c) 6m (d) 3m

Solution: We know (y/D) ≥ 1.22 (λ/d)

D ≤ (yd)/1.22λ = (10-3 *3*10-3)/ 1.22*5*10-7 = 30/6.1 = 5m

∴ Dmax = 5m

Problem 31(AIEEE 2003): The image formed by an objective of a compound

microscope is

(a) virtual and diminished

(b) real and diminished

(c) real and enlarged

(d) virtual and enlarged

Solution: objective of compound microscope is a convex lens. Convex lens forms real

and enlarged image when an object is placed between its focus and lens.

Page 255: Geometrical Optics Book With Special Lecture

ANUJ PHYSICAL SCIENCES PHYSICS FOR IIT-JEE /AIEEE/AIPMT

Office: 128/652 “Y” Block Kidwai Nagar Kanpur -11 UP, Website: www.apskanpur.in Mob: 09795647531, 09889787531, 0848664308 E-mail:[email protected]

255

How to Solve IIT-JEE Physics in few seconds?

Special Lecture:

(METHOD OF EXTREME CASES)

Idea of Applicability:

The idea pointed out in this article is very useful, when one is interested in arriving at the

answer to a question, rather than the steps involved in arriving thereat.

I have been involved in teaching IIT-JEE Physics for last 10 years. My experience tells that

the class of question that can be handled in this fashion is very large. The only limitations

are that the questions must be of Multiple Choice Questions and should not involve the

Numerical values in the choices provided. It is very easy to see that the range of

applicability is wide and that it covers a big percentage of the questions in any multiple

choice type examinations.

To demonstrate the idea best, it will be presented with help of various examples. Also it

applies to a better extent in Mathematics Multiple Choice Type Questions examinations.

(Just as my teaching experience goes in Physics, I am not pointing out examples from

Mathematics. Neither would the place here be sufficient to cover all types of examples. It

remains for the students to extend the idea in their own way because my experience tells

the brain of students is too much sharper than the brain of teachers).

I am giving very few examples here, but it will make the idea clear. The given examples are

only the representative of the ideas employed in this approach, which is being phrased as

the ‘Method of extreme cases’. My advice is that when you practice objective questions;

apply these tricks as much as possible.

(Anuj Dubey)

(A Physics Faculty for IIT-JEE)

Page 256: Geometrical Optics Book With Special Lecture

ANUJ PHYSICAL SCIENCES PHYSICS FOR IIT-JEE /AIEEE/AIPMT

Office: 128/652 “Y” Block Kidwai Nagar Kanpur -11 UP, Website: www.apskanpur.in Mob: 09795647531, 09889787531, 0848664308 E-mail:[email protected]

256

(Problems on Method of Extreme Cases)

Q1. See fig, the figure shows a uniform annulus of internal radii r & external radii R

respectively. The mass of portion in between is M. Find the moment of inertia of this

annulus about a centroid & perpendicular axis, the choices provided here,

A) ½ M (R2-r2) B) ½ M (R2+r2) C) M (R2-r2) D) M (R2+r2)

Q2. There is a pendulum hanging in a vehicle, which is accelerating horizontally with a

constant acceleration a (gravity is of course g). The pendulum gets inclined backwards.

Its equilibrium position makes an angle α with the vertical axis, where αis given by one of

the four choices

A) sin-1(g/a) B) cos-1(a/g) C) tan-1(a/g) D) tan-1(g/a)

Q3. There is a jeep “hauling up” a bucket of water. The height of the pulley is h while the

horizontal distance of jeep from the point below the pulley is x.

What is the acceleration of the bucket?

A) (hu)2 / (h2+x2)3/2 B) (xu)2 / ( h2+x2)3/2

C) (hu)2 / ( h2+x2)5/2 D) (u)2 / 2( h2+x2)1/2

Q4. A Particle is projected at an angle of elevation α. After t seconds, the angle of elevation

is β as seen from the point of projection. The initial velocity will be

A) gt/ 2 sin (α-β) B) gt cosβ/2 sin (α-β)

C) sin (α-β) /2gt D) 2 sin (α-β) / gt cosβ

Q5. From IIT-JEE Screening 2001: A coil having N turns is wound tightly on the form of a

spiral with inner 7 outer radii ‘a’ & ‘b’ respectively. When a current I pass through the coil,

the magnetic field at the center is

A) μoNI/a B) μoNI/b C) μoNI log {(b/a)/(b-a)} D) μoINlog {(b/a)/(b-a)}

Q6. IIT-JEE 2000 Screening: There is a rectangular slab of refractive index µ1 which is

immersed in water of refractive index µ2 (µ1 > µ2). A ray of light is incident on the surface

AB of the slab as shown in fig. The max value of angle of incidence αmax such that the ray

comes only from the other surface CD is given by-

A) sin-1[(µ1/µ2) cos {sin-1((µ2/µ1)}] B) sin-1(µ1/µ2)

C) sin-1[µ1 cos {sin-1(1/µ2)] D) sin-1(µ2/µ1)

Page 257: Geometrical Optics Book With Special Lecture

ANUJ PHYSICAL SCIENCES PHYSICS FOR IIT-JEE /AIEEE/AIPMT

Office: 128/652 “Y” Block Kidwai Nagar Kanpur -11 UP, Website: www.apskanpur.in Mob: 09795647531, 09889787531, 0848664308 E-mail:[email protected]

257

(Solutions of Examples on Method of Extreme Cases)

Q1. See fig, the figure shows a uniform annulus of internal radii r & external radii R

respectively. The mass of portion in between is M. Find the moment of inertia of this

annulus about a centroid & perpendicular axis, the choices provided here,

A) ½ M (R2-r2) B) ½ M (R2+r2) C) M (R2-r2) D) M (R2+r2)

Soln. IDEA:

The only knowledge is required here is that the moment of inertia of a ring and disc

about the standard axis are respectively MR2 & 1/2 MR2.

In the annulus, and here, in four choices provided above, put the extreme case

r=0, by doing so, the annulus will turn into a disc and relevant formula should

transform to ½ MR2.

Put the other extreme case r=R, This will transform

the annulus into a ring and the answer should change

to MR2.

APPLICATION OF IDEA:

M. I. OF DISC=1/2 MR2 If r=0 (1ST EXTREME)

M.I. OF RING= MR2 If r=R (2ND EXTREME)

OPTION (A) ½ M (R2-r2)

IF r=0 (1ST EXTREME) Then M.I. = 1/2 MR2 (RIGHT)

IF r=R (2ND EXTREME) Then M.I. =0 (WRONG)

OPTION (B) ½ M (R2 +r2)

IF r=0 (1ST EXTREME) Then M.I. = 1/2 MR2 (RIGHT) IF r=R (2ND EXTREME) Then M.I. =MR2 (RIGHT)

OPTION(C) M (R2-r2)

IF r=0 (1ST EXTREME) Then M.I. = MR2 (WRONG) IF r=R (2ND EXTREME) Then M.I. =0 (WRONG)

OPTION (D) M (R2+r2)

IF r=0 (1ST EXTREME) Then M.I. = MR2 (WRONG) IF r=R (2ND EXTREME) Then M.I. =2MR2 (WRONG)

Hence answer is

option (B) because

this satisfy only both

extreme cases,

It’s very easy, you see.

Now, I hope you

understood the

concept of extreme

cases.

Ok, if there is doubt in

understanding the

concept then I am

giving more examples.

Page 258: Geometrical Optics Book With Special Lecture

ANUJ PHYSICAL SCIENCES PHYSICS FOR IIT-JEE /AIEEE/AIPMT

Office: 128/652 “Y” Block Kidwai Nagar Kanpur -11 UP, Website: www.apskanpur.in Mob: 09795647531, 09889787531, 0848664308 E-mail:[email protected]

258

Q2. There is a pendulum hanging in a vehicle, which is accelerating horizontally with a

constant acceleration a (gravity is of course g). The pendulum gets inclined backwards. Its

equilibrium position makes an angle α with the vertical axis, where αis given by one of the

four choices

A) sin-1(g/a) B) cos-1(a/g) C) tan-1(a/g) D) tan-1(g/a)

a

g α

Soln. Idea: Put the extreme case a=0, α in this case must turn out to be zero.

A) α = sin-1(g/a) put a=0 , then we get α = sin-1(∞)

Which is an Absurd answer, has no physical meaning because the value of sin

α lies between -1 to +1.

B) α = cos-1(a/g) put a=0 , then we get α =Cos -1(0) which gives α=π/2

C) α = tan-1(a/g) put a=0 , then we get α =tan -1(0) which gives α=0

D) α = tan-1(g/a) put a=0 , then we get α =tan -1(∞) which gives α= π/2

Hence the choice (c) is only satisfying the extreme so correct option is (c).

Page 259: Geometrical Optics Book With Special Lecture

ANUJ PHYSICAL SCIENCES PHYSICS FOR IIT-JEE /AIEEE/AIPMT

Office: 128/652 “Y” Block Kidwai Nagar Kanpur -11 UP, Website: www.apskanpur.in Mob: 09795647531, 09889787531, 0848664308 E-mail:[email protected]

259

Q3. There is a jeep “hauling up” a bucket of water. The height of the pulley is h while the

horizontal distance of jeep from the point below the pulley is x. What is the acceleration

of the bucket?

A) (hu)2 / ( h2+x2)3/2 B) (xu)2 / ( h2+x2)3/2

C) (hu)2 / ( h2+x2)5/2 D) (u)2 / 2( h2+x2)1/2

Jeep B

X

H

U

Soln. Idea: Put the extreme case X=0, the value of initial acceleration should turn out to

be (u2/h) which is the formula of centripetal acceleration

A) (hu)2 / ( h2+x2)3/2 Put X=0 We get a=u2/h

B) (xu)2 / ( h2+x2)3/2 Put X=0 We get a=0

C) (hu)2 / (

h2+x2)5/2 Put X=0 We get a= u2/h3

(dimensionally wrong)

D) (u)2 / 2( h2+x2)1/2 Put X=0 We get a= u2/2h

Hence the answer should be (A) and this indeed is correct.

Page 260: Geometrical Optics Book With Special Lecture

ANUJ PHYSICAL SCIENCES PHYSICS FOR IIT-JEE /AIEEE/AIPMT

Office: 128/652 “Y” Block Kidwai Nagar Kanpur -11 UP, Website: www.apskanpur.in Mob: 09795647531, 09889787531, 0848664308 E-mail:[email protected]

260

Q4. A Particle is projected at an angle of elevation α. After t seconds, the angle of elevation is β as seen from the point of projection. The initial velocity will be A) gt/ 2 sin (α-β) B) gt cosβ/2 sin (α-β) C) sin (α-β) /2gt D) 2 sin (α-β) / gt cosβ

O

P

U

β

Soln. Idea. The choices (C) & (D) are dimensionally incorrect. So choice remains to select between (A) & (B). Now we are going to apply here method of extreme case. IST extreme case: Consider the case when β = 0 A) gt/ 2 sin (α-β) we get t=2usinα/g B) gt cosβ/2 sin (α-β) we get t=2usinα/g By using the extreme case you got same result in both options, so I hope u are confused, but now no need to worry apply another extreme case. 2nd extreme case: Consider an extreme in this situation when β= -90, & α = 0 A) u = gt/ 2 sin (α-β) we get, u = gt/2 B) u = gt cosβ/2 sin (α-β) we get u=0. (This is similar as Motion under gravity) Hence (B) is correct. Under suitable limits a finite answer.

Page 261: Geometrical Optics Book With Special Lecture

ANUJ PHYSICAL SCIENCES PHYSICS FOR IIT-JEE /AIEEE/AIPMT

Office: 128/652 “Y” Block Kidwai Nagar Kanpur -11 UP, Website: www.apskanpur.in Mob: 09795647531, 09889787531, 0848664308 E-mail:[email protected]

261

Q5. From IIT-JEE Screening 2001: A coil having N turns is wound tightly on the form

of a spiral with inner 7 outer radii ‘a’ & ‘b’ respectively. When a current I pass through

the coil, the magnetic field at the center is

A) μoNI/a B) μoNI/b C) μoNI log {(b/a)/(b-a)} D) μoINlog {(b/a)/(b-a)}

Sol. It is obvious that the answer has to depend on both a & b. Hence option (A) & (B)

are ruled out.

Dimensionally B= μoNI/length Hence option (D) is ruled out too, because it has N

powers of current.

And the answer has to be the only choice left after these eliminations i.e. (C).

NOTE: in option (C) if b tends to a applying limit concept, we can get B= µo N I/2a,

which is the expected answer as the spiral will become a circular loop of N turns.

Q6. IIT-JEE 2000 Screening See Fig. There is a rectangular slab of refractive index µ1

which is immersed in water of refractive index µ2 (µ1 > µ2). A ray of light is incident on

the surface AB of the slab as shown in fig. The max value of angle of incidence αmax such

that the ray comes only from the other surface CD is given by-

A) sin-1[(µ1/µ2) cos {sin-1((µ2/µ1)}]

B) sin-1(µ1/µ2)

C) sin-1[µ1 cos {sin-1(1/µ2)]

D) sin-1(µ2/µ1)

μ1 > μ2

A C

B D

αmax

Sol. Idea. Apply extreme case, Let µ1= µ2 = µ Then αmax = 0. (Must be)

µ1 > µ2, Therefore µ1/ µ2 >1 & µ2/ µ1 < 1

Page 262: Geometrical Optics Book With Special Lecture

ANUJ PHYSICAL SCIENCES PHYSICS FOR IIT-JEE /AIEEE/AIPMT

Office: 128/652 “Y” Block Kidwai Nagar Kanpur -11 UP, Website: www.apskanpur.in Mob: 09795647531, 09889787531, 0848664308 E-mail:[email protected]

262

)(sin2

11

x )(sincos1

21

x

x

)(sin

1&1

2

11

max

2

1

)}]1

(cos{sin[sin2

1

1

1

)}/1(sincos{sin 11

max xx

)(sin1

21

As we know, sinθ > 1 impossible & cosθ > 1 impossible.

Option (a) sin-1[(µ1/µ2) cos {sin-1((µ2/µ1)}]

sin-1((µ2/µ1) Possible value because (µ2/µ1)< 1

This is less than 1.

So it is less than 90o

Hence option (a) is possible.

Option (b) αmax = which is meaningless sin-1 (x), where x>1.

Option(c) αmax =

If µ1= µ2, which gives non-zero result, hence Option(c) is impossible.

Option (d) αmax =

If µ1= µ2, αmax = sin-1 (1) = 90o

Which gives non-zero result; hence Option (d) is impossible.

Hence Option (a) is only correct.

Page 263: Geometrical Optics Book With Special Lecture

ANUJ PHYSICAL SCIENCES PHYSICS FOR IIT-JEE /AIEEE/AIPMT

Office: 128/652 “Y” Block Kidwai Nagar Kanpur -11 UP, Website: www.apskanpur.in Mob: 09795647531, 09889787531, 0848664308 E-mail:[email protected]

263

Now you can analysis yourself what is the importance of this approach of “Method of

extreme case”. Infact this approach of extreme case and the corresponding method of

elimination of the choices, does not require more than a few seconds to tell the answers,

sometimes the solution is even shorter than the questions and this happens in most of the

cases.

Few serious students after solving the questions may ask me sir in problem no 5. you solved

the answers by elimination methods but how I really believe in this answer, so my reply is”

if method of extreme case fails then we can apply the first order approximation methods”.

My next lecture on this series

How to Solve IIT-JEE Physics in few seconds?

“First order approximation methods”

Problems for Practice:

1. From the top of a tower, a stone is thrown up and it reaches the ground in time t1.

A second stone is thrown down with the same speed and it reaches the ground in

time t2. A third stone is released from rest and it reaches the ground in time t3.

a) t3=(t1t2)1/2 b) t3=(t1+t2)/2 c) t32=(t12-t22) d) 1/t3=1/t2-1/t1

2. A particle is thrown with a speed u at an angle α to the horizontal, when it makes

an angle β with the horizontal, its speed becomes v, where

a) v=u cos α b) v=u cos α cos β c) v=u cos α sec β d) v=u sec α cos β

3. A beaker containing a liquid of density ρ moves up with an acceleration a , the

pressure due to the liquid at a depth h below the free surface of the liquid is

a) hρg b) hρ(g+a) c) hρ(g-a) d) hρg{ (g-a)/(g+a) }

4. A sample of radioactive material has mass m, decay constant λ and molecular

weight M. Avogadro constant= NA . The activity of the sample after time t is

a) (m NA/M) e- λt b) (m NA λ /M) e- λt c) (m NA /M λ) e- λt d) m / λ ( -e- λt )

5. A block of mass m slides down an inclined plane which makes an angle α with the

horizontal. The coefficient of friction between the block and the plane is μ. The force

exerted by the block on the plane is

a) mg cos α b) (μ2+1 )1/2 mg cos α c) μ mg cos α/(μ2+1 )1/2 d) μ mg cos α

6. A stationary swimmer S, inside the liquid of refractive index μ1 is at a distance d

from a fixed point P inside the liquid. A rectangular block of width t and refractive

index μ2(< μ1)is now placed between S and P. S will observe Pat a distance of

a) d-t{ (μ1/ μ2) -1)} b) d-t{1- (μ2/ μ1) } c) d+t{1- (μ2/ μ1) } d) d+t{ (μ1/ μ2) -1)}

Page 264: Geometrical Optics Book With Special Lecture

ANUJ PHYSICAL SCIENCES PHYSICS FOR IIT-JEE /AIEEE/AIPMT

Office: 128/652 “Y” Block Kidwai Nagar Kanpur -11 UP, Website: www.apskanpur.in Mob: 09795647531, 09889787531, 0848664308 E-mail:[email protected]

264